You are on page 1of 262

Question

ICAP
Bank

Financial Accounting
and Reporting II
Sixth edition published by
Emile Woolf Limited
Bracknell Enterprise & Innovation Hub
Ocean House, 12th Floor, The Ring
Bracknell, Berkshire, RG12 1AX United Kingdom
Email: info@ewiglobal.com
www.emilewoolf.com

© Emile Woolf International, February 2019

All rights reserved. No part of this publication may be reproduced, stored in a retrieval system, or transmitted,
in any form or by any means, electronic, mechanical, photocopying, recording, scanning or otherwise, without
the prior permission in writing of Emile Woolf Publishing Limited, or as expressly permitted by law, or under
the terms agreed with the appropriate reprographics rights organisation.

You must not circulate this book in any other binding or cover and you must impose the same condition on
any acquirer.

Notice
Emile Woolf International has made every effort to ensure that at the time of writing the contents of this study
text are accurate, but neither Emile Woolf International nor its directors or employees shall be under any
liability whatsoever for any inaccurate or misleading information this work could contain.

© Emile Woolf International ii The Institute of Chartered Accountants of Pakistan


Certificate in Accounting and Finance

C
Financial accounting and reporting II

Contents
Page

Question and Answers Index v


Questions
Section A Questions 1
Answers
Section B Answers 95

© Emile Woolf International iii The Institute of Chartered Accountants of Pakistan


Financial accounting and reporting II

© Emile Woolf International iv The Institute of Chartered Accountants of Pakistan


Certificate in Accounting and Finance

I
Financial accounting and reporting II

Index to questions and answers

Question Answer
page page

CHAPTER 2 – IAS 1: PRESENTATION OF FINANCIAL STATEMENTS

2.1 LARRY 1 96

2.2 MINGORA IMPORTS LIMITED 2 97

2.3 BARRY 3 99

2.4 OSCAR INC 4 101

2.5 CLIFTON PHARMA LIMITED 5 103

2.6 SARHAD SUGAR LIMITED 7 105

2.7 BSZ LIMITED 8 107

2.8 YASIR INDUSTRIES LIMITED 10 110

2.9 SHAHEEN LIMITED 11 114

2.10 MOONLIGHT PAKISTAN LIMITED 12 116

2.11 FIGS PAKISTAN LIMITED 14 118

2.12 WAH AGRIPROD LTD 15 121

CHAPTER 3 - CONSOLIDATED ACCOUNTS: STATEMENT OF FINANCIAL


POSITION - BASIC APPROACH

3.1 P AND S (I) 17 123

3.2 P AND S (II) 17 124

© Emile Woolf International v The Institute of Chartered Accountants of Pakistan


Financial accounting and reporting II

Question Answer
page page

3.3 BL AND FL 18 125

3.4 ML AND ZL 18 126

3.5 FLAMSTEED LTD AND HALLEY LTD 19 127

CHAPTER 4 - CONSOLIDATED ACCOUNTS: STATEMENT OF FINANCIAL


POSITION – COMPLICATIONS

4.1 HAIL 21 129

4.2 HAIRY 22 130

4.3 HARD 23 132

4.4 HALE 23 134

4.5 HELLO 24 135

4.6 HASAN LIMITED 25 136

4.7 GOLDEN LIMITED 26 140

4.8 YASIR LIMITED AND BILAL LIMITED 27 141

4.9 JASMEEN LIMITED 28 143

4.10 BRADLEY LTD 29 145

CHAPTER 5 - CONSOLIDATED ACCOUNTS: STATEMENT OF


COMPREHENSIVE INCOME

5.1 HARRY 31 147

5.2 HORNY 32 148

5.3 HANKS 32 150

5.4 OSCAR LTD 34 153

5.5 PRESENT LIMITED AND FUTURE LIMITED 35 155

5.6 FATIMA LIMTED AND ALI LIMITED 36 157

CHAPTER 6 - IAS 28: INVESTMENT IN ASSOCIATES AND JOINT VENTURES

6.1 HELIUM 38 159

6.2 HAMACHI LTD 38 160

6.3 HIDE 40 163

© Emile Woolf International vi The Institute of Chartered Accountants of Pakistan


Index to questions and answers

Question Answer
page page

6.4 HARK, SPARK AND ARK 40 164

6.5 P, S AND A 42 167

6.6 BL, ML AND ZL 43 169

6.7 MANTO LIMITED AND HALI LIMITED 45 171

CHAPTER 7 - IAS 8: ACCOUNTING POLICIES, CHANGES IN ACCOUNTING


ESTIMATES AND ERRORS

7.1 WONDER LIMITED 47 174

7.2 MOHANI MANUFACTURING LIMITED 48 175

7.3 MARVELLOUS LIMITED 48 176

7.4 CHAND PAINTS LIMITED 49 178

7.5 DAFFODIL LIMITED 50 179

7.6 ASIF ENGINEERING LIMITED 50 180

CHAPTER 8 - IAS 12: INCOME TAXES

8.1 FRANCESCA 52 182

8.2 SHEP (I) 52 182

8.3 SHEP (II) 53 183

8.4 SHEP (III) 53 185

8.5 SHEP (IV) 54 186

8.6 WAQAR LIMITED 54 187

8.7 SHAKIR INDUSTRIES 55 189

8.8 MARS LIMITED 56 190

8.9 BILAL ENGINEERING LIMITED 57 192

8.10 GALAXY INTERNATIONAL 57 194

8.11 APRICOT LIMITED 58 195

8.12 ROSE LIMITED 58 196

8.13 FLOOR AND TILES LIMITED 59 197

© Emile Woolf International vii The Institute of Chartered Accountants of Pakistan


Financial accounting and reporting II

Question Answer
page page

8.14 HADI LIMITED 60 199

CHAPTER 9 - IAS 21: FOREIGN CURRENCY TRANSACTIONS

9.1 DND LIMITED 62 202

9.2 ORLANDO 62 202

9.3 OMEGA LIMITED 63 203

9.4 KANGAROO LIMITED 63 204

9.5 MZA LIMITED 64 205

CHAPTER 10 - IAS 38: INTANGIBLE ASSETS

10.1 FAZAL 65 207

10.2 HENRY 65 207

10.3 TOBY 65 208

10.4 BROOKLYN 66 209

10.5 ZOUQ INC 66 210

10.6 RAISIN INTERNATIONAL 67 211

10.7 INTERNATIONAL ASSOCIATES LIMITED 68 212

10.8 TULIP LIMITED 69 213

10.9 ZEBRA LIMITED 69 213

10.10 SUNSHINE LIMITED 70 213

CHAPTER 11 - IAS 41: AGRICULTURE

11.1 GUJRANWALA FOODS LIMITED 71 214

11.2 HELIOS GROUP 72 215

11.3 THE DAIRY COMPANY 73 216

11.4 FATIMA LIMITED 73 217

11.5 ZOHA LIMITED 73 217

11.6 MISHALL LIMITED 73 218

© Emile Woolf International viii The Institute of Chartered Accountants of Pakistan


Index to questions and answers

Question Answer
page page

CHAPTER 12 - IFRS 9: FINANCIAL INSTRUMENTS: RECOGNITION AND


MEASUREMENT

12.1 AJI PANCA LTD 75 219

12.2 PASSILA LTD 75 220

12.3 RASHID INDUSTRIES LIMITED 76 220

CHAPTER 13 - IFRS 16: LEASES

13.1 X LTD 77 222

13.2 PROGRESS LTD 77 222

13.3 MIRACLE TEXTILE LIMITED 77 224

13.4 ACACIA LTD 78 225

13.5 SHOAIB LEASING LIMITED 78 225

13.6 SHALIMAR INDUSTRIES LIMITED 78 227

13.7 GUAVA LEASING LIMITED (GLL) 79 228

CHAPTER 14 – OTHER AREAS OF IFRSs (IFRS 8, IAS 10, IAS 37)

14.1 BADAR 80 229

14.2 GEORGINA 80 229

14.3 EARLEY INC 80 230

14.4 ACCOUNTING TREATMENT 81 231

14.5 J-MARK LIMITED 81 232

14.6 AKBER CHEMICALS LIMITED 82 233

14.7 QALLAT INDUSTRIES LIMITED 82 234

14.8 SKYLINE LIMITED 83 234

14.9 WALNUT LIMITED 84 235

14.10 ATTOCK TECHNOLOGIES LIMITED 84 235

14.11 SHAZAD INDUSTRIES LIMITED 85 236

14.12 AZ 85 237

© Emile Woolf International ix The Institute of Chartered Accountants of Pakistan


Financial accounting and reporting II

Question Answer
page page

14.13 ROWSLEY 85 237

14.14 MULTAN PETROCHEM LTD 86 239

14.15 VOILET POWER LIMITED 87 240

14.16 GOHAR LIMITED 87 240

14.17 JAY LIMITED 88 242

14.18 GLADIATOR LIMITED 88 242

14.19 BRAVO LIMITED 89 243

14.20 WASTE MANAGEMENT LIMITED 89 243

CHAPTER 15 - ETHICAL ISSUES IN FINANCIAL REPORTING

15.1 ETHICAL ISSUES 90 245

15.2 SINDH INDUSTRIES LTD 90 245

15.3 CHARMING LIMITED 91 248

15.4 MEHRAN LIMITED 92 249

15.5 MNZ LIMITED 92 249

15.6 UNIQUE ENGINEERING LIMITED 92 250

15.7 ALI AND BASHIR 93 250

15.8 FARAZ 94 251

© Emile Woolf International x The Institute of Chartered Accountants of Pakistan


Certificate in Accounting and Finance

A
Financial accounting and reporting II

SECTION
Questions
CHAPTER 1 – LEGAL BACKGROUND TO THE PREPARATION OF FINANCIAL
STATEMENTS
There are no questions specific to chapter one because the learning outcomes in this area concern the
preparation of financial statements. The relevant questions have been given in chapter 2 of this question bank.

CHAPTER 2 – IAS 1: PRESENTATION OF FINANCIAL STATEMENTS


2.1 LARRY
The trial balance of Larry at 31 December 2015 is as follows.
Rupees in million
Dr Cr
Administration charges 342
Bank account 89
Cash 2
Payables’ ledger 86
Accumulated amortisation on patents at 31 December 2015 5
Accumulated depreciation at 31 December 2015 918
Receivables’ ledger 189
Distribution expenses 175
Property, plant and equipment at cost 2,830
Interest received 20
Issued share capital 400
Loan 18
Patents at cost 26
Accumulated profits 1,562
Purchases 2,542
Sales 3,304
Inventories at 31 December 2014 118
6,313 6,313

© Emile Woolf International 1 The Institute of Chartered Accountants of Pakistan


Financial accounting and reporting II

The following information is also relevant.


(1) Inventories on 31 December 2015 amounted to Rs. 127 million.
(2) Current tax of Rs. 75 million is to be provided.
(3) The loan is repayable by equal annual instalments over three years.
Required
Prepare an statement of profit or loss (analysing expenses by function) for the year ended 31
December 2015 and a statement of financial position as at that date.

2.2 MINGORA IMPORTS LIMITED


The trial balance of Mingora Imports Limited at 31 December 2015 is as follows.
Rupees in million
Dr Cr
Patent rights 60
Work-in-progress, 1 January 2015 125
Leasehold buildings at cost 300
Ordinary share capital 600
Sales 1,740
Staff costs 260
Accumulated depreciation on buildings, 1 January 2015 60
Inventories of finished games, 1 January 2015 155
Consultancy fees 44
Directors’ salaries 360
Computers at cost 50
Accumulated depreciation on computers, 1 January 2015 20
Dividends paid 125
Cash 440
Receivables 420
Trade payables 92
Sundry expenses 294
Accumulated profits, 1 January 2015 121
2,633 2,633

The following information is also relevant.


(1) Closing inventories of finished goods are valued at Rs. 180 million. Work-in-progress has
increased to Rs. 140 million.
(2) The patent rights relate to a computer program with a three year lifespan.
(3) On 1 January 2015 buildings were revalued to Rs. 360 million. This has not yet been reflected
in the accounts. Computers are depreciated over five years. Buildings are now to be
depreciated over 30 years.
(4) An allowance for bad debts (irrecoverable debts) of 5% is to be created.
(5) There is an estimated bill for current tax of Rs. 120 million which has not yet been recognised.
Required
Prepare an statement of profit or loss (analysing expenses by nature) for the year ended 31 December
2015 and a statement of financial position as at that date.

© Emile Woolf International 2 The Institute of Chartered Accountants of Pakistan


Questions

2.3 BARRY
Barry has prepared the following draft financial statements for your review
Statement of profit or loss for year to 31st August 2015
Rs. in ‘000
Sales revenue 30,000
Raw materials consumed (9,500)
Manufacturing overheads (5,000)
Increase in inventories of work in progress and finished goods 1,400
Staff costs (4,700)
Distribution costs (900)
Depreciation (4,250)
Interest expense (350)
6,700

Statement of financial position as at 31st August 2015


Rs. in ‘000 Rs. in ‘000
Assets
Non-current
Freehold land and buildings 20,000
Plant and machinery 14,000
Fixtures and fittings 5,600
39,600
Current assets
Prepayments 200
Trade receivables 7,400
Cash at bank 700
Inventories 4,600
12,900
Total assets 52,500
Equity and liabilities
Equity shares of Rs. 1 each 21,000
Accumulated profit 14,000
Share premium 2,000
Total equity 37,000
Revaluation surplus 5,000
Current liabilities 5,300
Non-current liabilities
8% Debentures 2019 5,200
Total equity and liabilities 52,500

Additional information
1 Income tax of Rs. 2.1 million has yet to be provided for on profits for the current year. An unpaid
under-provision for the previous year’s liability of Rs. 400,000 has been identified on 5th
September 2015 and has not been reflected in the draft accounts.

© Emile Woolf International 3 The Institute of Chartered Accountants of Pakistan


Financial accounting and reporting II

2 There have been no additions to, or disposals of, non-current assets in the year but the assets
under construction have been completed in the year at an additional cost of Rs. 50,000. These
related to plant and machinery.
The cost and accumulated depreciation of non-current assets as at 1st September 2014 were
as follows:
Cost Depreciation
Rs. in ‘000 Rs. in ‘000
Freehold land and buildings 19,000 3,000
(land element Rs. 10 million)
Plant and machinery 20,100 4,000
Fixtures and fittings 10,000 3,700
Assets under construction 400 -

3 There was a revaluation of land and buildings during the year, creating the revaluation surplus
of Rs. 5 million (land element Rs. 1 million). The effect on depreciation has been to increase
the buildings charge by Rs. 300,000. Barry adopts a policy of transferring the revaluation
surplus included in equity to retained earnings as it is realised.
4 Staff costs comprise 70% factory staff, 20% general office staff and 10% goods delivery staff
5 An analysis of depreciation charge shows the following:
Rs. in ‘000
Buildings (50% production, 50% administration) 1,000
Plant and machinery 2,550
Fixtures and fittings (30% production, 70% administration) 700

Required
Prepare the following information in a form suitable for publication for Barry’s financial statements for
the year ended 31st August 2015.
 Statement of profit or loss
 Statement of financial position
 Reconciliation of opening and closing property, plant and equipment

2.4 OSCAR INC


The following trial balance has been extracted from the books of accounts of Oscar Inc as at 31
March 2015.

Rs. in ‘000
Dr Cr

Administrative expenses 210

Share capital 600

Receivables 470

Bank overdraft 80

Income tax (overprovision in 2014) 25

Provision 180

Distribution costs 420

© Emile Woolf International 4 The Institute of Chartered Accountants of Pakistan


Questions

Rs. in ‘000

Dr Cr

Non-current investments 560

Investment income 75

Plant and machinery


At cost 750

Accumulated depreciation (at 31 March 2015) 220

Retained earnings (at 1 April 2014) 180


Purchases 960

Inventory (at 1 April 2014) 140

Trade payables 260

Sales revenue 2,010

Interim dividend paid 120

3,630 3,630

Additional information
(1) Inventory at 31 March 2015 was valued at Rs. 150,000.
(2) The income tax charge based on the profits on ordinary activities is estimated to be Rs.
74,000.
(3) The provision is to be increased by Rs. 16,000.
(4) There were no purchases or disposals of fixed assets during the year.
Required
Prepare the company’s statement of profit or loss for the year to 31 March 2015 and a statement of
financial position as at that date in accordance with IAS 1.

2.5 CLIFTON PHARMA LIMITED


The following trial balance relates to Clifton Pharma Limited, a public listed company, at 30
September 2015.

Rs. in ‘000

Dr Cr

Cost of sales 134,000

Operating expenses 35,000

Loan interest paid (see note (1)) 1,500

Rental of vehicles (see note (2)) 7000

Revenue 338,300

Investment income 2,000

Leasehold property at cost (see note (4)) 250,000

© Emile Woolf International 5 The Institute of Chartered Accountants of Pakistan


Financial accounting and reporting II

Rs. in ‘000

Dr Cr

Plant and equipment at cost 197,000

Accumulated depreciation at 1 October 2014:

- leasehold property 40,000

- plant and equipment 47,000

Investments 94,000

Share capital 280,000

Share premium 20,000

Retained earnings at 1 October 2014 19,300

Loan notes (see note (1)) 50,000

Deferred tax balance at 1 October 2014 (see note (5)) 20,000

Inventory at 30 September 2015 23,700

Trade receivables 76,400

Trade payables 14,100

Bank 12,100

830,700 830,700
The following notes are relevant
(1) The effective interest rate on the loan notes is 6% per year.
(2) A recent review by the finance department of lease contract has reached the conclusion that
Rs. 7 million of the rental paid of vehicles relates to a lease rather than rental arrangement.
The lease was entered into on 1 October 2014 which was when the Rs. 7 million was paid: the
lease agreement is for a four-year period in total, and there will be three more annual payments
in advance of Rs. 7 million, payable on 1 October in each year. The vehicles in the lease
agreement had a fair value of Rs. 24 million at 1 October 2014 and they should be depreciated
using the straight line method to a nil residual value. The interest rate implicit in the lease is
10% per year.
(3) Other plant and equipment is depreciated at 20% per year by the reducing balance method.
All depreciation of property, plant and equipment should be charged to cost of sales.
(4) The leasehold property has a 25-year life and is amortised at a straight-line rate. On 30
September 2015 the leasehold property was re-valued to Rs. 220 million and the directors wish
to incorporate this re-valuation in the financial statements.
(5) The provision for income tax for the year ended 30 September 2015 has been estimated at Rs.
18 million. At 30 September 2015 there are taxable temporary differences of Rs. 92 million. The
rate of income tax on profits is 25%.
Required
(a) Prepare an statement of profit or loss for Clifton Pharma Limited for the year to 30 September
2015
(b) Prepare a statement of financial position (balance sheet) for Clifton Pharma Limited as at 30
September 2015

© Emile Woolf International 6 The Institute of Chartered Accountants of Pakistan


Questions

2.6 SARHAD SUGAR LIMITED


The following trial balance relates to Sarhad Sugar Limited at 30 September 2015:

Rs. in ‘000
Dr Cr
Leasehold property – at valuation 1 October 2014 (note (i)) 50,000
Plant and equipment – at cost (note (i)) 76,600
Plant and equipment – accumulated depreciation at 1 October 2014 24,600
Capitalised development expenditure – at 1 October 2014 (note (ii)) 20,000
Development expenditure – accumulated amortisation at 1 October
2014 6,000
Closing inventory at 30 September 2015 20,000
Trade receivables 43,100
Bank 1,300
Trade payables and provisions (note (iii)) 23,800
Revenue (note (i)) 300,000
Cost of sales 204,000
Distribution costs 14,500
Administrative expenses (note (iii)) 22,200
Interest on bank borrowings 1,000
Equity dividend paid 6,000
Research and development costs (note (ii)) 8,600
Share capital 70,000
Retained earnings at 1 October 2014 24,500
Deferred tax (note (v)) 5,800
Revaluation surplus (Leasehold property) 10,000
466,000 466,000

The following notes are relevant:


(i) Non-current assets – tangible:
The leasehold property had a remaining life of 20 years at 1 October 2014. The company’s
policy is to revalue its property at each year end and at 30 September 2015 it was valued at
Rs. 43 million.
On 1 October 2014 an item of plant was disposed of for Rs. 2·5 million cash. The proceeds
have been treated as sales revenue by Sarhad Sugar Limited. The plant is still included in the
above trial balance figures at its cost of Rs. 8 million and accumulated depreciation of Rs. 4
million (to the date of disposal).
All plant is depreciated at 20% per annum using the reducing balance method.
Depreciation and amortisation of all non-current assets is charged to cost of sales.
(ii) Non-current assets – intangible:
In addition to the capitalised development expenditure (of Rs. 20 million), further research and
development costs were incurred on a new project which commenced on 1 October 2014.

© Emile Woolf International 7 The Institute of Chartered Accountants of Pakistan


Financial accounting and reporting II

The research stage of the new project lasted until 31 December 2014 and incurred Rs. 1·4
million of costs. From that date the project incurred development costs of Rs. 800,000 per
month. On 1 April 2015 the directors became confident that the project would be successful
and yield a profit well in excess of its costs. The project is still in development at 30
September 2015.
Capitalised development expenditure is amortised at 20% per annum using the straight-line
method. All expensed research and development is charged to cost of sales.
(iii) Sarhad Sugar Limited is being sued by a customer for Rs. 2 million for breach of contract over
a cancelled order. Sarhad Sugar Limited has obtained legal opinion that there is a 20%
chance that Sarhad Sugar Limited will lose the case. Accordingly Sarhad Sugar Limited has
provided Rs. 400,000 (Rs. 2 million x 20%) included in administrative expenses in respect of
the claim. The unrecoverable legal costs of defending the action are estimated at Rs.
100,000. These have not been provided for as the legal action will not go to court until next
year.
(iv) The directors have estimated the provision for income tax for the year ended 30 September
2015 at Rs. 11·4 million. The required deferred tax provision at 30 September 2015 is Rs. 6
million.
Required
(a) Prepare the statement of profit or loss for the year ended 30 September 2015
(b) Prepare the statement of financial position as at 30 September 2015.
Note: notes to the financial statements are not required.

2.7 BSZ LIMITED


The post-closing trial balance of BSZ Limited, a listed company, as at June 30, 2017 is given
below:
Debit Credit
Rs. in million
Cash at banks – current accounts 7
Cash at banks – in saving accounts 22
Stocks in trade – closing 90
Accounts receivable 60
Provision for bad debts 3
Advances to suppliers 16
Advances to staff 6
Short term deposits 11
Prepayments 4
Sales tax receivable 12
Freehold land – at revalued amount 375
Furniture and fixtures - cost 27
Accumulated depreciation – Furniture and fixtures 8
Machines - cost 85
Accumulated depreciation – Machines 27
Building on freehold land – cost 150
Accumulated depreciation – Building 26
Computer software – cost 10

© Emile Woolf International 8 The Institute of Chartered Accountants of Pakistan


Questions

Debit Credit
Rs. in million
Accumulated amortization – Computer software 2
Deferred taxation 40
Short term loan 85
Accounts payable 75
Accrued liabilities 7
Provision for taxation 17
Issued, subscribed and paid up capital (Rs. 10 each) 400
Surplus on revaluation of fixed assets 120
Accumulated profits 65
875 875
Additional Information
(i) The first revaluation of freehold land was carried out in 2013 and resulted in a surplus of
Rs. 120 million. The valuation was carried out under market value basis by an independent
valuer, Mr. Dee, Chartered Civil Engineer of M/s SSS Consultants (Pvt.) Ltd., Islamabad.
(ii) The details relating to additions, disposal and depreciation/amortization of fixed assets,
during the year 2017 are given below:
 The company uses the straight line method for charging depreciation and amortization.
The building is depreciated at a rate of 5% whereas 10% is charged on machines,
furniture and fixtures and computer software.
 Construction on third floor of the building commenced on March 1, 2017 and is
expected to be completed on September 30, 2017. The cost incurred during the year
i.e. Rs. 20 million was capitalised on June 30, 2017.
 Furniture and fixtures worth Rs. 8 million were purchased on April 1, 2017.
 A machine was sold on February 28, 2017 to NJ Enterprise at a price of Rs. 13 million.
At the time of disposal, the cost and written down value of the machine was Rs. 15
million and Rs. 10 million respectively.
(iii) 50% of the accounts receivable were secured and considered good. 10% of the
unsecured accounts receivable were considered doubtful. Bad debts expenses for the year
amounted to Rs. 1.0 million. An amount of Rs. 1.4 million was written off during the year.
(iv) All advances given to suppliers are considered good and include an amount of Rs. 4.0
million paid for goods which will be supplied on December 31, 2018.
(v) Cash at banks in saving accounts carry interest / mark-up ranging from 3% to 7% per
annum.
(vi) The authorised share capital of the company is Rs. 500 million.
Required
Prepare the statement of financial position as at June 30, 2017 along with the relevant notes
showing all possible disclosures as required under the International Accounting Standards and
the Companies Act, 2017.
(Comparative figures and the note on accounting policies are not required.)

© Emile Woolf International 9 The Institute of Chartered Accountants of Pakistan


Financial accounting and reporting II

2.8 YASIR INDUSTRIES LIMITED


The following trial balance related to Yasir Industries Limited (YIL) for the year ended June 30, 2017:

Dr Cr
Rs. in million
Ordinary share capital (Rs. 10 each) - 120.00
Retained earnings - 10.20
Sales - 472.40
Purchases 175.70 -
Production labour 61.00
Manufacturing overheads 39.00
Inventories (July 1, 2016) 38.90
Administrative expenses 40.00 -
Distribution expenses 19.80 -
Financial charges 0.30 -
Cash and bank - 13.25
Trade creditors - 30.40
Accrued expenses - 16.20
10% redeemable preference shares - 40.00
Debentures - 80.00
Deferred tax (July 1, 2016) - 6.00
Suspense account 30.00 -
Leasehold property - at cost 230.00 -
Machines – at cost 168.60 -
Software – at cost 20.00 -
Acc. depreciation – Leasehold property (June 30, 2017) - 40.25
Acc. depreciation – Machines (June 30, 2017) - 48.60
Acc. amortization – Software (June 30, 2017) - 12.00
Trade receivables 66.00 -
889.30 889.30
Additional Information
(i) Sales include an amount of Rs. 27 million, made to a customer under sale or return
agreement. The sale has been made at cost plus 20% and the expiry date for the return of
these goods is July 31, 2017.
(ii) The value of inventories at June 30, 2017 was Rs. 42 million.
(iii) A fraud of Rs. 30 million was discovered in October 2016. A senior employee of the company
who left in June 2016, had embezzled the funds from YIL’s bank account. The chances of
recovery are remote. The amount is presently appearing in the suspense account.
(iv) On January 1, 2017 YIL issued debenture certificates which are repayable in 2022. Interest is
paid on these at 12% per annum.
(v) Financial charges comprise bank charges and bank commission.
(vi) The provision for current taxation for the year ended June 30, 2017 after making all the above
adjustments is estimated at Rs. 16.5 million.

© Emile Woolf International 10 The Institute of Chartered Accountants of Pakistan


Questions

(vii) The carrying value of YIL’s net assets as on June 30, 2017 exceeds their tax base by Rs. 30
million. The income tax rate applicable to the company is 30%.
(viii) On July 1, 2016, the leasehold property having a useful life of 40 years was revalued at Rs.
238 million. No adjustment in this regard has been made in the books.
(ix) Depreciation of leasehold property is charged using the straight line method. 50% of
depreciation is allocated to manufacturing, 30% to administration and 20% to selling and
distribution.
Required
In accordance with the requirements of the Companies Act, 2017 and International Accounting
Standards, prepare the:
(a) statement of financial position as of June 30, 2017.
(b) statement of profit or loss for the year ended June 30, 2017.
(Comparative figures and notes to the financial statements are not required.)

2.9 SHAHEEN LIMITED


Following is the trial balance of Shaheen Limited (SL) as at June 30, 2017:
Rs. in ‘000
Dr Cr
Sales revenue 200,000
Manufacturing costs 100,000
Selling and distribution costs 35,000
Administrative costs 30,000
Opening inventories 23,000
Interest on borrowings 5,000
Provision for income tax 2,000
Advance income tax paid 6,000
Property, plant and equipment 86,000
Accumulated depreciation on property, plant and equipment 12,000
Export licence 6,000
Trade receivables 37,800
Cash and bank balances 4,725
Other receivable and prepayments 14,000
Trade payables 12,000
Provisions for litigation 5,000
Long term borrowings 31,525
Deferred tax 5,000
Share capital (Rs. 10 each and fully paid) 60,000
Retained earnings 20,000
347,525 347,525
Additional information
(i) Sales last year (year ended 30 June 2016) included goods invoiced at Rs 10 million which
were sent to a customer on June 25, 2016 under a sale or return agreement, at cost plus
20%. The goods were returned on August 25, 2016. No correction has been made for the
return.
(ii) The export licence has been obtained for exporting a new product and is effective for five
years up to December 31, 2021. However, the exports commenced from July 1, 2017.

© Emile Woolf International 11 The Institute of Chartered Accountants of Pakistan


Financial accounting and reporting II

(iii) Closing inventories are valued at Rs. 30 million.


(iv) Details of property, plant and equipment are as follows:

Plant and
Land Buildings equipment
Rs in ‘000
Cost as at June 30, 2016 20,000 36,000 30,000
Fully depreciated amounts included in cost 3,000
Estimated useful life at the date of purchase 20 years 10 years

The company uses straight line method for charging depreciation. Depreciation is allocated
to manufacturing, distribution and administrative costs at 75%, 15% and 10% respectively.
(v) Rs. 6 million of the long term borrowings is of current maturity (i.e. will be repaid within 12
months).
(vi) During the year Rs. 5 million was paid in full and final settlement of income tax liability
against which a provision of Rs. 7.0 million had been made in the previous year. Current
year’s taxable income exceeds accounting income by Rs. 5 million of which 0.8 million are
permanent differences. Applicable tax rate for the company is 35%.
(vii) On July 30, 2017 the board of directors proposed a final dividend at 15% for the year ended
June 30, 2017 (2016: at 20%)
Required
In accordance with the requirements of the Companies Act, 2017 and International Financial
Reporting Standards, prepare:
(a) The statement of financial position as of June 30, 2017
(b) The statement of profit or loss for the year ended June 30, 2017
(c) The statement of changes in equity for the year ended June 30, 2017.
(Comparative figures and notes to the financial statements are not required)

2.10 MOONLIGHT PAKISTAN LIMITED


Following is the summarised trial balance of Moonlight Pakistan Limited (MPL), a listed company, for
the year ended December 31, 2017:

Rs. in million

Debit Credit

Land and buildings - at cost 2,600 -

Plants – at cost 2,104 -

Trade receivables 702 -

Stock in trade at December 31, 2017 758 -

Cash and bank 354 -

Cost of sales 1,784 -

Selling expenses 220 -

Administrative expenses 250 -

Financial charges 210 -

© Emile Woolf International 12 The Institute of Chartered Accountants of Pakistan


Questions

Rs. in million

Debit Credit

Accumulated depreciation as on January 1, 2017 – Buildings - 400

Accumulated depreciation as on January 1, 2017 – Plants - 670

Ordinary shares of Rs. 10 each fully paid - 1,200

Retained earnings as at January 1, 2017 - 510

12% Long term loan - 1,600

Provision for gratuity - 8

Deferred tax on January 1, 2017 - 22

Trade payables - 544

Right subscription received - 420

Revenue - 3,608

8,982 8,982

Additional Information
(i) The land and buildings were acquired on January 1, 2013. The cost of land was Rs. 600
million. On January 1, 2017 a professional valuation firm valued the buildings at Rs. 1,840
million with no change in the value of land. The estimated life at acquisition was 20 years
and the remaining life has not changed as a result of the valuation. 60% of depreciation on
buildings is allocated to manufacturing, 25% to selling and 15% to administration.
(ii) Plant is depreciated at 20% per annum using the reducing balance method.
(iii) On March 31, 2017 MPL made a bonus issue of one share for every six held. The issue
has not been recorded in the books of account.
(iv) Right shares were issued on September 1, 2017 at Rs. 12 per share.
(v) The interest on long term loan is payable on the first day of July and January. No accrual has
been made for the interest payable on January 1, 2015.
(vi) MPL operates an unfunded gratuity scheme for all its eligible employees. The provision
required as on December 31, 2017 is estimated at Rs. 23 million. Rs. 3 million were paid
during the year and debited to the provision for gratuity account. Cost of gratuity is allocated
to production, selling and administration expenses in the ratio of 60%: 20% : 20%.
(vii) The tax charge for the current year after making all related adjustments is estimated at Rs. 37
million. The timing differences related to taxation are estimated to increase by Rs. 80 million,
over the last year. The applicable income tax rate is 35%.
Required

In accordance with the requirements of Companies Act, 2017 and International Financial
Reporting Standards, prepare the following:
(a) Statement of Financial Position as of December 31, 2017.
(b) Statement of profit or loss for the year ended December 31, 2017.
(Comparative figures and notes to the financial statements are not required)

© Emile Woolf International 13 The Institute of Chartered Accountants of Pakistan


Financial accounting and reporting II

2.11 FIGS PAKISTAN LIMITED


Figs Pakistan Limited is a listed company engaged in the business of manufacturing and marketing
of personal care and food products. Following is an extract from its trial balance for the year ended
31 December 2017:

Debit Credit
Rs. in million
Sales - Manufactured goods 56,528
Sales - Imported goods 1,078
Scrap sales 16
Dividend income 12
Return on savings account 2
Sales tax - Imported goods 53
Sales tax - Manufactured goods 10,201
Sales discount 2,594
Raw material stock as on 1 January 2017 1,751
Work in process as on 1 January 2017 73
Finished goods (manufactured) as on 1 January 2017 1,210
Finished goods (imported) as on 1 January 2017 44
Purchases - Raw material 22,603
Purchases - Imported goods 658
Stores and spares consumed 180
Salaries, wages and benefits 2,367
Utilities 734
Depreciation and amortization 1,287
Stationery and office expenses 230
Repairs and maintenance 315
Advertisement and sales promotion 4,040
Outward freight and handling 1,279
Legal and professional charges 71
Auditor's remuneration 13
Donations 34
Workers Profit Participation Fund 257
Worker Welfare Fund 98
Loss on disposal of property, plant and equipment 10
Financial charges on short term borrowings 133
Exchange loss 22
Financial charges on lease 11

© Emile Woolf International 14 The Institute of Chartered Accountants of Pakistan


Questions

Additional information
(i) The position of inventories as at 31 December 2017 was as follows:

Rs. m
Raw material 2,125
Work in process 125
Finished goods (manufactured) 1,153
Finished goods (imported) 66

(ii) The basis of allocation of various expenses among cost of sales, distribution costs and
administrative expenses are as follows:
Cost of Distribution Administrative
sales costs expenses
% % %
Salaries, wages and benefits 55 30 15
Depreciation and amortization 70 20 10
Stationery and office expenses 25 40 35
Repairs and maintenance / Utilities 85 5 10
(iii) Salaries, wages and benefits include contributions to provident fund (defined contribution
plan) and gratuity fund (defined benefit plan) amounting to Rs. 54 million and Rs. 44 million
respectively.
(iv) Auditor’s remuneration includes taxation services and out-of-pocket expenses amounting to
Rs. 4 million and Rs. 1 million respectively.
(v) Donations include Rs. 5 million given to Dates Cancer Foundation (DCF). One of the
company’s directors, Mr. Peanut is a trustee of DCF.
(vi) The tax charge for the current year after making all related adjustments is estimated at Rs.
1,440 million. Taxable temporary differences of Rs. 3,120 originated in the year million, over
the last year. The applicable income tax rate is 35%.
(vii) 274 million ordinary shares were outstanding as on 31 December 2017.
(viii) There is no other comprehensive income for the year.
Required
Prepare the statement of profit or loss for the year ended 31 December 2017 along with the relevant
notes showing required disclosures as per the Companies Act, 2017 and International Financial
Reporting Standards. Comparatives are not required.

2.12 WAH AGRIPROD LTD


The following trial balance was extracted from Wah Agriprod Ltd’s books at 31 December 2016:
Rs.’000 Rs.’000
Revenue 855,000
Purchases 503,600
Distribution costs 31,950
Administrative expenses 104,400
Finance costs 1,020
Investment income 18,250
Leased property at cost 125,000
Plant and equipment at cost 98,800

© Emile Woolf International 15 The Institute of Chartered Accountants of Pakistan


Financial accounting and reporting II

Rs.’000 Rs.’000
Accumulated amortisation/depreciation at 1/1/2016:
Leased property 35,000
Plant and equipment 28,800
Financial instruments 40,500
Inventory at 1/1/2016 85,075
Trade receivables 72,400
Trade payables 62,550
Bank 5,175
Share premium 8,400
Equity shares (Rs. 1 per share) 115,000
Retained earnings at 1/1/2016 55,600
Investment in subsidiary 131,030
6% Convertible Loan Notes 10,000
1,193,775 1,193,775
The following notes are relevant:
(i) Revenue includes goods sold and dispatched from 15 December 2016, on a 30-day right of
return basis. Their selling price was Rs. 3.6 million and they were sold at a gross profit margin
of 20%. Wah Agriprod Ltd is uncertain as to whether any of these goods will be returned
within the 30-day period.
(ii) The directors decided to revalue the leased property in line with recent increases in market
values. On 1 September 2016, an independent surveyor valued the leased property at Rs.
99million, which the directors have accepted. The leased property was amortised over an
original life of 25 years which has not changed. Wah Agriprod Ltd does not make a transfer to
retained earnings in respect of excess amortisation.
(iii) The plant and equipment is depreciated at 15% per annum using the reducing balance
method and all depreciation and amortisation are charged to cost of sales. No depreciation or
amortisation has yet been charged on any non-current asset for the year ended 31 December
2016.
(iv) The financial instruments are investments in equities of public companies and had a fair value
of N39.7million on 31 December 2016. There were no purchases or disposals of any of these
investments during the year. Wah Agriprod Ltd has not made the election in accordance with
IFRS 9 on Financial Instruments. The company adopts this standard when accounting for its
financial assets.
(v) On 20 November 2016, Wah Agriprod Ltd’s share price stood at Rs. 2.20 per share. On this
date, a dividend that was calculated to give a dividend yield of 5% was paid by the company.
The dividend paid was included as part of administrative expenses figure shown in the trial
balance.
(vi) The inventory on Wah Agriprod Ltd’s premises at 31 December 2016, after stock taking was
valued at cost of Rs. 106million and a provision for income tax for the year then ended of Rs.
86.75million is required.
(vii) During the year, the company issued ten million shares at a premium of 20%. The conversion
rate for the loan note is Rs. 100 loan notes for three ordinary shares. The current market
price per share is Rs. 2.54.
Required
(a) Prepare the statement of profit or loss and other comprehensive income for the year ended 31
December 2016.
(b) Prepare the statement of changes in equity for the same period.

© Emile Woolf International 16 The Institute of Chartered Accountants of Pakistan


Questions

CHAPTER 3 – CONSOLIDATED ACCOUNTS: STATEMENTS OF FINANCIAL


POSITION – BASIC APPROACH
3.1 P AND S (I)
Statement of financial positions at 31 December 2018
P S
Rs.000 Rs.000
Non-current assets 50 40
Investment in S at cost 70 -
Current assets 30 40
Total assets 150 80

Ordinary share capital (Rs.1 shares) 100 50


Retained earnings 30 20
Current liabilities 20 10
Total equity and liabilities 150 80

P acquired all the shares in S on 30 June 2018 when the retained earnings of S amounted to
Rs.15,000.
Required:
Prepare consolidated statement of financial position as at December 31, 2018.

3.2 P AND S (II)


Statement of financial positions at 31 December 2018
P S
Rs.000 Rs.000
Non-current assets 50 40
Investment in S at cost 70 -
Current assets 30 40
Total assets 150 80

Ordinary share capital (Rs.1 shares) 100 50


Retained earnings 30 20
Current liabilities 20 10
Total equity and liabilities 150 80
P acquired 40,000 Rs.1 shares in S on 30 June 2018 for Rs.70,000, when the retained earnings of
S amounted to Rs.15,000. The group has a policy of measuring non-controlling interest at
proportionate share of net assets at the date of acquisition. 20% of goodwill has impaired to date.
Required:
Prepare consolidated statement of financial position as at December 31, 2018.

© Emile Woolf International 17 The Institute of Chartered Accountants of Pakistan


Financial accounting and reporting II

3.3 BL AND FL
The summarized draft statement of financial positions of the companies in a group at 31
December 2018 were
BL FL
Rs. Rs.
Property, plant and equipment 86,000 24,500
Investment in FL (at cost) 27,000 -
Current assets 20,000 10,000
133,000 34,500

Share capital (Rs.1) 100,000 20,000


Retained earnings 22,000 6,500
Current liabilities 11,000 8,000
133,000 34,500
Additional information is as follows:
a) BL acquired 16,000 ordinary shares in FL on 1 January 2018, when FL had accumulated profits
of Rs.6,000.
b) The subsidiary has not incorporated the fair values in its separate books and fair value
adjustments identified by the parent company at the date of acquisition are as follows: -

Particulars Carrying value Fair value at Exist on


at acquisition acquisition Reporting
date date date
Land 10,000 12,000 Yes
Inventory 6,000 4,500 No

The group has a policy of measuring non-controlling interest at proportionate share of net assets at
the date of acquisition. 20% of goodwill has impaired to date.
Required:
Prepare the consolidated statement of financial position at 31 December 2018.

3.4 ML AND ZL
Statements of financial position at 31 December 2018

ML ZL

Rs. Rs.
Property, plant and equipment 41,000 16,000
Investment in ZL (at cost) 19,000 -
Current assets 20,000 28,000
ZL current account 10,000 -

90,000 44,000

© Emile Woolf International 18 The Institute of Chartered Accountants of Pakistan


Questions

Share capital (Rs.1) 50,000 10,000


Retained earnings 30,000 20,000
Current liabilities 10,000 5,000
ML current account - 9,000

90,000 44,000

Additional information is as follows:


1. ML bought 7,500 shares in ZL on 1 January 2018 when the balance on the retained earnings
of ZL was Rs.12,000.
2. The current account difference has arisen as a cheque of Rs. 500 sent by ZL to P on 30
December 2018 was not received by ML until 3 January 2019, Rs. 300 purchases by ZL from
ML wrongly credited to some other creditor account and Rs. 200 charged by ML for certain
expenses paid on behalf of ZL.
3. No stock related to intercompany purchases exists at the reporting date.
4. The ML Group has the policy of measuring non-controlling interest at fair value (FV) and FV
of NCI was Rs. 6,000 at the date of acquisition.
5. Goodwill of Rs. 1,000 has been impaired to date
Required:
Prepare the consolidated statement of financial position at 31 December 2018.

3.5 FLAMSTEED LTD AND HALLEY LTD


The draft Statement of Financial Position of Flamsteed Ltd and Halley Ltd on 30 June 2016 were as
follows:
Statement of financial position as at 30 June 2016

Flamsteed Ltd Halley Ltd


Rs.’000 Rs.’000

Assets:

Non-current Assets:

Property, plant and equipment 100,000 80,000

40,000 ordinary shares in Halley at cost 60,000 -

160,000 80,000

Current assets:

Inventory 6,000 16,000

Owed by Flamsteed Ltd - 20,000

Receivables 32,000 14,000

Cash 4,000 -

42,000 50,000

Total assets 202,000 130,000

© Emile Woolf International 19 The Institute of Chartered Accountants of Pakistan


Financial accounting and reporting II

Flamsteed Ltd Halley Ltd


Rs.’000 Rs.’000

Equity and liabilities:

Equity (ordinary shares @ Rs. 1) 90,000 50,000

Revaluation surplus 24,000 10,000

Retained earnings 52,000 56,000

166,000 116,000

Current Liabilities:

Owed to Halley Ltd 16,000 -

Trade payables 20,000 14,000

36,000 14,000

Total equity and liabilities 202,000 130,000

Additional information:
(i) Flamsteed Ltd acquired its investment in Halley Ltd on 1 July 2015, when the retained
earnings of Halley Ltd stood at Rs. 12,000,000.
(ii) The agreed consideration was Rs. 60,000,000 at the date of acquisition and a further Rs.
20,000,000 on 1 July 2017, Flamsteed Ltd’s cost of capital is 7%.
(iii) Halley Ltd has an internally developed brand name – TOLX – which was valued at Rs.
10,000,000 at the date of acquisition.
(iv) There have been no changes in the capital or revaluation surplus of Halley Ltd since the date
its shares were purchased.
(v) At 30 June 2016, Halley had invoiced Flamsteed Ltd for goods to the value of Rs. 4,000,000
and Flamsteed Ltd had sent payment in full but this had not been received by Halley Ltd.
(vi) There is no impairment of goodwill.
(vii) It is the group’s policy to value non-controlling interest at full fair value.
(viii) At the acquisition date, the non-controlling interest was valued at Rs. 18,000,000.
Required
(a) Define Impairment loss in accordance with IAS 36 on Impairment of Assets.
(b) Explain any THREE sources of external information which an entity may consider in
assessing whether there is any indication that an asset may be impaired.
(c) Prepare an extract of consolidated Statement of Financial position of Flamsteed Ltd for
the year ended 30 June 2016, showing the assets side only.

© Emile Woolf International 20 The Institute of Chartered Accountants of Pakistan


Questions

CHAPTER 4 – CONSOLIDATED ACCOUNTS: STATEMENTS OF FINANCIAL


POSITION – COMPLICATIONS
4.1 HAIL
The following are the draft statements of financial position of Hail and its subsidiary Snow as at
31 December 2015.
Hail Snow
Rs. 000 Rs. 000
Assets
Non-current assets
Property, plant and equipment 161,000 85,000
Investments 68,000
Current assets
Cash 7,700 25,200
Trade receivables 92,500 45,800
Snow current account 15,000 -
Inventory 56,200 36,200
———– ——–—
400,400 192,200
———– ——–—
Equity and liabilities
Shareholders’ equity
Share capital 100,000 50,000
Retained earnings 185,400 41,200
Share premium - 5,000
Capital reserve - 20,000
———– ——–—
285,400 116,200
Current liabilities 115,000 68,000
Hail current account - 8,000
———– ——–—
400,400 192,200
———– ——–—
Notes
(1) Snow has 50,000 shares in issues. Hail acquired 45,000 of these on 1 January 2012 for a
cost of Rs. 65,000,000 when the balances on Snow’s reserves were:
Rs. 000
Share premium account 5,000
Capital reserve –
Retained earnings 10,000

(2) Hail declared a dividend of Rs. 3,000,000 before the year end and Snow declared one of Rs.
2,000,000. These transactions have not been accounted for.
(3) The current account difference is due to cash in transit.

Required
Prepare the consolidated statement of financial position as at 31 December 2015 of Hail.

© Emile Woolf International 21 The Institute of Chartered Accountants of Pakistan


Financial accounting and reporting II

4.2 HAIRY
The summarised statements of financial position of Hairy and Spider as at 31 December 2015 were
as follows.
Hairy Spider
Rs. 000 Rs. 000
Assets
Non-current assets
Property, plant and equipment 120,000 60,000
Investments 55,000 –
Current assets
Cash 11,000 4,000
Investments – 3,000
Trade receivables 72,600 19,100
Current account – Hairy – 3,200
Inventory 17,000 11,000
———– ———–
275,600 100,300
———– ———–
Equity and liabilities
Share capital 100,000 60,000
Share premium 20,000 –
Capital reserve 23,000 16,000
Retained earnings 91,900 7,300
Trade payables 38,000 17,000
Current account – Spider 2,700 –
———– ———–
275,600 100,300
———– ———–
The following information is relevant.
(1) On 31 December 2012, Hairy acquired 48,000 shares in Spider for Rs. 55,000,000 cash.
Spider has 60,000 shares in total.
(2) The inventory of Hairy includes Rs. 4,000,000 goods from Spider invoiced to Hairy at cost
plus 25%.
(3) The difference on the current account balances is due to cash in transit.
(4) The balance on Spider’s retained earnings was Rs. 2,300,000 at the date of acquisition. There
has been no movement in the balance on Spider’s capital reserve since the date of
acquisition.

Required
Prepare the consolidated statement of financial position of Hairy and its subsidiary Spider as at 31
December 2015.

© Emile Woolf International 22 The Institute of Chartered Accountants of Pakistan


Questions

4.3 HARD
On 31 December 2011, Hard acquired 60% of the ordinary share capital of Soft for Rs. 110 million.
At that date Soft had a retained earnings balance of Rs. 50 million and a share premium account
balance of Rs. 10 million.
The following statements of financial position have been prepared as at 31 December 2015.
Hard Soft
Rs. 000 Rs. 000
Assets
Non-current assets
Property, plant and equipment 225,000 175,000
Investments in Soft 110,000

Current assets 271,000 157,000


———– ———–
606,000 332,000
———– ———–
Equity and liabilities
Capital and reserves
Share capital 100,000 100,000
Share premium 15,000 10,000
Retained earnings 260,000 80,000
———– ———–
375,000 190,000
Current liabilities 231,000 142,000
———– ———–
606,000 332,000
———– ———–
During the year to 31 December 2015 Hard sold a tangible asset to Soft for Rs. 50 million. The asset
was originally purchased in the year to 31 December 2012 at a cost of Rs. 100 million and had a
useful economic life of five years.
Soft’s depreciation policy is 25% per annum based on cost. Both companies charge a full year’s
depreciation in the year of acquisition and none in the year of disposal.

Required
Prepare the consolidated statement of financial position of Hard and its subsidiary as at 31
December 2015.

4.4 HALE
On 1 July 2012 Hale acquired 128,000 of Sowen’s 160,000 shares. The following statements of
financial position have been prepared as at 31 December 2015.
Hale Sowen
Rs. 000 Rs. 000
Property, plant and equipment 152,000 129,600
Investment in Sowen 203,000 –
Inventory at cost 112,000 74,400
Receivables 104,000 84,000
Bank balance 41,000 8,000
———– ———–
612,000 296,000
═════ ═════

© Emile Woolf International 23 The Institute of Chartered Accountants of Pakistan


Financial accounting and reporting II

Hale Sowen
Rs. 000 Rs. 000
Share capital 100,000 160,000
Retained earnings 460,000 112,000
Payables 52,000 24,000
———– ———–
612,000 296,000
═════ ═════
The following information is available.
(1) At 1 July 2012 Sowen had a debit balance of Rs. 11 million on retained earnings.
(2) Property, plant and equipment of Sowen included land at a cost of Rs. 72 million. This land
had a fair value of Rs. 100,000 at the date of acquisition.
(3) The inventory of Sowen includes goods purchased from Hale for Rs. 16 million. Hale invoiced
those goods at cost plus 25%.

Required
Prepare the consolidated statement of financial position of Hale as at 31 December 2015.

4.5 HELLO
On 1 January 2012, Hello acquired 60% of the ordinary share capital of Solong for Rs. 110,000. At
that date Solong had a retained earnings balance of Rs. 60,000.
The following statements of financial position have been prepared as at 31 December 2015.
Hello Solong
Rs. Rs.
Assets
Non-current assets
Property, plant and equipment 225,000 175,000
Investments in Solong 110,000

Current assets 271,000 157,000


———– ———–
606,000 332,000
———– ———–

Equity and liabilities


Capital and reserves
Share capital 100,000 100,000
Retained earnings 275,000 90,000
———– ———–
375,000 190,000
Current liabilities 231,000 142,000
———– ———–
606,000 332,000
———– ———–

The fair value of Solong’s net assets at the date of acquisition was determined to be Rs. 170,000.
The difference between the book value and the fair value of the new assets at the date of acquisition
was due to an item of plant which had a useful life of 10 years from the date of acquisition.

Required
Prepare the consolidated statement of financial position of Hello and its subsidiary as at 31
December 2015.

© Emile Woolf International 24 The Institute of Chartered Accountants of Pakistan


Questions

4.6 HASAN LIMITED


On 1 April 2014, Hasan Limited acquired 90% of the equity shares in Shakeel Limited. On the same
day Hasan Limited accepted a 10% loan note from Shakeel Limited for Rs. 200,000 which was
repayable at Rs. 40,000 per annum (on 31 March each year) over the next five years. Shakeel
Limited’s retained earnings at the date of acquisition were Rs. 2,200,000.
Statements of financial position as at 31 March 2015
Hasan Shakeel
Limited Limited
Rs. 000 Rs. 000
Non-current assets
Property, plant and equipment 2,120 1,990
Intangible – software – 1,800
Investments – equity in Shakeel Limited 4,110 –
Investments – 10% loan note Shakeel 200 –
Limited
Investments – others 65 210
6,495 4,000
Current assets
Inventories 719 560
Trade receivables 524 328
Shakeel Limited current account 75 –
Cash 20
1,338 888
Total assets 7,833 4,888

Equity and liabilities:


Capital and reserves
Equity shares of Rs. 1 each 2,000 1,500
Share premium 2,000 500
Retained earnings 2,900 1,955
6,900 3,955
Non-current liabilities
10% Loan note from Hasan Limited – 160
Government grant 230 40
230 200
Current liabilities
Trade payables 475 472
Hasan Limited current account – 60
Income taxes payable 228 174
Operating overdraft – 27
703 733
Total equity and liabilities 7,833 4,888

© Emile Woolf International 25 The Institute of Chartered Accountants of Pakistan


Financial accounting and reporting II

The following information is relevant:


(i) Included in Shakeel Limited’s property at the date of acquisition was a leasehold property
recorded at its depreciated historical cost of Rs. 400,000. The leasehold had been sub-let for
its remaining life of only four years at an annual rental of Rs. 80,000 payable in advance on 1
April each year. The directors of Hasan Limited are of the opinion that the fair value of this
leasehold is best reflected by the present value of its future cash flows. An appropriate cost of
capital for the group is 10% per annum.
The present value of a Rs. 1 annuity received at the end of each year where interest rates are
10% can be taken as:
3 year annuity Rs. 2.50
4 year annuity Rs. 3.20
(ii) The software of Shakeel Limited represents the depreciated cost of the development of an
integrated business accounting package. It was completed at a capitalised cost of Rs.
2,400,000 and went on sale on 1 April 2013. Shakeel Limited’s directors are depreciating the
software on a straight-line basis over an eight-year life (i.e. Rs. 300,000 per annum).
However, the directors of Hasan Limited are of the opinion that a five-year life would be more
appropriate as sales of business software rarely exceed this period.
(iii) The inventory of Hasan Limited on 31 March 2015 contains goods at a transfer price of Rs.
25,000 that were supplied by Shakeel Limited who had marked them up with a profit of 25%
on cost. Unrealised profits are adjusted for against the profit of the company that made them.
(iv) On 31 March 2015 Shakeel Limited remitted to Hasan Limited a cash payment of Rs. 55,000.
This was not received by Hasan Limited until early April. It was made up of an annual
repayment of the 10% loan note of Rs. 40,000 (the interest had already been paid) and Rs.
15,000 of the current account balance.
(v) The accounting policy of Hasan Limited for non-controlling interests (NCI) in a subsidiary is to
value NCI at a proportionate share of the net assets.
(v) An impairment test at 31 March 2015 on the consolidated goodwill concluded that it should be
written down by Rs. 120,000. No other assets were impaired.

Required
Prepare the consolidated statement of financial position of Hasan Limited as at 31 March 2015.
4.7 GOLDEN LIMITED
The draft summarized statements of financial position of Golden Limited (GL) and its subsidiary
Silver Limited (SL) as at 31 December 2016 are as follows:
GL SL
---------- Rs. in million ----------
Building 1,600 500
Plant & machinery 1,465 690
Investment in SL 327 -
Current assets 2,068 780
5,460 1,970

Share capital (Rs. 10 each) 980 450


Share premium 730 150
Retained earnings 3,150 210
4,860 810
Liabilities 600 1,160
5,460 1,970
(i) GL acquired 60% of the shares of SL on 1 April 2016 at following consideration:
 Issuance of 20 million ordinary shares at premium of Rs. 2 each;

© Emile Woolf International 26 The Institute of Chartered Accountants of Pakistan


Questions

 Cash amounting to Rs. 87 million, which includes consultancy charges of Rs. 10 million
and legal expenses of Rs. 5 million.
The market value of each share of GL and SL on acquisition date was Rs. 25 and Rs. 11
respectively. At acquisition date, retained earnings of SL were Rs. 100 million.
(ii) The following table sets out those items whose fair value on the acquisition date was different
from their book value. These values have not been incorporated in SL’s books of account.
Book value Fair value
---------Rs. in million---------
Building 250 170
Inventory 112 62
Provision for bad debts (15) (24)
(iii) Upon acquisition of SL, a contract for management services was also signed under which GL
would provide various management services to SL at an annual fee of Rs. 50 million from the
date of acquisition. The payment would be made in two equal instalments payable in arrears on
1 April and 1 October.
(iv) On 30 September 2016, GL acquired a plant from SL in exchange of a building which was
currently not in use of GL. The details of plant and building are as follows:

Accumulated
Cost *Exchange price
depreciation
------------------- Rs. in million ------------------------
Building 240 130 120
Plant 200 80 120
* Equivalent to fair value

Both companies follow cost model for subsequent measurement of property, plant and
equipment and charge depreciation on building and plant at 5% and 20% respectively on cost.
(v) SL paid an interim cash dividend of 10% on 31 July 2016.
(vi) GL values non-controlling interest at the acquisition date at its fair value.

Required
Prepare a consolidated statement of financial position as at 31 December 2016 in accordance with
the requirements of International Financial Reporting Standards.

4.8 YASIR LIMITED AND BILAL LIMITED


Following information has been extracted from the financial statements of Yasir Limited (YL) and
Bilal Limited (BL) for the year ended 30 June 2016.
YL BL YL BL
Assets Equity & Liabilities
Rs. in million Rs. in million
Fixed assets 250 540 Share capital (Rs. 10 each) 750 500
Accumulated depreciation (70) (70) Retained earnings 340 258
180 470 1,090 758
Investment in BL – at cost 675 - Loan from YL - 12
Loan to BL 16 - Creditors & other liabilities 75 51
Stock in trade 160 150
Other current assets 71 50
Cash and bank 63 151
1,165 821 1,165 821

© Emile Woolf International 27 The Institute of Chartered Accountants of Pakistan


Financial accounting and reporting II

Additional information:
(i) On 1 July 2014, YL acquired 75% shares of BL at Rs. 18 per share. On the acquisition date,
fair value of BL’s net assets was equal to its book value except for an office building whose
fair value exceeded its carrying value by Rs. 12 million. Both companies provide depreciation
on building at 5% on straight line basis.
(ii) Year-wise net profit of both companies are given below:
2016 2015
-------- Rs. in million --------
YL 219 105
BL 11 168
(iii) The following inter-company sales were made during the year ended 30 June 2016:
Included in buyer’s
Sales
closing stock in trade Profit %
------------ Rs. in million ------------
YL to BL 120 20 30% on cost
BL to YL 80 32 15% on sale
(iv) BL declared interim dividend of 12% in the year 2015 and final dividend of 20% for the year
2016.
(v) The loan was granted by YL to BL on 1 July 2014 and carries interest rate of 12% payable
annually. The principal is repayable in five equal annual instalments of Rs. 4 million each. On
30 June 2016, BL issued a cheque of Rs. 5.92 million which was received by YL on 2 July
2016. No interest has been accrued by YL.
(vi) YL values non-controlling interest on the date of acquisition at its fair value. BL’s share price
was Rs. 15 on acquisition date.
(vii) An impairment test has indicated that goodwill of BL was impaired by 10% on 30 June 2016.
There was no impairment during the previous year.

Required:
Prepare a consolidated statement of financial position as at 30 June 2016 in accordance with the
requirements of International Financial Reporting Standards.

4.9 JASMEEN LIMITED


Following are the draft statement of financial position of Jasmine Limited (JL) and its subsidiary,
Sunflower Limited (SL) as on 31 December 2017:
JL SL
------ Rs. in million ------
Property, plant and equipment 880 330
Intangible assets 40 50
Investment in SL 520 -
Loan to JL - 120
Current assets 640 345
2,080 845
Share capital (Rs. 10 each) 700 200
Share premium 240 -
Retained earnings 720 410
Loan from SL 96 -
Current liabilities 324 235
2,080 845

© Emile Woolf International 28 The Institute of Chartered Accountants of Pakistan


Questions

Additional information:
(i) JL acquired 75% shares of SL on 1 January 2017. Cost of investment in JL’s books consists
of:
 10 million JL's ordinary shares issued at Rs. 24 per share; and
 cash payment of Rs. 280 million (including professional fee of Rs. 10 million for advice on
acquisition of SL)
(ii) On acquisition date, carrying value of SL's net assets was equal to fair value except an
intangible asset (brand) whose fair value was Rs. 40 million as against carrying value of Rs.
25 million. The remaining useful life of the brand is estimated at 5 years. The recoverable
amount of the brand at 31 December 2017 was estimated at Rs. 28 million.
(iii) JL values non-controlling interest at fair value. The market price of SL's shares was Rs. 36 at
the date of acquisition, which has increased to Rs. 40 as of 31 December 2017.
(iv) JL and SL showed a net profit of Rs. 200 million and Rs. 60 million respectively for the year
ended 31 December 2017.
(v) The loan was granted on 1 July 2017 and carries mark-up of 10% per annum. A cheque of
Rs. 30 million including interest was dispatched by JL on 31 December 2017 but was received
by SL after the year end. No interest has been accrued by SL in its financial statements.
(vi) On 1 May 2017 SL sold a machine to JL for Rs. 52 million at a gain of Rs. 12 million.
However, no payment has yet been made by JL. The remaining useful life of the machine at
the time of disposal was 2 years.
(vii) During the year, JL made sales of Rs. 250 million to SL at 20% above cost. 60% of these
goods are included in SL’s closing stock.
(viii) SL declared interim cash dividend of 10% in November 2017 which was paid on 2 January
2018. The dividend has correctly been recorded by both companies.

Required:
Prepare JL's consolidated statement of financial position as at 31 December 2017.

4.10 BRADLEY LTD


Bradley Ltd’s purchased 960 million shares in Bliss Ltd a year ago when Bliss had a credit balance
of Rs. 190million in retained earnings. The fair value of the non-controlling interest at the date of
acquisition was Rs. 330million. At the date of acquisition, the freehold land of Bliss Ltd was valued
at Rs. 140million in excess of its carrying value. The revaluation has not been recorded in the
accounts of Bliss.
The statements of financial position of Bradley Ltd and Bliss Ltd as at 31 December 2016 are as
follows:

Bradley Ltd Bliss Ltd


------------------- Rs. Million -------------------
Non Current Assets
Land and building 630 556
Machinery and equipment 570 440
Investment in Bliss Ltd. 1,320 -
2,520 996
Current Assets
Inventories 714 504
Trade receivables 1,050 252
Cash/bank 316 2,080 60 816
4,600 1,812

© Emile Woolf International 29 The Institute of Chartered Accountants of Pakistan


Financial accounting and reporting II

Bradley Ltd Bliss Ltd


------------------- Rs. Million -------------------
Ordinary Shares at Rs. 1 each 3,000 1,200
Retained Earnings 1,160 424
Shareholders fund 4,160 1,624

Current Liabilities
Trade payables 440 188
4,600 1,812

Bliss Ltd owes Bradley Ltd Rs. 50million for goods purchased during the year. Inventory of Bliss Ltd
includes goods bought from Bradley Ltd at the price that includes a profit to Bradley Ltd of Rs.
24million.
The management of Bradley Ltd wants the financial statements to be consolidated and wishes to
know whether there is goodwill on acquisition of Bliss Ltd and the amount involved.

Required
Prepare the consolidated statement of financial position as at 31 December 2016.

© Emile Woolf International 30 The Institute of Chartered Accountants of Pakistan


Questions

CHAPTER 5 – CONSOLIDATED ACCOUNTS: STATEMENTS OF COMPREHENSIVE


INCOME
5.1 HARRY
The following are the statements of profit or loss for the year ended 31 December 2015 of Harry and
its subsidiary Sally.

Harry Sally

Rs. 000 Rs. 000

Revenue 1,120 390

Cost of sales (610) (220)

Gross profit 510 170

Distribution costs (50) (40)

Administration costs (55) (45)

Operating profit 405 85

Investment income 20 4

Finance costs (18) (4)

Profit before tax 407 85

Income tax expense (140) (25)

Profit for the year 267 60

Rs. 000 Rs. 000

Retained profit brought forward 100 45

Profit for year 267 60

Dividends paid and proposed (50) (20)

Retained profit carried forward 317 85

The following information is relevant.


(1) Harry acquired 75% of Sally six years ago when Sally’s retained earnings were Rs. 9,000.
(2) Harry made sales to Sally totalling Rs. 100,000 in the year. At the year end the statement of
financial position of Sally included inventory purchased from Harry. Harry had taken a profit of
Rs. 3,000 on this inventory.
(3) Harry’s investment income includes Rs. 15,000 being its share of Sally’s dividends.

Required
Prepare a consolidated statement of profit or loss and a working showing the movement on
consolidated retained profit for the year ended 31 December 2015.

© Emile Woolf International 31 The Institute of Chartered Accountants of Pakistan


Financial accounting and reporting II

5.2 HORNY
Statements of profit or loss for the year ended 31 December 2015.

Horny Smooth
Rs. 000 Rs. 000
Revenue 304,900 195,300
Cost of sales (144,200) (98,550)
Gross profit 160,700 96,750
Operating costs (76,450) (52,100)
Operating profit 84,250 44,650
Investment income 10,500 2,600
Profit before tax 94,750 47,250
Income tax expense(42,900) (16,500)
Profit for the year 51,850 30,750

Statement of changes in equity (extracts) for the year ended 31 December 2015.

Horny Smooth
Rs. 000 Rs. 000
Retained earnings brought forward 80,200 31,000
Profit for the year 51,850 30,750
Proposed ordinary dividend (20,000) -

112,050 61,750

The following information is also available.


(1) Horny acquired 75% of the share capital of Smooth on 31 August 2015.
(2) Negative goodwill of Rs. 3.8 million arose on the acquisition.
(3) Profits of both companies are deemed to accrue evenly over the year except for the
investment income of Smooth all of which was received in November 2015.
(4) Horny has bought goods from Smooth throughout the year at Rs. 2 million per month. At the
year-end Horny does not hold any inventory purchased from Smooth.

Required
Prepare the consolidated statement of profit or loss and a working showing the movement on
consolidated retained profit for the year ended 31 December 2015.

5.3 HANKS
Statements of financial position as at 31 December 2015
Hanks Streep Scott
Rs. 000 Rs. 000 Rs. 000
Assets
Non-current assets
Property, plant and equipment 32,000 25,000 20,000
Investments 33,500 – –
———– ——— ———
65,500 25,000 20,000

© Emile Woolf International 32 The Institute of Chartered Accountants of Pakistan


Questions

Hanks Streep Scott


Rs. 000 Rs. 000 Rs. 000
Current assets
Cash at bank and in hand 9,500 2,000 4,000
Trade receivables 20,000 8,000 17,000
Inventory 30,000 18,000 18,000
———– ——— ———
125,000 53,000 59,000
———– ——— ———
Equity and liabilities
Share capital 40,000 10,000 15,000
Share premium account 6,500 – –
Retained earnings 55,000 37,000 27,000
———– ——— ———
101,500 47,000 42,000
Current liabilities 23,500 6,000 17,000
———– ——— ———
125,000 53,000 59,000
═════ ═════ ═════
Statements of profit or loss for the year ended 31 December 2015
Hanks Streep Scott
Rs. 000 Rs. 000 Rs. 000
Revenue 125,000 117,000 82,000
Cost of sales (65,000) (64,000) (42,000)
———– ———– ———
Gross profit 60,000 53,000 40,000
Distribution costs (21,000) (14,000) (16,000)
Administrative expenses (14,000) (8,000) (7,000)
———– ———– ———
Profit before taxation 25,000 31,000 17,000
Income tax expense (10,000) (9,000) (5,000)
———– ———– ———
Profit after tax 15,000 22,000 12,000
═════ ═════ ═════
Statement of changes in equity (extract) for the year ending 31 December 2015
Hanks Streep Scott
Rs. 000 Rs. 000 Rs. 000
Retained earnings brought forward 40,000 15,000 15,000
Retained profit for the financial year 15,000 22,000 12,000
Dividends – – –
———– ———– ———
Retained earnings carried forward 55,000 37,000 27,000
═════ ═════ ═════
You are given the following additional information
(1) Hanks owns 80% of Streep’s shares. These were purchased in 2012 for Rs. 20.5 million cash,
when the balance on Streep’s retained earnings stood at Rs. 7million.

© Emile Woolf International 33 The Institute of Chartered Accountants of Pakistan


Financial accounting and reporting II

(2) In 2010 Hanks purchased 60% of the shares of Scott by the issue of shares with a nominal
value of Rs. 6.5 million. These shares were issued at a premium of Rs. 6.5 million. At that
date the retained earnings of Scott stood at Rs. 3 million and the fair value of the net assets of
Scott was Rs. 24 million. It was agreed that any undervaluation of the net assets should be
attributed to land. This land was still held at 31 December 2015.
(3) Included in the inventory of Scott and Streep at 31 December 2015 are goods purchased from
Hanks for Rs. 5.2 million and Rs. 3.9 million respectively. Hanks aims to earn a profit of 30%
on cost. Total sales from Hanks to Scott and to Streep were Rs. 8 million and Rs. 6 million
respectively.
(4) Hanks and Streep each proposed a dividend before the year end of Rs. 2 million and Rs. 2.5
million respectively. No accounting entries have yet been made for these.
(5) Hanks has carried out annual impairment tests on goodwill in accordance with IFRS 3 and
IAS 36. The estimated recoverable amount of goodwill at 31 December 2012 was Rs. 5
million and at 31 December 2015 was Rs. 4.5 million.

Required
Prepare the consolidated statement of profit or loss and consolidated statement of changes in equity
for the year ended 31 December 2015 and the consolidated statement of financial position at that
date.

5.4 OSCAR LIMITED


The summarized trial balances of Oscar Limited (OL) and United Limited (UL) as at 31 December
2015 are as follows:

Oscar Limited (OL) United Limited (UL)


Debit Credit Debit Credit
------------- Rs. in million -------------
Sales 835 645
Cost of sales 525 396
Operating expense 115 102
Tax expense 65 48
Share capital (Rs. 10 each) 600 250
Share premium 150 60
Retained earnings as at 1 January 2015 265 179
Current liabilities 115 105
Property, plant and equipment 390 350
Cost of investment 500
Stock-in-trade 125 115
Trade receivables 140 125
Cash and bank 105 103
1,965 1,965 1,239 1,239

Additional information:
(i) On 1 May 2015, OL acquired 80% shares of UL. UL has not recognised the value of
brand in its books of account. At the date of acquisition, the fair value of brand was assessed
at Rs. 45 million. The remaining useful life of the brand was estimated as 15 years.

© Emile Woolf International 34 The Institute of Chartered Accountants of Pakistan


Questions

(ii) OL charged Rs. 2.5 million monthly to UL for management services provided from the date of
acquisition and has credited it to operating expenses.
(iii) On 1 October 2015, UL sold a machine to OL for Rs. 24 million. The machine had been
purchased on 1 October 2013 for Rs. 26 million. On the date of acquisition the machine was
assessed as having a useful life of ten years and that estimate has not changed. Gain on
disposal was erroneously credited to sales account.
(iv) Other inter-company transactions during the year 2015 were as follows:

Included in buyer’s closing


Sales
stock-in-trade Profit %
------------ Rs. in million ------------
OL to UL 60 20 25% of cost
UL to OL 30 5 20% of sales

UL settled the inter-company balance as on 31 December 2015 by issuing a cheque of Rs.


30 million. However, the cheque was received by OL on 1 January 2016.
(i) The non-controlling interest is measured at the proportionate share of UL’s identifiable net
assets.
It may be assumed that profits of both companies had accrued evenly during the year.

Required:
Prepare consolidated statement of comprehensive income for the year ended 31 December 2015
and consolidated statement of financial position as at 31 December 2015.

5.5 PRESENT LIMITED AND FUTURE LIMITED


The following balances are extracted from the records of Present Limited (PL) and Future Limited
(FL) for the year ended 30 June 2017:

PL FL
Debit Credit Debit Credit
--------------- Rs. In million ---------------
Sales 2,060 1,524
Cost of sales 1,300 846
Selling and administrative expenses 350 225
Investment income 190 50
Gain on disposal of fixed assets - net 35
Taxation 80 60
Share capital (Rs. 10 each) 3,500 2,600
Retained earnings as on 30 June 2017 1,996 704

Additional information:
(i) PL acquired 65% shares of FL on 1 September 2016 against the following consideration:
 Cash payment of Rs. 900 million.
 Issuance of shares having nominal value of Rs. 1,000 million.
The fair value of each share of PL and FL on acquisition date was Rs. 16 and Rs. 12
respectively. Retained earnings of PL and FL on the acquisition date were Rs. 1,671 million
and Rs. 506.5 million respectively.

© Emile Woolf International 35 The Institute of Chartered Accountants of Pakistan


Financial accounting and reporting II

At acquisition date, fair value of FL’s net assets was equal to their book value except a
brand which had not been recognised by FL. The fair value of the brand is assessed at Rs.
90 million. PL estimates that benefit would be obtained from the brand for the next 10 years.
(ii) The incomes and expenses of FL had accrued evenly during the year except investment
income. The investment income is exempt from tax and had been recognised in August
2016 and received in September 2016.
(iii) On 1 January 2017 PL sold a manufacturing plant having carrying value of Rs. 42 million to
FL against cash consideration of Rs. 30 million. The plant had a remaining useful life of 6
years on the date of disposal.
(iv) On 1 February 2017 FL delivered goods having sale price of Rs. 100 million to PL on ‘sale
or return basis’. 40% of these goods were returned on 1 May 2017 and the remaining were
accepted by PL. 20% of the goods accepted were included in the closing inventory of PL.
FL earned a profit of 33.33% on cost.
(v) Both companies paid interim cash dividend at the rate of 5% in May 2017.
(vi) An impairment test carried out at year end has indicated that goodwill of FL has been
impaired by 10%.
(vii) PL measures the non-controlling interest at its fair value.

Required:
a) Prepare consolidated statement of profit or loss for the year ended 30 June 2017.
b) Compute the amounts of consolidated retained earnings and non-controlling interest as would
appear in the consolidated statement of financial position as at 30 June 2017.

5.6 FATIMA LIMITED AND ALI LIMITED


Following is the summarised trial balance of Fatima Limited (FL) and its subsidiary, Ali Limited (AL)
for the year ended December 31, 2018:
FL AL
-------Rs. in million-------
Cash and bank balances 4,920 2,700
Accounts receivable 6,240 6,580
Stocks in trade – closing 14,460 5,680
Investment in AL 10,500 -
Other investments 20,100 -
Property, plant and equipment 22,500 5,940
Cost of sales 49,200 21,000
Operating expenses 3,600 5,400
Accumulated depreciation (5,760) (1,260)
Ordinary share capital (Rs. 10 each) (30,000) (6,000)
Retained earnings – opening (33,780) (4,800)
Sales (57,600) (33,800)
Accounts payable (2,760) (1,440)
Gain on sale of fixed assets (540) -
Dividend income (1,080) -

© Emile Woolf International 36 The Institute of Chartered Accountants of Pakistan


Questions

Following additional information is also available:


i. On January 1, 2018, FL acquired 480 million shares of AL from its major shareholder for
Rs.10,500 million.
ii. The following inter company sales were made during the year 2018:

Sales Included in buyer’s Amount receivable/ Gross profit%


closing stocks in trade payable at year end on Sales
---------------------Rs. in million---------------------
FL to AL 2,400 900 300 20
AL to FL 3,600 1,200 500 30
FL and AL value stock in trade at the lower of cost or net realisable value. While valuing FL’s
stock in trade, the stock purchased from AL has been written down by Rs. 100 million.
iii. On July 1, 2018, FL sold certain plants and machineries to AL. Details of the transaction are as
follows:

Rs. in million
Sales value 144

Less: Cost of plant and machineries 150


Accumulated depreciation (60)
Net book value 90
Gain on sale of plant 54

iv. The plants and machineries were purchased on January 1, 2016, and were being depreciated
on straight line method over a period of five years. AL computed depreciation thereon using the
same method based on the remaining useful life.
v. FL billed Rs. 100 million to each subsidiary for management services provided during the year
2018 and credited it to operating expenses. The invoices were paid on December 15, 2018.
vi. Details of cash dividend are as follows:

Dividend
Date of declaration Date of payment %
FL November 25, 2018 January 5, 2019 20
AL October 15, 2018 November 20, 2018 10

Required:
Prepare consolidated statement of financial position and statement of comprehensive income of FL
for the year ended December 31, 2018. Ignore tax and corresponding figures.

© Emile Woolf International 37 The Institute of Chartered Accountants of Pakistan


Financial accounting and reporting II

CHAPTER 6: IAS 28: INVESTMENT IN ASSOCIATES AND JOINT VENTURES


6.1 HELIUM
The draft statements of financial position as at 31 December 2016 of three companies are set out
below.
Helium Sulphur Arsenic
Rs.000 Rs.000 Rs.000
Assets
Non-current assets
Property, plant and equipment 400 100 160
Investments:
- shares in Sulphur (60%) 75 – –
- shares in Arsenic (30%) 30 – –

Current assets 445 160 80


—— —— ——
950 260 240
—— —— ——
Equity and liabilities
Share capital 100 30 60
Retained earnings 650 180 100
Non-current loans 200 50 80
—— —— ——
950 260 240
—— —— ——
The reserves of Sulphur and Arsenic when the investments were acquired were Rs. 70,000 and Rs.
30,000 respectively

Required
Prepare the consolidated statement of financial position as at 31 December 2016.

6.2 HAMACHI LTD


Hamachi Ltd acquired 90% of Saba Ltd’s Rs. 1 ordinary shares on 1 April 2014 paying Rs. 3.00 per
share. The balance on Saba Ltd’s retained earnings at this date was Rs. 800,000. On 1 October
2015, Hamachi Ltd acquired 30% of Anogo Ltd’s Rs. 1 ordinary shares for Rs. 3.50 per share. The
statements of financial position of the three companies at 31 March 2016 are shown below:

Hamachi Ltd Saba Ltd Anogo Ltd


Rs.000 Rs.000 Rs.000 Rs.000 Rs.000 Rs.000
Non-current assets
Property, plant and equipment 8,050 3,600 1,650
Investments 4,000 910 nil
12,050 4,510 1,650
Current assets
Inventory 830 340 250
Accounts receivable 520 290 350
Bank 240 nil 100
1,590 630 700
Total assets 13,640 5,140 2,350

© Emile Woolf International 38 The Institute of Chartered Accountants of Pakistan


Questions

Hamachi Ltd Saba Ltd Anogo Ltd


Rs.000 Rs.000 Rs.000 Rs.000 Rs.000 Rs.000
Equity and liabilities
Equity:
Ordinary shares of Rs. 1 each 5,000 1,200 600
Reserves:
Retained earnings b/f 6,000 1,400 800
Profit year to 31 March 2016 1,500 900 600
7,500 2,300 1,400
12,500 3,500 2,000
Non-current liabilities
10% Loan notes 500 240 nil
Current liabilities
Accounts payable 420 960 200
Taxation 220 250 150
Overdraft nil 190 nil
640 1,400 350
Total equity and liabilities 13,640 5,140 2,350

The following information is relevant


(i) Fair value adjustments
On 1 April 2014 Saba Ltd owned an investment property that had a fair value of Rs. 120,000
in excess of its carrying value (book value). The value of this property has not changed since
acquisition. This property is included within investments in the balance sheet.
Just prior to its acquisition, Saba Ltd was successful in applying for a six-year licence to
dispose of hazardous waste. The licence was granted by the government at no cost, however
Hamachi Ltd estimated that the licence was worth Rs. 180,000 at the date of acquisition.
(ii) In January 2016 Hamachi Ltd sold goods to Anogo Ltd for Rs. 65,000. These were transferred
at a mark-up of 30% on cost. Two thirds of these goods were still in the inventory of Anogo
Ltd at 31 March 2016.
(iii) To facilitate the consolidation procedures the group insists that all inter company current
account balances are settled prior to the year-end. However a cheque for Rs. 40,000 from
Saba Ltd to Hamachi Ltd was not received until early April 2016. Inter company balances are
included in accounts receivable and payable as appropriate.
(iv) Anogo Ltd is to be treated as an associated company of Hamachi Ltd.
(v) An impairment test at 31 March 2016 on the consolidated goodwill of Saba Ltd concluded that
it should be written down by Rs. 468,000. No other assets were impaired.

Required
(a) Prepare the consolidated statement of financial position of Hamachi Ltd as at 31 March 2016.

(b) Discuss the matters to consider in determining whether an investment in another company
constitutes associated company status.

© Emile Woolf International 39 The Institute of Chartered Accountants of Pakistan


Financial accounting and reporting II

6.3 HIDE
Hide holds 80% of the ordinary share capital of Seek (acquired on 1 February 2016) and 30% of the
ordinary share capital of Arrive (acquired on 1 July 2015).
Hide had no other investments.
The draft statements of profit or loss for the year ended 30 June 2016, are set out below.
Hide Seek Arrive
Rs.000 Rs.000 Rs.000
Revenue 12,614 6,160 8,640
Operating expenses (11,318) (5,524) (7,614)
Dividends receivable 150 – –
——— ——– ——–
1,446 636 1,026
Income tax (621) (275) (432)
——— ——– ——–
Profit after taxation 825 361 594
——— ——– ——–
Included in the inventory of Seek at 30 June 2016 was Rs. 50,000 for goods purchased from Hide in
May 2016 which the latter company had invoiced at cost plus 25%. These were the only goods sold
by Hide to Seek but it did make sales of Rs. 180,000 to Arrive during the year. None of these goods
remained in Arrive’s inventory at the year end.

Required
Prepare a consolidated statement of profit or loss for Hide for the year ended 30 June 2016.

6.4 HARK, SPARK AND ARK


Hark acquired the following non-current investments on 1 April 2015:
(1) 4 million equity shares in Spark, by means of an exchange of one share in Handel for every
one share in Spark, plus Rs. 6.05 million in cash. The professional fees associated with the
acquisition amounted to Rs. 1 million. The market price of shares in Hark at the date of the
acquisition was Rs. 9 per share. The market price of Spark shares just before the acquisition
was Rs. 7. The cash part of the consideration is deferred and will not be paid until two years
after the acquisition.
(2) 25% of the equity shares in Ark, at a cost of Rs. 6 per share. The money to make this
payment was obtained by issuing one million new shares in Hark at Rs. 9 per share.
None of these transactions has yet been recorded in the summary statements of financial position
that are shown below.
The summarised draft statements of financial position of the three companies at 31 March 2016 are
as follows.

Statement of financial position Hark Spark Ark


Rs. million Rs. million Rs. million
Assets
Non-current assets
Property, plant and equipment 60.0 31.0 16.0
Other equity investments 0.8 nil nil
60.8 31.0 16.0
Current assets 18.2 8.0 9.0
Total assets 79.0 39.0 25.0

© Emile Woolf International 40 The Institute of Chartered Accountants of Pakistan


Questions

Statement of financial position Hark Spark Ark


Rs. million Rs. million Rs. million

Equity and liabilities

Equity shares of Rs. 1 each 16.0 5.0 6.0

Share premium 2.0 4.0 4.0

Retained earnings: at 1 April 2015 36.0 16.0 8.0

- for year ended 31 March 2016 8.0 3.0 2.0

62.0 28.0 20.0

Non-current liabilities

6% loan notes 10.0 - -

7% loan notes - 6.0 3.0

Current liabilities 7.0 5.0 2.0

Total equity and liabilities 79.0 39.0 25.0

The following information is relevant:


(1) Hark has chosen to value the non-controlling interest in Spark using the fair value method
permitted by IFRS 3 (revised). The fair value of the non-controlling interests at the acquisition
date is estimated to be the market value of the shares before the acquisition.
(2) At the date of acquisition of Spark, the fair values of its assets were equal to their carrying
amounts.
(3) The cost of capital of Hark is 10% per year.
(4) During the year ended 31 March 2016, Spark sold goods to Hark for Rs. 3.6 million, at a
mark-up of 50% on cost. Hark had 75% of these goods in its inventory at 31 March 2016.
(5) There were no intra-group receivables and payables at 31 March 2016.
(6) On 1 April 2015, Hark sold a group of machines to Spark at their agreed fair value of Rs. 3
million. At the time of the sale, the carrying amount of the machines was Rs. 2 million. The
estimated remaining useful life of the plant at the date of the sale was four years. Plant and
machinery is depreciated to a residual value of nil using straight-line depreciation and at 1
April 2015 the machines had an estimated remaining life of five years.
(7) “Other equity investments” are included in the summary statement of financial position of Hark
at their fair value on 1April 2015. Their fair value at 31 March 2016 is Rs.0.65 million.
(8) Impairment tests were carried out on 31 March 2016. These show that there is no impairment
of the value of the investment in Ark or in the consolidated goodwill.
(9) No dividends were paid during the year by any of the three companies.

Required
Prepare the consolidated statement of financial position for Hark as at 31 March 2016.

© Emile Woolf International 41 The Institute of Chartered Accountants of Pakistan


Financial accounting and reporting II

6.5 P, S AND A
The statements of financial position of three entities P, S and A are shown below, as at 31
December Year 5. However, the statement of financial position of P records its investment in Entity
A incorrectly.

P S A
Rs. Rs. Rs.
Non-current assets
Property, plant and equipment 450,000 240,000 460,000
Investment in S at cost 320,000 - -
Investment in A at cost 140,000 - -
––––––––– ––––––––– –––––––––
910,000 240,000 460,000
Current assets
Inventory 70,000 90,000 70,000
Current account with P - 60,000 -
Current account with A 20,000 - -
Other current assets 110,000 130,000 40,000
––––––––– ––––––––– –––––––––
Total assets 1,110,000 520,000 570,000
––––––––– ––––––––– –––––––––
Equity and reserves
Equity shares of Rs. 1 100,000 200,000 100,000
Share premium 160,000 80,000 120,000
Accumulated profits 650,000 140,000 250,000
––––––––– ––––––––– –––––––––
910,000 420,000 470,000
Long-term liabilities 40,000 20,000 30,000
Current liabilities
Current account with P - - 20,000
Current account with S 60,000 - -
Other current liabilities 100,000 80,000 50,000
––––––––– ––––––––– –––––––––
1,110,000 520,000 570,000
––––––––– ––––––––– –––––––––

Additional information
P bought 150,000 shares in S several years ago when the fair value of the net assets of S was Rs.
340,000.
P bought 30,000 shares in A several years ago when A’s accumulated profits were Rs. 150,000.
There has been no change in the issued share capital or share premium of either S or A since P
acquired its shares in them.
There has been impairment of Rs. 20,000 in the goodwill relating to the investment in S, but no
impairment in the value of the investment in A.

© Emile Woolf International 42 The Institute of Chartered Accountants of Pakistan


Questions

At 31 December Year 5, A holds inventory purchased during the year from P which is valued at Rs.
16,000 and P holds inventory purchased from S which is valued at Rs. 40,000. Sales from P to A
and from S to P are priced at a mark-up of one-third on cost.
None of the entities has paid a dividend during the year.
P uses the partial goodwill method to account for goodwill and no goodwill is attributed to the non-
controlling interests in S.

Required
Prepare the consolidated statement of financial position of the P group as at 31 December Year 5.

6.6 BL, ML AND ZL


Bilal Limited (BL), acquired a subsidiary, Mishall Limited (ML), on July 01, 2014 and an associate,
Zoha Limited (ZL), on January 01, 2017. The details of the acquisition at the respective dates are as
follows: -

Fair value of Ordinary


Ordinary Reserves
Share net assets Cost of Share
Investment share Retained
Premium at investment capital
capital earnings
acquisition acquired
Re. 1
------------------------------Rs. in million---------------------------------
each

ML 400 800 765 320


160 140

ZL 220 652 203 55


269 83

The draft financial statements for the year ended June 30, 2018 are: -
Statement of financial position
as at June 30, 2018
BL ML ZL
---------------Rs. in million---------------
Non-current assets
Property, plant and equipment 1,012 920 442
Intangible assets - 350 27
Investment in ML 765 - -
Investment in ZL 203 - -
1,980 1,270 469

Statement of financial position


as at June 30, 2018
BL ML ZL
---------------Rs. in million---------------
Current assets
Inventories 620 1,460 214
Trade receivables 950 529 330
Cash and cash equivalents 900 510 45
2,470 2,499 589
4,450 3,769 1,058

© Emile Woolf International 43 The Institute of Chartered Accountants of Pakistan


Financial accounting and reporting II

Statement of financial position


as at June 30, 2018
BL ML ZL
---------------Rs. in million---------------
Equity
Share capital 1,000 400 220
Share premium 200 140 83
Retained earnings 1,370 929 361
2,570 1,469 664
Current liabilities
Trade and other payables 1,880 2,300 394
4,450 3,769 1,058

Statement of comprehensive income


for the year ended June 30, 2018
BL ML ZL
---------------Rs. in million---------------
Revenue 4,480 4,200 1,460
Cost of
(2,690) (2,940) (1,020)
sales
Gross profit 1,790 1,260 440
Distribution and administrative cost (620) (290) (196)
Finance cost (50) 80 (24)
Dividend income 260 - -
Profit before tax 1,380 1,050 220
Income tax expense (330) (274) (72)
Profit for the year 1,050 776 148
Dividend paid for the year 250 300 80
Retained earnings brought forward 570 613 293

Additional information:
a) The BL Group has the policy of measuring NCI at fair value at the date of acquisition and Fair
Value of NCI was Rs. 210 million at the date of acquisition.
b) Neither ML nor ZL had reserves other than retained earnings and share premium at the date of
acquisition. Neither issued new shares since acquisition.
c) The fair value difference on the subsidiary relates to property, plant and equipment being
depreciated through cost of sales over the remaining useful life of 10 years from the acquisition
date. The fair value difference on the associate relates to a piece of land which has not been
sold since acquisition.
d) ML’s intangible assets include Rs. 87 million of training and marketing cost incurred during the
year ended June 30, 2018. The directors of ML believe that these should be capitalized as they
relate to the startup period of a new business, and intend to amortize the balance over five years
from July 01, 2018.

© Emile Woolf International 44 The Institute of Chartered Accountants of Pakistan


Questions

e) During the year ended June 30, 2018 ML sold goods to BL for Rs. 1,300 million. The company
makes a profit of 30% on the selling price. Rs. 140 million of these goods were held by BL on
June 30, 2018 (Rs. 60 million on June 30, 2017).
f) BL sold goods worth Rs. 1,000 to ZL during the year by charging 25% margin on sales, 10% of
the goods still remains unsold by ZL.
g) Annual impairment tests have indicated impairment losses of Rs. 100 million relating to the
recognized goodwill of ML including Rs. 25 million in the current year. No impairment losses to
date have been necessary for the investment in ZL.

Required:
Prepare the Consolidated statement of financial position and the statement of comprehensive
income for the year ended June 30, 2018 for the BL Group.

6.7 MANTO LIMITED AND HALI LIMITED


Qudsia Limited (QL) has investments in two companies as detailed below:
Manto Limited (ML)
 On 1 January 2010, QL acquired 40 million ordinary shares in ML, when its retained earnings
were Rs. 150 million.
 The fair value of ML’s net assets on the acquisition date was equal to their carrying amounts.
Hali Limited (HL)
 On 30 November 2012, QL acquired 16 million ordinary shares in HL, when its retained
earnings stood at Rs. 224 million.
 The purchase consideration was made up of:
 Rs. 190 million in cash, paid on acquisition; and
 4 million shares in QL. At the date of acquisition, QL’s shares were being traded at
Rs. 15 per share but the price had risen to Rs. 16 per share by the time the shares
were issued on 1 January 2013.
 The fair value of the net assets of HL on the date of acquisition by QL was equal to their
carrying amounts, except a building whose fair value exceeded its carrying amount by Rs. 28
million. The building had a remaining useful life of seven years on 30 November 2012.
The draft summarised statements of financial position of the three companies on 31 December 2012
are shown below:

QL ML HL
---------Rs. in million---------
Assets
Property, plant and equipment 5,000 550 500
Investment in ML 630 - -
Investment in HL 190 - -
Current assets 5,480 400 350
11,300 950 850
Equity and liabilities
Ordinary share capital (Rs.10 each) 6,000 500 400
Retained earnings 2,900 100 240
Current liabilities 2,400 350 210
11,300 950 850

© Emile Woolf International 45 The Institute of Chartered Accountants of Pakistan


Financial accounting and reporting II

The following additional information is available:


i. QL considers ML as a cash-generating unit (CGU). As on 31 December 2012, the
recoverable amount of the CGU was estimated at Rs. 700 million.
ii. QL values the non-controlling interest at its proportionate share of the fair value of the
subsidiary’s net identifiable assets.
iii. On 1 October 2012, ML sold a machine to QL for Rs. 24 million. The machine had been
purchased on 1 October 2010 for Rs. 26 million. The machine was originally assessed as
having a useful life of ten years and that estimate has not changed.
iv. In December 2012, QL sold goods to HL at cost plus 30%. The amount invoiced was Rs. 52
million. These goods remained unsold at year end and the invoiced amount was also paid
subsequent to the year end.

Required:
Prepare a consolidated statement of financial position for QL as on 31 December 2012 in
accordance with the requirements of International Financial Reporting Standards.

© Emile Woolf International 46 The Institute of Chartered Accountants of Pakistan


Questions

CHAPTER 7 – IAS 8: ACCOUNTING POLICIES, CHANGES IN ACCOUNTING


ESTIMATES AND ERRORS
7.1 WONDER LIMITED
Wonder Limited (WL) is engaged in the manufacturing and sale of textile machinery. Following are
the draft extracts of the statement of financial position and the statement of profit or loss for the year
ended 30 June 2015:
Statement of Financial Position

2015 2014

Rs. m Rs. m

Property, plant and equipment 189 130

Retained earnings 166 108

Deferred tax liability 45 27

Statement of profit or loss

2015 2014

Rs. m Rs. m

Profit before taxation 90 120

Taxation 32 42

Profit after taxation 58 78

Following additional information has not been taken into account in the preparation of the
above financial statements:
(i) Cost of repairs amounting to Rs. 20 million was erroneously debited to the machinery
account on 1 October 2013. The estimated useful life of the machine is 10 years.
(ii) On 1 July 2014, WL reviewed the estimated useful life of its plant and revised it from 5
years to 8 years. The plant was purchased on 1 July 2013 at a cost of Rs. 70 million.
Depreciation is provided under the straight line method. Applicable tax rate is 30%.

Required
Prepare relevant extracts (including comparative figures) for the year ended 30 June 2015 related to
the following:
(a) Statement of financial position
(b) Statement of profit or loss
(c) Statement of changes in equity
(d) Correction of error note

© Emile Woolf International 47 The Institute of Chartered Accountants of Pakistan


Financial accounting and reporting II

7.2 MOHANI MANUFACTURING LIMITED


Mohani Manufacturing Limited is engaged in manufacturing of spare parts for motor car
assemblers. The audited financial statements for the year ended December 31, 2014 disclosed that
the profit and retained earnings were Rs. 21 million and Rs. 89 million respectively. The draft
financial statements for the year show a profit of Rs. 15 million. However, following adjustments are
required to be made:
(i) The management of the company has decided to change the method for valuation of raw
materials from FIFO to weighted average. The value of inventory under each method is as
follows:

FIFO Weighted Average


Rs. m Rs. m
December 31, 2013 37.0 35.5
December 31, 2014 42.3 44.5
December 31, 2015 58.4 54.4
(ii) In 2014, the company purchased a plant for Rs. 100 million. Depreciation on plant was
recorded at Rs. 25 million instead of Rs. 10 million. This error was discovered after the
publication of financial statements for the year ended December 31, 2014. The error is
considered to be material.

Required
Produce an extract showing the movement in retained earnings, as would appear in the
statement of changes in equity for the year ended December 31, 2015.

7.3 MARVELLOUS LIMITED


Following information has been extracted from the draft financial statements of Marvellous Limited
(ML) for the year ended 30 June 2017:
Statement of financial position
2017 2016
Rs. in million
Property, plant and equipment 700 612
Retained earnings 275 240
Deferred tax liability 58 52
Provision for taxation 12 16
Statement of profit or loss
Profit before taxation 65 85
Taxation 30 25
Profit after taxation 35 60
The following matters are under consideration of the management:
 It was identified that ML’s obligation to incur decommissioning cost related to a plant has not
been recognised. The plant was acquired on 1 July 2014 and had been depreciated on straight
line basis over a useful life of four years. The expected cost of decommissioning at the end of
the life is Rs. 50 million. Applicable discount rate is 8%.
 In view of significant change in the expected pattern of economic benefits from an item of the
equipment, it has been decided to change the depreciation method from reducing balance to
straight line. The equipment was purchased on 1 July 2015 at a cost of Rs. 80 million having
estimated useful life of 5 years and residual value of Rs. 16 million. The depreciation at the rate
of 27.5% on reducing balance method is included in the above draft financial statements.

© Emile Woolf International 48 The Institute of Chartered Accountants of Pakistan


Questions

The following balances pertain to ML’s statement of financial position as on 30 June 2015:
Rs. in million
Property, plant and equipment 650
Retained earnings 180
Deferred tax liability 40
Provision for taxation 24
Applicable tax rate is 30%. Tax authorities consider decommissioning cost as an expense
when paid.

Required
Prepare extracts from the following (including comparative figures) for the year ended 30 June 2017:
(a) Statement of financial position
(b) Statement of profit or loss
(c) Correction of error note

7.4 CHAND PAINTS LIMITED


Chand Paints Limited (CPL) is engaged in the manufacturing of chemicals and paints. In April 2016
it was discovered that certain errors had been made in the financial statements for the year ended
30 June 2015. The errors were corrected in 2016. The details are as follows:
2015
2016 2015
After correction
(Draft) Audited
of errors
---------- Rs. in million -----------
Statement of comprehensive income
Sales tax, commission and discounts (7,939) (8,246) (7,916)
Cost of sales (45,508) (44,606) (44,633)
Selling and distribution expenses (2,940) (2,635) (2,441)
Administration expenses (2,356) (2,254) (2,149)
Other operating charges (495) (467) (515)
Other operating income 920 427 509
Profit for the year 4,089 3,723 4,359

Statement of financial position


Trade and other receivables 1,839 1,613 2,025
Trade and other payables 11,600 8,894 8,670
The share capital and un-appropriated profit of CPL as on 1 July 2014 was Rs. 10,400 million and
Rs. 19,089 million respectively.
The details of dividend declared are as follows:

2016 2015
Cash dividend – Interim 10% 5%
– Final 15% 10%

Required
(a) Prepare a correction of error note to be included in the financial statements for the year ended
30 June 2016. (Ignore earnings per share and taxation)
(b) Prepare the statement of changes in equity for the year ended 30 June 2016.

© Emile Woolf International 49 The Institute of Chartered Accountants of Pakistan


Financial accounting and reporting II

7.5 DAFFODIL LIMITED


For the purpose of preparation of statement of changes in equity for the year ended 31 December
2017, Daffodil Limited (DL) has extracted the following information:
2017 2016 2015
Draft Audited Audited
--------- Rs. in million ---------
Net profit 650 318 214
Transfer to general reserves 112 - 141
Transfer of incremental depreciation - 49 55
Final cash dividend - - 7.5%

Additional information:
(i) Details of share issues:
 25% right shares were issued on 1 May 2016 at Rs. 18 per share. The market price per
share immediately before the entitlement date was also Rs. 18 per share.
 A bonus issue of 10% was made on 1 April 2017 as final dividend for 2016.
 50 million right shares were issued on 1 July 2017 at Rs. 15 per share. The market price
per share immediately before the entitlement date was Rs. 25 per share.
 A bonus issue of 15% was made on 1 September 2017 as interim dividend.
(ii) After preparing draft financial statements, it was discovered that depreciation on a plant costing
Rs. 700 million has been charged @ 25% under reducing balance method, from the date of
commencement of manufacturing i.e. 1 July 2014. However, the plant was available for use on 1
February 2014.
(iii) Share capital and reserves as at 31 December:

2015 2014
--------- Rs. in million ---------
Ordinary share capital (Rs. 10 each) 1,600 1,600
General reserves 1,850 1,709
Retained earnings 1,430 1,302

Required:
Prepare DL’s statement of changes in equity for the year ended 31 December 2017 along with
comparative figures. (Ignore taxation)

7.6 ASIF ENGINEERING LIMITED


The following information has been taken from the financial statements of Asif Engineering Limited
(AEL) for the year ended 31 December 2015:

2015 (draft) 2014 2013


---------- Rs. in million ----------
Property, plant equipment 2,430 2,402 2,105
Stores and spares 73 80 70
Retained earnings as at 31 December 353 224 101
Net profit 129 123 112

In the above financial statements, AEL has recognised consumption of spare parts as expense. AEL
has now decided to change its above policy and classify consumption of spares having useful life of
more than one year as capital spares under property, plant and equipment.

© Emile Woolf International 50 The Institute of Chartered Accountants of Pakistan


Questions

Following information pertains to capital spares consumed during the past three years:

Parts issued during the year Useful life of the


Year ended issued parts
Rs. in million

31 December 2013 55 5 years

31 December 2014 39 3 years

31 December 2015 44 4 years

Depreciation on these parts is to be charged using straight line method over its useful life.

Required:
In accordance with the requirements of International Financial Reporting Standards, prepare the
revised extracts (including comparative figures) of the following:
(a) Statement of financial position as at 31 December 2015
(b) statement of comprehensive income for the year ended 31 December 2015
(c) Statement of changes in equity for the year ended 31 December 2015 (ignore taxation)

© Emile Woolf International 51 The Institute of Chartered Accountants of Pakistan


Financial accounting and reporting II

CHAPTER 8 – IAS 12: INCOME TAXES


8.1 FRANCESCA
On 30 June 2014 Francesca Company had a credit balance on its deferred tax account of Rs.
1,340,600 all in respect of the difference between depreciation and capital allowances.
During the year ended 30 June 2015 the following transactions took place.
(1) Rs. 45 million was charged against profit in respect of depreciation. The tax computation
showed capital allowances of Rs. 50 million.
(2) Interest receivable of Rs. 50,000 was reflected in profit for the period. However, only Rs.
45,000 of interest was actually received during the year. Interest is not taxed until it is
received.
(3) Interest payable of Rs. 32,000 was treated as an expense for the period. However, only Rs.
28,000 of interest was actually paid during the year. Interest is not an allowable expense for
tax purposes until it is paid.
(4) During the year Francesca incurred development costs of Rs. 500,600, which it has
capitalised. Development costs are an allowable expense for tax purposes in the period in
which they are paid.
(5) Land and buildings with a net book value of Rs. 4,900,500 were revalued to Rs. 6 million.
The tax rate is 30%. Francesca has a right of offset between its deferred tax liabilities and its
deferred tax assets.

Required
Calculate the deferred tax liability on 30 June 2015. Show where the increase or decrease in the
liability in the year would be charged or credited.

8.2 SHEP (I)


Shep was incorporated on 1 January 2015. In the year ended 31 December 2015 the company
made a profit before taxation of Rs. 121,000
During the period Shep made the following capital additions.
Rs.
Plant 48,000
Motor vehicles 12,000
During the period:
Accounting depreciation 11,000
Tax depreciation 15,000
Tax is chargeable at a rate of 30%.

Required
(a) Calculate the corporate income tax liability for the year ended 31st December 2015.
(b) Calculate the deferred tax balance that is required in the statement of financial position as at
31st December 2015.
(c) Prepare a note showing the movement on the deferred tax account and thus calculate the
deferred tax charge for the year ended 31st December 2015
(d) Prepare the statement of profit or loss note which shows the compilation of the tax expense
for the year ended 31st December 2015.

© Emile Woolf International 52 The Institute of Chartered Accountants of Pakistan


Questions

8.3 SHEP (II)


Continuing from the previous year. The following information is relevant for the year ended 31 st
December 2016.
(a) Capital transactions
Rs.
Depreciation charged 14,000
Tax allowances 16,000
(b) Interest payable
On 1st April 2016 the company issued Rs. 25,000 of 8% convertible loan stock. Interest is paid
in arrears on 30th September and 30th March. Assume that tax relief on interest expense is
only given when the interest is paid.
(c) Interest receivable
On 1st April Shep purchased debentures having a nominal value of Rs. 4,000. Interest at 15%
pa is receivable on 30th September and 30th March. Assume that interest income is not taxed
until the cash is actually received.
(d) Provision for warranty
In preparing the financial statements for the year to 31st December 2016, Shep has
recognised a provision for warranty payments in the amount of Rs. 1,200. This has been
correctly recognised in accordance with IAS 37 and the amount has been expensed. Assume
that tax relief on the warranty cost is only given when the expense is paid.
(e) Fine
During the period Shep has paid a fine of Rs. 6,000. The fine is not tax deductible.
(f) Further information
The accounting profit before tax for the year was Rs. 125,000.
Tax is chargeable at a rate of 30%.

Required
(a) Calculate the corporate income tax liability for the year ended 31st December 2016.
(b) Calculate the deferred tax balance that is required in the statement of financial position as at
31st December 2016.
(c) Prepare a note showing the movement on the deferred tax account and thus calculate the
deferred tax charge for the year ended 31st December 2016
(d) Prepare the statement of profit or loss note which shows the compilation of the tax expense
for the year ended 31st December 2016.
(e) Prepare a note to reconcile the product of the accounting profit and the tax rate to the tax
expense for year ended 31st December 2016.

8.4 SHEP (III)


Continuing from the previous year. The following information is relevant for the year ended 31st
December 2017.
(a) Interest payable/Interest receivable
Shep still has Rs. 25,000 of 8% convertible loan stack in issue and still retains its holding in
the debentures purchased in 2004.
(b) Provision for warranty
During the year Shep had paid out Rs. 500 in warranty claims and provided for a further Rs.
2,000.

© Emile Woolf International 53 The Institute of Chartered Accountants of Pakistan


Financial accounting and reporting II

(d) Development costs


During 2017 Shep has capitalised development expenditure of Rs. 17,800 in accordance with
the provisions of IAS 38. Assume that tax relief on this expenditure is taken in full in the period
in which it is incurred.

(e) Further information


Rs.
Profit before taxation 175,000
Depreciation charged 18,500
Tax allowable depreciation 24,700

(f) Entertainment
Shep paid for a large office party during 2017 to celebrate a successful first two years of the
business. This cost Rs. 20,000. Assume that this expenditure is not tax deductible.
Tax is chargeable at a rate of 30%.

Required
(a) Calculate the corporate income tax liability for the year ended 31st December 2017.
(b) Calculate the deferred tax balance that is required in the statement of financial position as at
31st December 2017.
(c) Prepare a note showing the movement on the deferred tax account and thus calculate the
deferred tax charge for the year ended 31st December 2017
(d) Prepare the statement of profit or loss note which shows the compilation of the tax expense
for the year ended 31st December 2017.
(e) Prepare a note to reconcile the product of the accounting profit and the tax rate to the tax
expense for year ended 31st December 2017.

8.5 SHEP (IV)


Using the information provided in “Shep III” and assume that Shep is subject to a higher tax rate of
34% in 2017.

Required
(a) Calculate the corporate income tax liability for the year ended 31st December 2017.
(b) Calculate the deferred tax balance that is required in the statement of financial position as at
31st December 2017.
(c) Prepare a note showing the movement on the deferred tax account and thus calculate the
deferred tax charge for the year ended 31st December 2017
(d) Prepare the statement of profit or loss note which shows the compilation of the tax expense
for the year ended 31st December 2017.
(e) Prepare a note to reconcile the product of the accounting profit and the tax rate to the tax
expense for year ended 31st December 2017.

8.6 WAQAR LIMITED


Waqar Limited has provided you the following information for determining its tax and deferred tax
expense for the year 2014 and 2015:
(i) During the year ended December 31, 2015, the company’s accounting profit before tax
amounted to Rs. 40 million (2014: Rs. 30 million). The profit includes capital gains amounting
to Rs. 10 million (2014: Rs. 8 million) which are exempt from tax.

© Emile Woolf International 54 The Institute of Chartered Accountants of Pakistan


Questions

(ii) The accounting written down values of the fixed assets, as at December 31, 2013 were as
follows:
Accumulated Written
Cost Depreciation down value
Rs. m Rs. m Rs. m
Machinery 200 25 175
Furniture and fittings 50 10 40
No additions or disposals of fixed assets were made in the years 2014 and 2015.
(iii) Machinery was acquired on January 1, 2013 and is being depreciated on straight- line
basis over its estimated useful life of 8 years. The tax base of machinery as at December 31,
2013 was Rs. 90 million.
(iv) Furniture and fittings are also depreciated on the straight line basis at the rate of 10% per
annum. The tax base of furniture and fittings as at December 31, 2013 was Rs. 40.5 million.
(v) Normal rate of tax depreciation on both types of assets is 10% on written down value.
(vi) The tax rates for 2013, 2014 and 2015 were 35%, 35% and 30% respectively.
Required
For each year:
(a) Calculate the corporate income tax liability for the year.
(b) Calculate the deferred tax balance that is required in the statement of financial position as at
the year end.
(c) Prepare a note showing the movement on the deferred tax account and thus calculate the
deferred tax charge for the year.
(d) Prepare the statement of profit or loss note which shows the compilation of the tax expense.
(e) Prepare a note to reconcile the product of the accounting profit and the tax rate to the tax
expense.

8.7 SHAKIR INDUSTRIES


Given below is the statement of profit or loss of Shakir Industries for the year ended December
31, 2015:

2015
Rs. m
Sales 143.00
Cost of goods sold (96.60)
Gross profit 46.40
Operating expenses (28.70)
Operating profit Other income 17.70
Profit before interest and tax 3.40
Financial charges 21.10
Profit before tax (5.30)
15.80
Following information is available:
(i) Operating expenses include an amount of Rs. 0.7 million paid as penalty to SECP on non-
compliance of certain requirements of the Companies Act, 2017.
(ii) During the year, the company made a provision of Rs. 2.4 million for gratuity. The actual
payment on account of gratuity to outgoing members was Rs. 1.6 million.

© Emile Woolf International 55 The Institute of Chartered Accountants of Pakistan


Financial accounting and reporting II

(iii) Lease payments made during the year amounted to Rs. 0.65 million which include
financial charges of Rs. 0.15 million. As at December 31, 2015, obligations against assets
subject to finance lease stood at Rs. 1.2 million. The movement in assets held under
finance lease is as follows:

Rs. m
Opening balance – 01/01/2015 2.50
Depreciation for the year (0.7)
Closing balance – 31/12/2015 1.80
(iv) The details of owned fixed assets are as follows:

Accounting Tax
Rs. m Rs. m
Opening balance – 01/01/2015 12.50 10.20
Purchased during the year 5.30
5.3 5.30
5.3
Depreciation for the year (1.10)
(1.1) (1.65)
(1.65)
Closing balance – 31/12/2015 5.30
16.70 5.30
13.85
(1.10) (1.65)
(v) Capital work-in-progress as on December 31, 2015 include financial charges of Rs. 2.3
million which have been capitalised in accordance with IAS-23 “Borrowing Costs”. However,
the entire financial charges are admissible, under the Income Tax Ordinance, 2001.
(vi) Deferred tax liability and provision for gratuity as at January 1, 2015 was Rs. 0.55 million
and Rs. 0.7 million respectively.
(vii) Applicable income tax rate is 35%.

Required
Based on the available information, compute the current and deferred tax expenses for the year
ended December 31, 2015.

8.8 MARS LIMITED


Mars Limited (ML) is engaged in the manufacturing of chemicals. On July 1, 2014 it obtained a
motor vehicle on lease from a bank. Details of the lease agreement are as follows:
(i) Cost of motor vehicle is Rs. 1,600,000.
(ii) Instalments of Rs. 480,000 are to be paid annually in advance.
(iii) The lease term and useful life is 4 years and 5 years respectively.
(iv) The interest rate implicit in the lease is 13.701%.
ML follows a policy of depreciating the motor vehicles over their useful life, on the straight-line
method. However, the tax department allows only the lease payments as a deduction from taxable
profits.
The tax rate applicable to the company is 30%. ML’s accounting profit before tax for the year
ended June 30, 2015 is Rs. 4,900,000.
There are no temporary differences other than those evident from the information provided
above.

Required
(a) Prepare journal entries in the books of Mars Limited for the year ended June 30, 2015 to
record the above transactions including tax and deferred tax.
(b) Prepare a note to the financial statements related to disclosure of finance lease liability, in
accordance with the requirements of IFRS.
(Ignore comparative figures.)

© Emile Woolf International 56 The Institute of Chartered Accountants of Pakistan


Questions

8.9 BILAL ENGINEERING LIMITED


Bilal Engineering Limited earned profit before tax amounting to Rs. 50 million during the year
ended December 31, 2015. The accountant of the company has submitted draft accounts to the
Finance Manager along with the following information which he believes could be useful in
determining the amount of taxation:
(i) Accounting deprecation for the year is Rs. 10 million which includes Rs. 1 million charged
on the difference between cost and revalued amount.
(ii) A motor vehicle costing Rs. 1 million was taken on lease in 2014. Related clauses of the
lease agreement are as under:
 Annual instalment of Rs. 0.3 million is payable annually in advance.
 The lease term and useful life is 4 years and 5 years respectively.
 The interest rate implicit in the lease is 13.701% per annum.
 Accounting depreciation on the leased vehicle is included in the depreciation referred
to in para (i) above.
(iii) Tax depreciation on the assets owned by the company is Rs. 7 million.
(iv) Research and development expenses of Rs. 15 million were incurred in 2013 and are being
amortised over a period of 15 years. For tax purposes research and development expenses
are allowed to be written off in 10 years. However, 10% of these expenses were not verifiable
and have not been claimed.
(v) Expenses amounting to Rs. 0.25 million were disallowed in 2012. Out of these Rs. 0.15
million were allowed in appeal, during the current year. The company had initially expected
that the full amount would be allowed but has decided not to file a further appeal.
(vi) The applicable tax rate is 35%.

Required
(a) Prepare journal entries in respect of taxation, for the year ended December 31, 2015.
(b) Prepare a reconciliation to explain the relationship between tax expense and accounting profit
as is required to be disclosed under IAS 12 Income Taxes.

8.10 GALAXY INTERNATIONAL


The following information relates to Galaxy International (GI), a listed company, which was
incorporated on January 1, 2014.
(i) The (loss) / profit before taxation for the years ended December 31, 2014 and 2015 amounted
to (Rs. 1.75 million) and Rs. 23.5 million respectively.
(ii) The details of accounting and tax depreciation on fixed assets is as follows:
2015 2014
Rs. m Rs. m
Accounting depreciation 15 15
Tax depreciation 6 45

(iii) In 2014, GI accrued certain expenses amounting to Rs. 2 million which were disallowed by
the tax authorities. However, these expenses are expected to be allowed on the basis of
payment in 2015.
(iv) GI earned interest on Special Investment Bonds amounting to Rs. 1.0 million and Rs. 1.25
million in the years 2014 and 2015 respectively. This income is exempt from tax.
(v) GI operates an unfunded gratuity scheme. The provision during the years 2014 and 2015
amounted to Rs. 1.7 million and Rs. 2.2 million respectively. No payment has so far been
made on account of gratuity.
(vi) The applicable tax rate is 35%.

© Emile Woolf International 57 The Institute of Chartered Accountants of Pakistan


Financial accounting and reporting II

Required
Prepare a note on taxation for inclusion in the company’s financial statements for the year ended
December 31, 2015 giving appropriate disclosures relating to current and deferred tax expenses
including a reconciliation to explain the relationship between tax expense and accounting profit.

8.11 APRICOT LIMITED


The following information relates to Apricot Limited (AL), a listed company, for the financial year
ended 31 December 2015:
(i) The profit before tax for the year amounted to Rs. 60 million (2014: Rs. 45 million).
(ii) The accounting and tax written down value of fixed assets as on 31 December 2014 was Rs.
95 million and Rs. 90 million respectively. Accounting depreciation for the year is Rs. 10
million (2014: Rs. 9 million) whereas tax depreciation for the year is Rs. 8 million (2014: Rs. 7
million).
(iii) During the year, AL sold a machine for Rs. 3 million and recognised a profit of Rs. 0.5 million.
The tax written down value of the machine as on 31 December 2014 was Rs. 2 million.
There were no other additions/disposals of fixed assets in 2014 and 2015.
(iv) AL earned capital gain of Rs. 6 million (2014:Nil) on sale of shares of a listed company. This
income is exempt from tax.
(v) Bad debt expenses recognised during the year was Rs. 5 million (2014: Rs. 7 million).
(vi) Bad debts written off during the year amounted to Rs. 3 million (2014: Rs. 4 million).
(vii) Deferred tax liability and provision for bad debts as on 31 December 2011 was Rs.
18.90 million and Rs. 9 million respectively.
(viii) The company’s assessed brought forward losses up to 31 December 2011 amounted to
Rs. 19.25 million.
(ix) Applicable tax rate is 35%.
Required
Prepare a note on taxation for inclusion in AL’s financial statements for the year ended 31
December 2015 giving appropriate disclosures relating to current and deferred tax expenses
including comparative figures for 2014 and a reconciliation to explain the relationship between 2015
tax expense and 2015 accounting profit.

8.12 ROSE LIMITED


Rose Limited (RL) is finalizing its financial statements for the year ended 31 December 2017. In this
respect, the following information has been gathered:
(i) Applicable tax rate is 30% except stated otherwise.
(ii) During the year RL incurred advertising cost of Rs. 15 million.
This cost is to be allowed as tax deduction over 5 years from 2017 to 2021.
(iii) Trade and other payables amounted to Rs. 40 million as on 31 December 2017 which
include unearned commission of Rs. 10 million.
Commission is taxable when it is earned by the company. Tax base of remaining trade and
other payables is Rs. 25 million.
(iv) Other receivables amounted to Rs. 17 million as on 31 December 2017 which include
dividend receivable of Rs. 8 million.
Dividend income was taxable on receipt basis at 20% in 2017. However, with effect from 1
January 2018, dividend received is exempt from tax. Tax base of remaining other
receivables is Rs. 6 million.
(v) On 1 April 2017, RL invested Rs. 40 million in a fixed deposit account for one year at 10%
per annum. Interest will be received on maturity.

© Emile Woolf International 58 The Institute of Chartered Accountants of Pakistan


Questions

Interest was taxable on receipt basis at 10% in 2017. However, with effect from 1 January
2018, interest received is taxable at 15%.
(vi) On 1 January 2016, a machine was acquired on lease for a period of 4 years at annual
lease rental of Rs. 28 million, payable in advance. Interest rate implicit in the lease is 10%.
Under the tax laws, all lease related payments are allowed in the year of payment.
(vii) Details of fixed assets are as follows:
 On 1 January 2017 RL acquired a plant at a cost of Rs. 250 million. It has been
depreciated on straight line basis over a useful life of six years. RL is also obliged to
incur decommissioning cost of Rs. 50 million at the end of useful life of the plant.
Applicable discount rate is 8%.
 On 1 July 2017 RL sold one of its four buildings for Rs. 60 million. These buildings were
acquired on 1 January 2013 at a cost of Rs. 100 million each having useful life of 30
years.
The dismantling costs will be allowed for tax purposes when paid. Tax depreciation rate for all
owned fixed assets is 10% on reducing balance method. Further, full year’s tax depreciation is
allowed in year of purchase while no depreciation is allowed in year of disposal.

Required:
Compute the deferred tax liability/asset to be recognised in RL’s statement of financial position as on
31 December 2017.

8.13 FLOOR AND TILES LIMITED


Following are the relevant extracts from the financial statements of Floor & Tiles Limited (FTL) for
the year ended 31 December 2015:

Rs. in million
Profit before tax 80
Provision for gratuity for the year 12
Bad debts expense for the year 10
Capital gain (exempt from tax) 5

The following information is also available:


(i) Opening balances of deferred tax liability, provision for bad debts and provision for gratuity
were Rs. 5.28 million, Rs. 2 million and Rs. 13 million respectively.
(ii) The cost and other details related to buildings (owned) included in property, plant and
equipment are as follows:

Rs. in million
Opening balance (purchased on 1 January 2013) 350
Cost of a building sold on 30 April 2015 (for Rs. 35 million) 30
Purchased on 1 July 2015 40

(iii) Accounting depreciation on buildings is calculated @ 5% per annum on straight line basis
whereas tax depreciation is calculated @ 10% on reducing balance method. Accounting
depreciation of all other owned assets included in property, plant and equipment is same as
tax depreciation.
(iv) On 1 January 2015, a machine costing Rs. 120 million was acquired on finance lease.
Some of the relevant information is as follows:
 The lease term as well as the useful life is 5 years.
 Annual lease rentals amounting to Rs. 30 million are payable in advance.

© Emile Woolf International 59 The Institute of Chartered Accountants of Pakistan


Financial accounting and reporting II

 The interest rate implicit in the lease is 12.59%.


 This machine would be depreciated over its useful life on straight line method.
(v) On 1 June 2015, an amount of Rs. 1 million was paid as penalty to the provincial
government due to non-compliance of environmental laws.
(vi) The amount of gratuity paid to outgoing members was Rs. 10 million.
(vii) During the year, entertainment expenses and repair expenses amounting to Rs. 6 million
and Rs. 8 million respectively, pertaining to year ended 31 December 2013 were
disallowed. FTL has decided to file appeal only against the decision regarding repair
expenses.
(viii) Applicable tax rate is 32%.

Required:
Prepare a note on taxation (expense) for inclusion in FTL’s financial statements for the year ended
31 December 2015 giving appropriate disclosures relating to current and deferred tax expenses
including a reconciliation to explain the relationship between tax expense and accounting profit.

8.14 HADI LIMITED


The following trial balance pertains to Hadi Limited (HL) for the year ended 31 December 2016:

Debit Credit
Description
------- Rs. in ‘000 -------
Capital work-in-progress 145,000
Plant and machinery – at cost 305,000
Trade receivables 61,400
Stock-in-trade 79,600
Cash and bank 33,444
Cost of sales 78,664
Administrative expenses 37,636
Ordinary share capital (Rs. 10 each) 241,000
Retained earnings 69,050
Accumulated depreciation – Plant and machinery 53,250
Trade payables 60,912
10% long term loan 75,000
Provision for warranty 10,000
Provision for bad debts 5,000
Deferred tax liability 25,125
Sales 201,407
740,744 740,744

While finalizing the financial statements of HL from the above trial balance, the following issues have
been noted:
(ii) No depreciation has been charged in the current year. Depreciation is provided at 10% per
annum using the straight line method.
(iii) A machine which was purchased on 1 January 2015 for Rs. 25 million was traded-in, on 1 July
2016 for a new and more sophisticated machine. The disposal was not recorded and the new
machine was capitalized at Rs. 15 million being the net amount paid to supplier. The trade-in
allowance amounted to Rs. 20 million.

© Emile Woolf International 60 The Institute of Chartered Accountants of Pakistan


Questions

(iv) Taxation authorities allow initial and normal depreciation at 25% and 15% respectively using
reducing balance method. No tax depreciation is allowed in the year of disposal. The tax written
down value of the plant and machinery as on 1 January 2016 was Rs. 153 million.
(v) HL maintains a provision for doubtful debts at 6% of trade receivables. On 1 February 2017, a
customer owing Rs. 10 million at year-end was declared bankrupt. HL estimates that 20% of
the amount would be received on liquidation.
(vi) The long term loan of Rs. 75 million was obtained on 1 January 2016, to finance the capital
work-in-progress. HL capitalizes the finance cost on such loan in accordance with IAS-23
‘Borrowing cost’. However, the financial charges are admissible as an expense, under the tax
laws.
(vii) HL sells goods with a 1-year warranty and it is estimated that warranty expenses are 3% of
annual sales. Actual payments during the year, against warranty claims of the products sold
during current and previous years were Rs. 2.5 million and Rs. 8 million respectively. These
have been debited to administrative expenses.
(viii) On 1 January 2016, HL started research and development work for a new product. On 1 May
2016, the recognition criteria for capitalization of internally generated asset was met. The
product was launched on 1 November 2016.
HL incurred Rs. 20 million from commencement of research and development work till
launching of the product and charged it to cost of goods sold. It is estimated that the useful life
of this new product will be 20 years. It may be assumed that all costs accrued evenly over the
period.
On 31 December 2016, the recoverable amount of the development expenditure was Rs. 10
million. For tax purposes, research and development costs are allowed to be amortized over
10 years.
(ix) Applicable tax rate is 30%.

Required:
(a) Prepare statement of comprehensive income for the year ended 31 December 2016 in
accordance with the requirements of International Financial Reporting Standards.
(b) Compute the current and deferred tax expenses for the year ended 31 December
2016.

© Emile Woolf International 61 The Institute of Chartered Accountants of Pakistan


Financial accounting and reporting II

CHAPTER 9: IAS 21: FOREIGN CURRENCY TRANSACTIONS


9.1 DND LIMITED
DND Limited is a listed company, having its operations within Pakistan. During the year ended December
31, 2016, the company contracted to purchase plants and machineries from a US Company. The terms
and conditions thereof , are given below:
(i) Total cost of contract = US$ 100,000.
(ii) Payment to be made in accordance with the following schedule:
Payment Dates Amount Payable
On signing the contract July 01, 2016 US$ 20,000
On shipment* September 30, 2016 US$ 50,000
After installation and test run January 31, 2017 US$ 30,000
*(risk and rewards of ownership are transferred on shipment)
The contract went through in accordance with the schedule and the company made all the payments on
time. The following exchange rates are available:

Dates Exchange Rates


July 1, 2016 US$ 1 = Rs. 60.50
September 30, 2016 US$ 1 = Rs. 61.00
December 31, 2016 US$ 1 = Rs. 61.20
January 31, 2017 US$ 1 = Rs. 61.50

Required
Prepare journals to show how the above contract should be accounted for under IAS 21.

9.2 ORLANDO
Orlando is an entity whose functional currency is the US dollar. It prepares its financial statements to
30 June each year. The following transactions take place on 21 May Year 4 when the spot exchange
rate was $1 = €0.8.
Goods were sold to Koln, a customer in Germany, for €96,000.
A specialised piece of machinery was bought from Frankfurt, a German supplier. The invoice for the
machinery is for €1,000,000.
The company receives €96,000 from Koln on 12 June Year 4.
At 31 June Year 4 it still owns the machinery purchased from Frankfurt. No depreciation has been
charged on the asset for the current period to 30 June Year 4.
The liability for the machine is settled on 31 July Year 4.
Relevant $/€ exchange rates are:
12 June Year 4 $1 = €0.9
30 June Year 4 $1 = €0.7
31 July Year 4 $1 = €0.8

Required
Show the effect on profit or loss of these transactions for:
(a) the year to 30 June Year 4
(b) the year to 30 June Year 5

© Emile Woolf International 62 The Institute of Chartered Accountants of Pakistan


Questions

9.3 OMEGA LIMITED


Omega Limited (OL) is incorporated and listed in Pakistan. On 1 May 2012, it acquired 20,000 ordinary
shares (2% shareholding) in Al-Wadi Limited (AWL), a Dubai based company at a cost of AED 240,000
which was equivalent to Rs. 6,000,000. The face value of the shares is AED 10 each. OL intends to
hold the shares to avail benefits of regular dividends and capital gains.
On 1 June 2013, AWL was acquired by Hilal Limited (HL), which issued three shares in HL in exchange
for every four shares held in AWL.
Other relevant information is as under:
AWL HL
Final dividend received on 31 March 2013:
Cash 15% -
Bonus shares 10% -
Final cash dividend received on 10 April 2014 - 20%
Fair value per share as at: 31 December 2012 AED 13.00 -
1 June 2013 AED 14.00 AED 18.00
31 December 2013 - AED 19.50
Exchange rates on various dates were as follows:

31-Dec-2012 31-Mar-2013 1-Jun-2013 31-Dec-2013 10-Apr-2014

1 AED Rs. 25.00 Rs. 26.50 Rs. 28.00 Rs. 28.70 Rs. 28.20

Required
Determine the amounts (duly classified under appropriate heads) that would be included in OL’s
statement of comprehensive income for the year ended 31 December 2013 in respect of the above
investment.

9.4 KANGAROO LIMITED


Kangaroo Limited (KL), a Pakistan based company, is preparing its financial statements for the year
ended 31 December 2017. Following transactions were carried out during the year.
(i) Foreign currency transactions:
KL purchased an investment property in United States for USD 2.6 million. 10% advance
payment was made on 1 May 2017 and 70% payment was made on 1 July 2017 on transfer
of title and possession of the property. The remaining amount was paid on 1 August 2017.
On 1 September 2017, KL rented out this property at annual rent of USD 0.24 million for one
year and received full amount in advance on the same date.
KL uses fair value model for its investment property. On 31 December 2017, an independent
valuer determined that fair value of the property was USD 2.5 million.
Following spot exchange rates are available:

Date 1-May-2017 1-Jul-2017 1-Aug-2017 1-Sept-2017 31-Dec-2017

USD 1 Rs. 100 Rs. 105 Rs. 108 Rs. 110 Rs. 116

Following average exchange rates are also available:

Period 2017 Jul to Dec 2017 Sep to Dec 2017

USD 1 Rs. 105 Rs. 111 Rs. 113

© Emile Woolf International 63 The Institute of Chartered Accountants of Pakistan


Financial accounting and reporting II

(ii) Equity investments:


On 1 May 2017 KL acquired following equity investments:
Purchase price Transaction cost Total
-------------------- Rs. in million -- ------------------
Investment A 100 2 102
Investment B 150 3 153
Investment A was designated as measured at fair value through profit or loss whereas investment B
was irrevocably elected at initial recognition as measured at fair value through other comprehensive
income.
In October 2017, KL earned dividend of Rs. 12 million and Rs. 9 million on investment A and B
respectively.
20% of investment A and 30% of investment B were sold for Rs. 23 million and Rs. 50 million
respectively in November 2017. Transaction cost was paid at 2%.
As on 31 December 2017, fair values of the remaining investments are given below:
Fair value Transaction cost on disposal Net amount
------------- Rs. in million -------------
Investment A 105 2.1 102.9
Investment B 130 2.6 127.4
Required:
Prepare the extracts relevant to the above transactions from KL’s statements of financial position
and comprehensive income for the year ended 31 December 2017, in accordance with the IFRSs.
(Comparative figures and notes to the financial statements are not required)

9.5 MZA LIMITED


MZA Limited a dollar based entity, was involved in the following transactions in foreign currencies
during the year ended December 31, 2018.
a) MZA Limited bought equipment for 130,000 Dinars on March 04, 20X3 and paid for on August
25, 2018 in Dollars.
b) On February 27 2018 MZA Limited sold goods which had cost $46,000 for $68,000 to a
company whose currency was Krams. The proceeds were received on May 25, 2018.
c) On September 02, 2018 MZA Limited sold goods which cost $17,000 for $ 24,000 to a company
whose currency was Sarils. The amount was outstanding at December 31, 20X3 but the
proceeds were received in sarils on February 07, 20X4 when the exchange rate was S
2.306=$1, the directors of MZA Limited approved the final accounts on March 28, 20X4.
d) MZA Limited borrowed 426,000 rolands on May 25, 20X3 and is repayable in two years time.
Exchange rates relevant to the above transactions to $1 are given below: -
Date Rolans Dinars Krams Saril
27-Feb-18 - - 7.000 -
4-Mar-18 - 0.650 - -
25-May-18 1.500 - 6.700 -
25-Aug-18 - 0.500 - -
2-Sep-18 - - - 2.224
31-Dec-18 1.800 0.540 7.500 2.250

Required:
For each of the above transactions calculate the gross profit or loss and foreign currency gain or
loss which would be included in the company’s financial statements for the ended December 31,
2018 as required by IAS-21.

© Emile Woolf International 64 The Institute of Chartered Accountants of Pakistan


Questions

CHAPTER 10 – IAS 38: INTANGIBLE ASSETS


10.1 FAZAL
The following information relates to the financial statements of Fazal for the year to 31 March 2015.
The IT division has begun a training course for all managers in a new programming language at a
cost of Rs. 200,000. The consultants running the training course have quantified the present value of
the training benefits over the next two years to be Rs. 400,000. The project cost has been included
in the statement of financial position as a current asset. The accounting policy note identifies that the
costs will be written off over the next two years to match the benefits.

Required
Explain the correct accounting treatment for the above (with calculations if appropriate).

10.2 HENRY
During 2015 Henry has the following research and development projects in progress.
Project A was completed at the end of 2014. Development expenditure brought forward at the
beginning of 2015 was Rs. 412,500 on this project. Savings in production costs arising from this
project are first expected to arise in 2015. In 2015 savings are expected to be Rs. 100,000, followed
by savings of Rs. 300,000 in 2016 and Rs. 200,000 in 2017.
Project B commenced on 1 April 2015. Costs incurred during the year were Rs. 56,000. In addition
to these costs a machine was purchased on 1 April 2015 for Rs. 30,000 for use on the project. This
machine has a useful life of five years. At the end of 2015 there were still some uncertainties
surrounding the completion of the project.
Project C had been started in 2014. In 2014 the costs relating to this project of Rs. 36,700 had been
written off, as at the end of 2014 there were still some uncertainties surrounding the completion of
the project. Those uncertainties have now been resolved and a further Rs. 45,000 costs incurred
during the year.
Required
Show how the above would appear in the financial statements (including notes to the financial
statements) of Henry as of 31 December 2015.

10.3 TOBY
Toby entered into the following transactions during the year ended 31 December 2015. The directors
of Toby wish to capitalise all assets wherever possible.
(1) On 1 January Toby acquired the net assets of George for Rs. 105,000. The assets acquired
had the following book and fair values.
Book value Fair value
Rs. Rs.
Goodwill 5,000 5,000
Patents 15,000 20,000
Non-current assets 40,000 50,000
Other sundry net assets 30,000 25,000
––––––– ––––––––
90,000 100,000
═════ ═════
The patent expires at the end of 2022. The goodwill arising from the above had a recoverable
value at the end of 2015 of Rs. 7,000.
(2) On 1 April Toby acquired a brand from a competitor for Rs. 50,000. The directors of Toby
have assessed the useful life of the brand as five years.

© Emile Woolf International 65 The Institute of Chartered Accountants of Pakistan


Financial accounting and reporting II

(3) During the year Toby spent Rs. 40,000 on developing a new brand name. The development
was completed on 30 June. The useful life of this brand has been assessed as eight years.
(4) The directors of Toby believe that there is total goodwill of Rs. 2 million within Toby and that
this has an indefinite useful life.

Required
Prepare the note to the financial statements for intangible assets as at 31 December 2015.

10.4 BROOKLYN
Brooklyn is a bio-technology company performing research for pharmaceutical companies. The
finance director has contacted your financial consulting company to arrange a meeting to discuss
issues relevant to the preparation of the financial statements for the year to 30th June 2015. Your
initial telephone conversation has provided the necessary background information.
1 On 1st August 2014 Brooklyn began investigating a new bio-process. On 1st September 2015,
the new process was widely supported by the scientific community and the feasibility project
was approved. A grant was then obtained relating to future work. Several pharmaceutical
companies have expressed an interest in buying the ‘know how’ when the project completes
in June 2016. The nominal ledger account set up for the project shows that the expenditure
incurred between 1st August 2014 and 30th June 2015 was Rs. 300,000 per month.
2 In August 2015, an employee lodged a legal claim against the company for damage to his
health as a result of working for the company for the two years through to 31 st March 2014
when he had to retire due to ill health. He has argued that his health deteriorated as a result of
the stress from his position in the organisation. Brooklyn has denied the claim and has
appointed an employment lawyer to assist with contesting the case. The lawyer has advised
that there is a 25% chance that the claim will be rejected, 50% chance that the damages will
be Rs. 600,000 and 25% chance of Rs. 1 million. The company has an insurance policy that
will pay 10% of any damages to the company. The lawyer has said that the case could take
until 30th June 2018 to resolve. The present value of the estimated damages discounted at 8%
is Rs. 476,280 and Rs. 793,800 respectively.
3 Brooklyn owns several buildings, which include an administrative office in the centre of
London. The company has revalued these on a regular basis every five years and the next
valuation is due on 30th June 2017. Property prices have increased since the last review and
particularly for the London premises. The cost of engaging a professionally qualified valuer is
very expensive and so to reduce costs the finance director is proposing that the property
manager, who is a professionally qualified valuer, should value the London property and that
the increase in value should be included in the financial statements. The finance director is of
the opinion that the property prices may fall next year.

Required
Prepare notes for your meeting with the finance director which explain and justify the accounting
treatment of these issues, preparing calculations where appropriate and identifying matters on which
your require further information.

10.5 ZOUQ INC


Zouq Inc. is a multinational company. As part of its vision to expand its business in South Asia, it
purchased a 90% share of a locally incorporated company, Momin Limited. Following are the brief
details of the acquisition:

Date of acquisition January 1, 2014


Total paid up capital of Momin Limited (Rs. 10 each) 500,000,000
Purchase price per share Rs. 30
Net assets of Momin Limited (as per 2013 audited financial statements) 650,000,000
Fair value of net assets (other than intangible assets) of Momin Limited 1,100,000,000

© Emile Woolf International 66 The Institute of Chartered Accountants of Pakistan


Questions

Momin Limited has an established line of products under the brand name of “Badar”. On behalf
of Zouq Inc., a firm of specialists has valued the brand name at Rs. 100 million with an estimated
useful life of 10 years at January 1, 2014. It is expected that the benefits will be spread equally over
the brand’s useful life.
An impairment test of goodwill and brand was carried out on December 31, 2014 which
indicated an impairment of Rs. 50 million in the value of goodwill.
An impairment test carried out on December 31, 2015 indicated a decrease of Rs. 13.5 million in
the carrying value of the brand.

Required:
(a) What are the requirements of International Accounting Standards relating to amortization of
intangible assets having finite life?
(b) Prepare the ledger accounts for goodwill and the brand, showing initial recognition and all
subsequent adjustments.

10.6 RAISIN INTERNATIONAL


(a) Discuss the criteria that should be used while recognizing intangible assets arising from
research and development work.
(b) Raisin International (RI) is planning to expand its line of products. The related information for
the year ended 31 December 2015 is as follows:
(i) Research and development of a new product commenced on 1 January 2015. On 1
October 2015, the recognition criteria for capitalization of an internally generated
intangible asset were met. It is estimated that the product would have a useful life
of 7 years. Details of expenditures incurred are as follows:

Rs. m

Research work 4.50

Development work 9.00

Training of production staff 0.50

Cost of trial run 0.80

Total costs 14.80

(ii) The right to manufacture a well-established product under a patent for a period of five
years was purchased on 1 March 2015 for Rs. 17 million. The patent has an expected
remaining useful life of 10 years. RI has the option to renew the patent for a further
period of five years for a sum of Rs. 12 million.
(iii) RI has acquired a brand at a cost of Rs. 2 million. The cost was incurred in the month of
June 2015. The life of the brand is expected to be 10 years. Currently, there is no
active market for this brand. However, RI is planning to launch an aggressive marketing
campaign in February 2016.
(iv) In September 2014, RI developed a new production process and capitalised it as an
intangible asset at Rs. 7 million. The new process is expected to have an indefinite
useful life. During 2015, RI incurred further development expenditure of Rs. 3 million
on the new process which meets the recognition criteria for capitalization of an
intangible asset.

Required
In the light of International Financial Reporting Standards, explain how each of the above transaction
should be accounted for in the financial statements of Raisin International for the year ended 31
December 2015.

© Emile Woolf International 67 The Institute of Chartered Accountants of Pakistan


Financial accounting and reporting II

10.7 INTERNATIONAL ASSOCIATES LIMITED


Following information pertains to International Associates Limited (IAL):
i. Intangible assets as at 30 June 2015 were as follows:

Brands Software License

Useful life (years) 10 5 Indefinite

--------- Rs. in million ---------

Cost 200 80 15

Accumulated amortization / impairment 40 48 -

ii. Details of expenses incurred on a project to improve IAL’s existing production process are as
under:

Period Rs. in million

Up to June 2015 20

July 2015 – March 2016 45

Expenses were incurred evenly during the above period. On 30 September 2015, it was
established that the project is commercially viable. The new process became operational with
effect from 1 April 2016 and it is anticipated that it will generate cost savings of Rs. 10 million
per annum for a period of 10 years.
iii. On 1 August 2015, IAL entered into an agreement to acquire an ERP software which would
replace its existing accounting software. The new software became operational on 1 April 2016.
IAL incurred following expenditure in respect of the ERP software:

Description Rs. in million

Purchase price (including 15% sales tax) 115

Training of staff 2

Consultancy charges for implementation of ERP 5

ERP software has an estimated useful life of 15 years. However, IAL expects to use it for a
period of 10 years. The existing accounting software has become redundant and is of no use for
the company.
iv. During the year ended 30 June 2016, IAL spent Rs. 10 million on development of a new brand.
Useful life of the brand is estimated as ten years.
v. The license appearing in IAL’s books was issued by the government for an indefinite period.
However, on 1 January 2016 the Government introduced a legislation under which the existing
license would have to be renewed after ten years.
vi. IAL uses cost model to value its intangible assets and amortises them on straight-line basis.

Required
Prepare a note on ‘intangible assets’ for inclusion in IAL’s financial statements for the year ended
30 June 2016 in accordance with International Financial Reporting Standards.

© Emile Woolf International 68 The Institute of Chartered Accountants of Pakistan


Questions

10.8 TULIP LIMITED


Draft financial statements of Tulip Limited (TL) for the year ended 31 December 2017 show the
following amounts:
Rs. in million

Total assets 2,700

Total liabilities 1,620

Net profit for the year 398

While reviewing the draft financial statements, following matters have been noted:
TL commenced development of a new product on 1 January 2017. Following directly attributable
costs have been incurred upto the launching date of 1 October 2017 and have been capitalized as
intangible asset:

Rs. in million

Staff salary 30

Equipment (having useful life of 5 years) 360

Consumables 90

Consultant fee 212

Total 692

The recognition criteria for capitalization of internally generated intangible assets was met on 1
March 2017. All costs have been incurred evenly during the period except equipment which was
purchased specifically for this product on 1 January 2017.
TL estimated that useful life of this new product will be 10 years. However, TL had not charged any
amortization in 2017.

Required
Determine the revised amounts of total assets and net profit, after incorporating the impact of above
adjustment(s), if any.

10.9 ZEBRA LIMITED


During the year ended 31 December 2017, following transactions were made by Zebra
Limited (ZL):
On 1 April 2017 ZL acquired a licence for operating a TV channel for Rs. 86.3 million out of which
Rs. 50 million was paid immediately. The balance amount is payable on 1 April 2019. A mega social
media and print media campaign was launched to promote the channel at a cost of Rs. 10 million.
The transmission of the channel started on 1 August 2017.
The license is valid for 5 years but is renewable every five years at a cost of Rs. 35 million.
Since the renewal cost is significant, the management intends to renew the license only once and
sell it at the end of 8 years.
In the absence of any active market, the management has estimated that residual value of the
license would be Rs. 15 million and Rs. 20 million at the end of 5 years and 8 years respectively.
Applicable discount rate is 10% p.a.

Required:
Discuss how these transactions should be recorded in ZL’s books of accounts for the year ended 31
December 2017.

© Emile Woolf International 69 The Institute of Chartered Accountants of Pakistan


Financial accounting and reporting II

10.10 SUNSHINE LIMITED


On 1 July 2016, Sunshine Limited (SL) acquired four licenses namely A, B, C and D for a period of
ten years. The following information is available in respect of these licenses:
(i)

A B C D
Cost of license (Rs. in million) 200 230 90 60

Expected period of cash generation 12 years indefinite 6 years 12 years


from acquisition date
Active market value at 30 June 170 300 65 No active market
2017 (Rs. in million)
Renewal cost (Rs. in million) 65 85 2 1

(ii) The renewal would allow SL to use the licenses for another five years.
(iii) SL uses the revaluation model for subsequent measurement of its intangible assets.
(iv) An independent valuer has estimated the value of license ‘D’ at Rs. 130 million.

Required:
Determine the amounts that should be recognised in respect of the licenses in the statement of
financial position and statement of profit or loss for the year ended 30 June 2017.

© Emile Woolf International 70 The Institute of Chartered Accountants of Pakistan


Questions

CHAPTER 11: IAS 41: Agriculture


11.1 GUJRANWALA FOODS LIMITED
Gujranwala Foods Limited has the following information in its records for the year ended 31 January
2017.

Chickpea Apricot Dates Onion


Rs.’000 Rs.’000 Rs.’000 Rs.’000
Sales 200 300 750 250
Purchases 75 150 300 80
Subsidies 40 80 60 60
Own consumption 45 40 75 20
Opening Inventory 40 40 70 -
Closing Inventory 60 110 300 60

The farm expenses for the period 1 February 2016 to 30 April 2017 are as follows:

Rs.
Casual labour 20,000
Regular workers 30,000
Land preparation and clearing costs 80,000
Hire of tractors 60,000

The farm’s non- current assets for the year ended 31 January 2017 were as follows:
Rs.
Farm’s irrigation at cost 800,000
Farm’s implement and equipment 400,000

Additional Information:
(i) Farm’s irrigation costs are to be written off over 10 years
(ii) Farm’s implement and equipment were purchased on 31 April 2016 and these are to be
depreciated at 20% per annum.

Required
(a) Prepare Gujranwala Foods Limited’s gross output and statement of profit or loss for the year
ended 31 January 2017.
(b) In accordance with IAS 41 on Agriculture, you are required to define the following terms:
(i) Biological assets
(ii) Biological transformation
(iii) Harvest

© Emile Woolf International 71 The Institute of Chartered Accountants of Pakistan


Financial accounting and reporting II

11.2 HELIOS GROUP


Helios Ltd is an agricultural production company.
Helios Ltd acquired 70% of the ordinary shares of Sol Ltd, an agricultural based company for Rs.
600 million on 1 January 2015, when the reserves of Sol Ltd were Rs. 300 million. At the date of
acquisition the fair value of the non-controlling interest in Sol Ltd was Rs. 160 million.
The fair values of the net assets of Sol Ltd are the same as their carrying values with the exception
of a plot of agricultural land. This land was carried by Sol Ltd at its cost of Rs. 300 million. It was
estimated at a fair value of Rs. 360 million.
Statement of financial position of Helios Ltd and Sol Ltd as at 31December 2016
Helios Ltd Sol Ltd
Rs.’000 Rs.’000
Assets
Non-current assets:
Property, plant and equipment 600,000 450,000
Investments 800,000 -

Current assets:
Inventories 160,000 150,000
Trade & other receivables 120,000 280,000
Cash and cash equivalent 20,000 50,000
Total assets 1,700,000 930,000
Equity and liabilities:
Ordinary share capital 160,000 120,000
Share premium 40,000 20,000
Reserves 590,000 500,000
Non-current liabilities:
Loan notes 600,000 170,000
Current liabilities
Trade & other payables 310,000 120,000
Total equity & liabilities 1,700,000 930,000
Additional information:
Immediately after acquisition, the following agricultural products were procured and included in
property, plant and equipment and inventories of Sol Ltd as at 31 December 2016:

(i) Included in property, plant and equipment of Sol Ltd are: Rs.’000
Dairy livestock – immature 40,000
Dairy livestock – mature 50,000
(ii) Included in inventories of Sol Ltd is:
Cotton plants 20,000
Required
(a) Prepare the consolidated statement of financial position for Helios Ltd group as at 31
December 2016 as expected for an agricultural business.
(b) State how to measure agricultural products harvested by an entity in line with the
requirements of IAS 41 on Agriculture.

© Emile Woolf International 72 The Institute of Chartered Accountants of Pakistan


Questions

11.3 THE DAIRY COMPANY


(i) Briefly explain the term “biological asset” and state when a biological asset is recognised in
the financial statements under the International Financial Reporting Standards.
(ii) The Dairy Company (TDC) owns three farms and has a stock of 3,200 cows. During the year
ended 30 June 2015, 300 animals were born, all of which survived and were still owned by
TDC at year-end. Of those, 225 are infants whereas 75 are nine month old having market
values of Rs. 26,000 and Rs. 53,000 per animal respectively. The incidental costs are 2% of
the transaction price.

Required:
In accordance with the requirements of the International Financial Reporting Standards, discuss how
the gain in respect of the new born cows should be recognized in TDC’s financial statements for
the year ended 30 June 2015. (Show all necessary computations)

11.4 FATIMA LIMITED


Fatima Limited on adoption of IAS 41 has reclassified certain assets as biological assets. The total
value of the group’s forest assets is Rs.2,000 million comprising:

Rs. in million
Freestanding trees 2,500
Land under trees 500
Roads in forests 400
3,400

Required:
Show how the forests would be classified in the financial statements.

11.5 ZOHA LIMITED


Zoha Limited has these balances in its financial records:
Value of biological asset at cost 31/12/2018 600
Fair valuation surplus on initial recognition at fair value 31/12/2018 700
Change in fair value to 12/31/2019 due to growth and price fluctuations 100
Decrease in fair value due to harvest 90

Required:
Show how these values would be incorporated into the statement of financial position and statement
of comprehensive income at December 31, 2019.

11.6 MISHALL LIMITED


Mishall Limited a public limited company, Dairy, produces milk on its farms. It produces 30% of the
country’s milk that is consumed. Dairy owns 450 farms and has a stock of 210,000 cows and
105,000 heifers. The farms produce 8 million kilograms of milk a year, and the average inventory
held is 150,000 kilograms of milk. However, the company is currently holding stocks of 500,000
kilograms of milk in powder form. At December 31, 2018, the herds are:
 210,000 cows (3 years old), all purchased on or before January 1, 2018
 75,000 heifers, average age 1.5 years, purchased on June 1, 2018
 30,000 heifers, average age 2 years, purchased on January 1, 2018

© Emile Woolf International 73 The Institute of Chartered Accountants of Pakistan


Financial accounting and reporting II

No animals were born or sold in the year.

The unit values less estimated point-of-sale costs were Rs.


1-year-old animal at December 31, 2018: 32
2-year-old animal at December 31, 2018: 45
1.5-year-old animal at December 31, 2018: 36
3-year-old animal at December 31, 2018: 50
1-year-old animal at January 1, 2018 and June 1, 2018: 30
2-year-old animal at January 1, 2018: 40

The company has had problems during the year: Contaminated milk was sold to customers. As a
result, milk consumption has gone down. The government has decided to compensate farmers for
potential loss in revenue from the sale of milk. This fact was published in the national press on
November 1, 2018. Dairy received an official letter on December 10, 2018, stating that Rs.5 million
would be paid to it on March 2, 2019. The company’s business is spread over different parts of the
country. The only region affected by the contamination was Lahore, where the government curtailed
milk production in the region. The cattle were unaffected by the contamination and were healthy.
The company estimates that the future discounted cash flow income from the cattle in the Lahore
region amounted to Rs.4 million, after taking into account the government restriction order. The
company feels that it cannot measure the fair value of the cows in the region because of the
problems created by the contamination. There are 60,000 cows and 20,000 heifers in the region. All
these animals had been purchased on January 1, 2018. A rival company had offered Dairy Rs.3
million for these animals after point-of-sale costs and further offered Rs.6 million for the farms
themselves in that region. Dairy has no intention of selling the farms at present. The company has
been applying IAS 41 since January 1, 2018.

Required:
Advise the directors on how the biological assets and produce of Dairy should be accounted for
under IAS 41, discussing the implications for the financial statements.

© Emile Woolf International 74 The Institute of Chartered Accountants of Pakistan


Questions

CHAPTER 12: IFRS 9: FINANCIAL INSTRUMENTS: RECOGNITION AND


MEASUREMENT
12.1 AJI PANCA LTD
On 1 January Year 1 Aji Panca Ltd has the following capital and reserves.

Equity Rs.
Share capital (Rs. 1 ordinary shares) 1,000,000
Share premium 200,000
Retained earnings 5,670,300
––––––––––
6,870,300
––––––––––

During Year 1 the following transactions took place.


1 January An issue of Rs. 100,000 8% Rs. 1 redeemable preference shares at a premium
of 60%. Issue costs are Rs. 2,237. Redemption is at 100% premium on 31
December Year 5. The effective rate of interest is 9.5%.
31 March An issue of 300,000 ordinary shares at a price of Rs. 1.30 per share. Issue costs,
net of tax benefit, were Rs. 20,000
30 June A 1 for 4 bonus issue of ordinary shares.
Profit for the year, before accounting for the above, was Rs. 508,500. The dividends on the
redeemable preference shares have been charged to retained earnings.

Required
Set out capital and reserves and liabilities resulting from the above on 31 December Year 1.

12.2 PASSILA LTD


On 1 July 2016, Passila Ltd, issued 20,000 8% debentures at Rs. 97.50. The security is redeemable
in five years’ time. The interest on the debentures is payable bi-annually on 30 June and 31
December.
On 31 December 2016, the Company’s year-end date, the debentures were quoted on the Karachi
Stock Exchange for Rs. 96.00. The company accountant has suggested each of the following as
possible valuation basis for reporting the debentures liability on the statement of financial position as
at 31 December 2016:
(i) Face value of the debentures.
(ii) Face value of the debenture plus interest payment for five years.
(III) Market value on the statement of financial position as at the year end.

Required
(a) Determine the face value of the debentures and the proceeds accruing to the company.
(b) Determine the amount and explain the nature of the differences between the face value and
the market value of the debentures on 1 July, 2016.
(c) Distinguish between nominal and effective rate of interest.
(d) Determine the nominal interest payable on the debentures for the year ended 31 December
2016.
(e) State arguments for or against each of the suggested alternatives for reporting the debentures
liability on the statement of financial position as at 31 December 2016.

© Emile Woolf International 75 The Institute of Chartered Accountants of Pakistan


Financial accounting and reporting II

12.3 RASHID INDUSTRIES LIMITED


On 15 October 2016, Rashid Industries Limited (RIL) made the following investments:

Percentage of *Cost of investment


Name of Investees No. of shares
shareholding acquired (Rs. in million)

Karim Limited (KL) 155,000 4% 20

Bashir Limited (BL) 135,000 2% 65

* including transaction cost


Investment in KL was made with no intention to sell the shares while investment in BL was made
with the intention to sell the shares before 31 December 2016.
The board of directors in its meeting held on 30 November 2016 decided that since the future
prospects of BL are quite attractive, its shares should be held till 30 June 2018. The market rate on
30 November 2016 was Rs. 621.
On 31 December 2016, RIL decided to record an impairment loss of Rs. 5 million against investment
in KL. The market price of shares of KL and BL as on 31 December 2016 was Rs. 80 and Rs. 600
respectively.
RIL’s broker normally charges transaction costs of 0.2%.

Required:
Explain the accounting treatment of above transactions in accordance with International Financial
Reporting Standards.

© Emile Woolf International 76 The Institute of Chartered Accountants of Pakistan


Questions

CHAPTER 13 – IFRS 16: LEASES


13.1 X LTD
X Ltd is considering acquiring a machine. It has two options; cash purchase at a cost of
Rs.11,420,000 or a lease.
The terms of the lease are as follows:
(i) The lease period is for four years from 1 January 2016 with an annual rental of Rs.4,000,000
payable on 31 December each year.
(ii) The lessee is required to pay all repairs, maintenance and other incidental costs.
(iii) The interest rate implicit in the lease is 15% p.a.
Note:
Estimated useful economic life span of the machine is four years.

Required
(a) Prepare a schedule of the allocation of the finance charges in the books of X Limited for the
entire lease period.
(b) Prepare an extract of the Statement of Financial Position of X Limited as on 31 December
2016.

13.2 PROGRESS LTD


Progress Ltd. acquired a machine from Fine Rentals Ltd. on January 3, 2016 under a lease
agreement extending over three years.
The agreement required them to make an initial deposit of Rs.1,280,000 to be followed by three
annual payments of Rs.800,000 on 31 December each year starting from 2016.
The cash price of the machinery was Rs.3,200,000 and Fine Rentals Ltd. added 12% interest which
was duly communicated to Progress Ltd.
The annuity method is used to allocate interest.

Required
(a) Compute the interest element and the capital portion of the annual repayments; and
(b) Show the journal entries that will record the transaction resulting from the lease agreement.

13.3 MIRACLE TEXTILE LIMITED


On 1 July 2014, Miracle Textile Limited (MTL) acquired a machine on lease, from a bank.
Details of the lease are as follows:
(i) Cost of machine is Rs.20 million.
(ii) The lease term and useful life is 4 years and 10 years respectively.
(iii) Instalment of Rs.5.80 million is to be paid annually in advance on 1 July.
(iv) The interest rate implicit in the lease is 15.725879%.
(v) At the end of lease term, MTL has an option to purchase the machine on payment of
Rs.2 million. The fair value of the machine at the end of lease term is expected to be
Rs.3 million.
MTL depreciates the machine on the straight line method to a nil residual value.

Required
Prepare relevant extracts of the statement of financial position and related notes to the financial
statements for the year ended 30 June 2016 along with comparative figures. Ignore taxation.

© Emile Woolf International 77 The Institute of Chartered Accountants of Pakistan


Financial accounting and reporting II

13.4 ACACIA LTD


On 1 April 2015 Acacia Ltd entered into the following lease agreement.
Plant with a fair value of Rs.275,000 was leased under an agreement which requires Acacia Ltd to
make annual payments of Rs.78,250 on 1 April each year, commencing on 1 April 2015, for four
years. After the four years Acacia Ltd has the option to continue to lease the plant at a nominal rent
for a further three years and is likely to do so as the asset has an estimated useful life of six years.
The present value of the lease payments is Rs.272,850. Acacia Ltd is responsible for insuring and
maintaining the plant during the period of the lease.
Acacia Ltd allocates finance charges on an actuarial basis. The interest rate implicit in the lease is
10%.

Required
Prepare all relevant extracts from Acacia Ltd's financial statements for the year ended 31 March
2016.

13.5 SHOAIB LEASING LIMITED


Shoaib Leasing Limited (the lessor) has entered into a three year agreement with Sarfaraz Limited
(the lessee) to lease a machine with an expected useful life of 4 years. The cost of machine is Rs.
2,100,000.
The following information relating to lease transaction is available:
(i) Date of commencement of lease is July 1, 2016.
(ii) The lease contains a purchase bargain option at Rs.100,000. At the end of the lease term, the
value of the machine will be Rs.300,000.
(iii) Lease instalments of Rs.860,000 are payable annually, in arrears, on June 30.
(iv) The implicit interest rate is 12.9972%.

Required
(a) Prepare the journal entries for the years ending June 30, 2017, 2018 and 2019 in the books
of lessor. Ignore tax.
(b) Produce extracts from the statement of financial position including relevant notes as at
June 30, 2017 to show how the transactions carried out in 2017 would be reflected in the
financial statements of the lessor. (Disclosure of accounting policy is not required.)

13.6 SHALIMAR INDUSTRIES LIMITED


Shalimar Industries (SI) is engaged in the manufacturing of tractors. The tractors are sold both on
cash and finance lease basis. The cash selling price and cost of each tractor is Rs. 2.0 million and
Rs. 1.6 million respectively.
On 1 January 2015, SI sold ten tractors to Caravan Transport (CT) on lease. The terms of the lease
and related information are as follows:
(i) The lease period is 4 years, whereas useful life of each tractor is 5 years.
(ii) The total unguaranteed residual value at the end of lease term is Rs. 1 million.
(iii) Lease rentals amounting to Rs. 6,375,454 per annum are payable in arrears.
The rate implicit in the lease is 12%.

Required
In accordance with the requirements of International Financial Reporting Standards, prepare:
(a) Journal entries in the books of SI to record the transactions for the year ended 31 December
2015.
(b) A note for inclusion in SI’s financial statements, for the year ended 31 December 2015.

© Emile Woolf International 78 The Institute of Chartered Accountants of Pakistan


Questions

13.7 GUAVA LEASING LIMITED (GLL)


Guava Leasing Limited (GLL), had leased a machinery to Honeyberry Limited (HL) on 1 July 2017
on the following terms:
(i) The non-cancellable lease period is 3.5 years. Each semi-annual lease instalment of Rs. 48
million is receivable in arrears.
(ii) The lease contains an option to extend the lease term by 1.5 years. Each semi annual lease
instalment in the extended period will be of Rs. 15 million, receivable in arrears. It is
reasonably certain that HL will exercise this option.
(iii) The rate implicit in the lease is 10% per annum.
(iv) The useful life of machinery is 6 years.
(v) The unguaranteed residual value at the end of lease term is estimated at Rs. 20 million.
GLL incurred a direct cost of Rs. 10 million and general overheads of Rs. 0.5 million to
complete the transaction.

Required:
Prepare note(s) for inclusion in GLL’s financial statements, for the year ended 30 June 2018.

© Emile Woolf International 79 The Institute of Chartered Accountants of Pakistan


Financial accounting and reporting II

CHAPTER 14 – OTHER AREAS OF IFRSs (IFRS 8, IAS 10, IAS 37)


14.1 BADAR
The following information relates to the financial statements of Badar for the year to 31 March 2015.
The mining division of Badar has a 3 year operating licence from an overseas government. This
allows it to mine and extract copper from a particular site. When the licence began on 1 April 2014,
Badar started to build on the site. The cost of the construction was Rs. 500,000.
The overseas country has no particular environmental decommissioning laws. In its past financial
statements Badar has given information about the company’s environmental policy and has provided
examples to demonstrate that it is a responsible company that believes in restoring mining sites at
the end of the extraction period. The cost of removing the construction at the end of the three years
is estimated to be Rs. 100,000.
The cost of the site currently shown in the trial balance is Rs. 500,000. The company has a cost of
borrowing of 10%.

Required
Explain the correct accounting treatment for the above (with calculations if appropriate).

14.2 GEORGINA
Georgina Company is preparing its financial statements for the year ended 30 September 2015. The
following matters are all outstanding at the year end.
(1) Georgina is facing litigation for damages from a customer for the supply of faulty goods on 1
September 2015. The claim, which is for Rs. 500,000, was received on 15 October 2015.
Georgina’s legal advisors consider that Georgina is liable and that it is likely that this claim will
succeed. On 25 October 2015 Georgina sent a counter-claim to its suppliers for Rs. 400,000.
Georgina’s legal advisors are unsure whether or not this claim will succeed.
(2) Georgina’s sales director, who was dismissed on 15 September, has lodged a claim for Rs.
100,000 for unfair dismissal. Georgina’s legal advisors believe that there is no case to answer
and therefore think it is unlikely that this claim will succeed.
(3) Although Georgina has no legal obligation to do so, it has habitually operated a policy of
allowing customers to return goods within 28 days, even where those goods are not faulty.
Georgina estimates that such returns usually amount to 1% of sales. Sales in September
2015 were Rs. 400,000. By the end of October 2015, prior to the drafting of the financial
statements, goods sold in September for Rs. 3,500 had been returned.
(4) On 15 September 2015 Georgina announced in the press that it is to close one of its divisions
in January 2016. A detailed closure plan is in place and the costs of closure are reliably
estimated at Rs. 300,000, including Rs. 50,000 for staff relocation.

Required
State, with reasons, how the above should be treated in Georgina’s financial statements for the year
ended 30 September 2015.

14.3 EARLEY INC


Earley Inc is finalising its accounts for the year ended 31 December 2014. The following events have
arisen since the year end and the financial director has asked you to comment on the final accounts.
(a) At 31 December 2014 trade receivables included a figure of Rs. 250,000 in respect of
Nedengy Inc. On 8 March 2015, when the current debt was Rs. 200,000, Nedengy Inc went
into receivership. Recent correspondence with the receiver indicates that no dividend will be
paid to unsecured creditors.
(b) On 15 March 2015 Earley Inc sold its former head office building, Whitley Wood, for Rs. 2.7
million. At the year end the building was unoccupied and carried at a value of Rs. 3.1 million.

© Emile Woolf International 80 The Institute of Chartered Accountants of Pakistan


Questions

(c) Inventories at the year-end included Rs. 650,000 of a new electric tricycle, the Opasney. In
January 2015 the European Union declared the tricycle to be unsafe and prohibited it from
sale. An alternative market, in Bongolia, is being investigated, although the current price is
expected to be cost less 30%.
(d) Stingy Inc, a subsidiary in Outer Sonning, was nationalised in February 2015. The Outer
Sonning authorities have refused to pay any compensation. The net assets of Stingy Inc have
been valued at Rs. 200,000 at the year end.
(e) Freak floods caused Rs. 150,000 damage to the Southcote branch of Earley Inc in January
2015. The branch was fully insured.
(f) On 1 April 2015 Earley Inc announced a 1 for 1 rights issue aiming to raise Rs. 15 million.

Required
Explain how you would respond to the matters listed above.

14.4 ACCOUNTING TREATMENT


You have been asked to advise on the appropriate accounting treatment for the following situations
arising in the books of various companies. The year end in each case can be taken as 31 December
2015 and you should assume that the amounts involved are material in each case.
(a) At the year end there was a debit balance in the books of a company for Rs. 15,000,
representing an estimate of the amount receivable from an insurance company for an
accident claim. In February 2016, before the directors had agreed the final draft of the
published accounts, correspondence with lawyers indicated that Rs. 18,600 might be payable
on certain conditions.
(b) A company has an item of equipment which cost Rs. 400,000 in 2012 and was expected to
last for ten years. At the beginning of the 2015 financial year the book value was Rs. 280,000.
It is now thought that the company will soon cease to make the product for which the
equipment was specifically purchased. Its recoverable amount is only Rs. 80,000 at 31
December 2015.
(c) On 30 November a company entered into a legal action defending a claim for supplying faulty
machinery. The company’s solicitors advise that there is a 20% probability that the claim will
succeed. The amount of the claim is Rs. 500,000.
(d) An item has been produced at a manufacturing cost of Rs. 1,800 against a customer’s order
at an agreed price of Rs. 2,300. The item was in inventory at the year-end awaiting delivery
instructions. In January 2016 the customer was declared bankrupt and the most reasonable
course of action seems to be to make a modification to the unit, costing approximately Rs.
300, which is expected to make it marketable with other customers at a price of about Rs.
1,900.
(e) At 31 December a company has a total potential liability of Rs. 1,000,400 for warranty work on
contracts. Past experience shows that 10% of these costs are likely to be incurred, that 30%
may be incurred but that the remaining 60% is highly unlikely to be incurred.

Required
For each of the above situations outline the accounting treatment you would recommend and give
the reasoning of principles involved. The accounting treatment should refer to entries in the books
and/or the year-end financial statements as appropriate.

14.5 J-MART LIMITED


(a) Explain the terms “adjusting events” and “non-adjusting events” and give three examples of
each.
(b) J-Mart Limited, a chain of departmental stores has distributed its operations into four
Divisions i.e. Food, Furniture, Clothing and Household Appliances. The following information
has been extracted from the records:
(i) The company allows the dissatisfied customers to return the goods within 30 days. It is
estimated that 5% of the sales made in June 2015 will be refunded in July 2015.

© Emile Woolf International 81 The Institute of Chartered Accountants of Pakistan


Financial accounting and reporting II

(ii) On June 2, 2015, three employees were seriously injured as a result of a fire at the
company’s warehouse. They have lodged claims seeking damages of Rs. 2.0 million
from the company. The company’s lawyers have advised that it is probable that the
court may award compensation of Rs. 400,000.
(iii) Under a new legislation, the company is required to fit smoke detectors at all the stores
by December 31, 2015. The company has not yet installed the smoke detectors.
(iv) On June 20, 2015, the board of directors decided to close down the Household
Appliances Division. However, the decision was made public after June 30, 2015.
(v) The company has a large warehouse in Lahore which was acquired under a three-
year rent agreement signed on April 1, 2014. The agreement is non- cancellable
and the company cannot sub-let the warehouse. However, due to operational
difficulties, the company shifted the warehouse to a new location.
(vi) A 15% cash dividend was declared on July 5, 2015.

Required
Describe how each of the above issue should be dealt with in the financial statements for the year
ended June 30, 2010. Support your point of view in the light of relevant International Accounting
Standards.

14.6 AKBER CHEMICALS LIMITED


Akber Chemicals Limited is engaged in the business of manufacture and sale of different type
of chemicals. The following transactions have not yet been incorporated in the financial statements
for the year ended June 30, 2015:
(a) On June 15, 2015, one of its tankers carrying chemicals fell into a canal, thus polluting the
water. The company has never faced such a situation before. The company has neither any
legal obligation to clean the canal nor does it have any published environmental policy. In a
meeting held on July 26, 2015 the Board of Directors decided to clean the canal, which is
estimated to cost Rs. 5.5 million.
(b) During the second week of July 2015, a significant decline in the demand for company’s
products was observed which also led to a decrease in net realizable value of finished goods.
It was estimated that goods costing Rs. 25 million as at June 30, 2015 would only fetch
Rs. 23 million.
(c) On June 21, 2015, a customer lodged a claim of Rs. 2 million with the company as a
consignment dispatched on June 1, 2015 was not according to the agreed specifications. The
company’s inspection team found that this defect arose because of inferior quality of raw
materials supplied by the vendor. On June 28, 2015, the company lodged a claim for
damages of Rs. 5.0 million, with its vendor, which include reimbursement of the cost of raw
materials. The company anticipates that it will have to pay compensation to its customer and
would be able to recover 50% of the amount claimed from the vendor.

Required
Discuss how Akber Chemicals (Pvt.) Limited would deal with the above situations in its financial
statements for the year ended June 30, 2015. Explain your point of view with reference to the
guidance contained in the International Financial Reporting Standards.

14.7 QALLAT INDUSTRIES LIMITED


The following information pertains to Qallat Industries Limited (QIL) for its financial year ended
June 30, 2015:
(i) QIL sells all its products on one-year warranty which covers all types of defects. Previous
history indicates that 2% of the products contain major defects whereas 10% have minor
defects. It is estimated that if major defects were detected in all the products sold, repair
cost of Rs. 150 million would result. If minor defects were detected in all products sold,
repair cost of Rs. 70 million would result. Total sales for the year are amounted to Rs. 830
million.

© Emile Woolf International 82 The Institute of Chartered Accountants of Pakistan


Questions

(ii) QIL has two large warehouses, A and B. These were acquired under non-cancellable
lease agreements. Details are as follows:

Warehouse A Warehouse B
Effective date of agreement July 1, 2010 January 1, 2013
Lease period 10 years 8 years
Rental amount per month Rs. 450,000 Rs. 300,000

On account of serious operating difficulties, QIL vacated both the warehouses on January 1,
2015 and moved to a warehouse situated close to its factory. On the same day QIL sub-
let Warehouse A at Rs. 250,000 per month for the remaining lease period. Warehouse B
was sub-let on March 1, 2015 for Rs. 350,000 per month for the remaining lease period.
(iii) On July 18, 2015, QIL was sued by an employee claiming damages for Rs. 6 million on
account of an injury caused to him due to alleged violation of safety regulations on the part of
the company, while he was working on the machine on June 15, 2015. Before filing the suit,
he contacted the management on June 29, 2015 and asked for compensation of Rs. 4 million
which was turned down by the management. The lawyer of the company anticipates that the
court may award compensation ranging between Rs. 1.5 million to Rs. 3 million. However, in
his view the most probable amount is Rs. 2 million.
(iv) On November 1, 2014 a new law was introduced requiring all factories to install specialised
safety equipment within four months. The Equipment costing Rs. 5.0 million was ordered on
December 15, 2014 against 100% advance payment but the supplier delayed installation to
July 31, 2015. On August 5, 2015 the company received a notice from the authorities levying
a penalty of Rs. 0.4 million i.e. Rs. 0.1 million for each month during which the violation
continued. QIL has lodged a claim for recovery of the penalty from the supplier of the
equipment.

Required
Describe how each of the above issues should be dealt with in the financial statements for the year
ended June 30, 2015. Support your answer in the light of relevant International Accounting
Standards and quantify the effect where possible.

14.8 SKYLINE LIMITED


The following information pertains to Skyline Limited (SL) for the financial year ended December 31,
2015:
(i) A customer who owed Rs. 1 million was declared bankrupt after his warehouse was destroyed
by fire on February 10, 2016. It is expected that the customer would be able to recover 50% of
the loss from the insurance company.
(ii) An employee of SL forged the signatures of directors and made cash withdrawals of Rs. 7.5
million from the bank. Of these, Rs. 1.5 million were withdrawn before December 31, 2015.
Investigations revealed that an employee of the bank was also involved and therefore, under
a settlement arrangement, the bank paid 60% of the amount to SL on January 27, 2016.
(iii) SL has filed a claim against one of its vendors for supplying defective goods. SL’s legal
consultant is confident that damages of Rs. 1 million would be paid to SL. The supplier has
already reimbursed the actual cost of the defective goods.
(iv) A suit for infringement of patents, seeking damages of Rs. 2 million, was filed by a third party.
SL’s legal consultant is of the opinion that an unfavourable outcome is most likely. On the
basis of past experience he has advised that there is 60% probability that the amount of
damages would be Rs. 1 million and 40% likelihood that the amount would be Rs. 1.5 million.

Required
Advise SL about the amount of provision that should be incorporated and the disclosures that are
required to be made in the financial statements for the year ended December 31, 2015.

© Emile Woolf International 83 The Institute of Chartered Accountants of Pakistan


Financial accounting and reporting II

14.9 WALNUT LIMITED


Walnut Limited (WL) is engaged in the business of import and distribution of electronic appliances.
The following events took place subsequent to the reporting period i.e. 31 December 2015:
(i) On 15 January 2016, one of WL’s competitors announced launching of an upgraded version
of DVD players. WL’s inventories include a large stock of existing version of DVD players
which are valued at Rs. 15 million. Because of the introduction of the upgraded version, the
net realizable value of the existing version in WL’s inventory at 31 December 2015 has
reduced to Rs. 12.5 million.
(ii) On 20 December 2015, the board of directors decided to close down the division which
imports and sells mobile sets. This decision was made public on 29 December 2015.
However, the business was actually closed on 29 February 2016.
Net costs incurred in connection with the closure of this division were as follows:
Rs. m
Redundancy costs 1.50
Staff training 0.15
Operating loss from 1 July 2015 to closure of division 0.80
Less: Profit on sale of remaining mobile sets (0.50)
1.95

(iii) On 16 January 2016, LED TV sets valuing Rs. 3 million were stolen from a warehouse.
These sets were included in WL’s inventory as at 31 December 2015.
(iv) WL owns 9,000 shares of a listed company whose price as on 31 December 2015 was Rs. 22
per share. During February 2016, the share price declined significantly after the government
announced a new legislation which would adversely affect the company’s operations. No
provision in this regard has been made in the draft financial statements.
(v) On 31 January 2016, a customer announced voluntary liquidation. On 31 December 2015, this
customer owed Rs. 1.5 million.
(vi) On 15 February 2016, WL announced final dividend for the year ended 31 December 2015
comprising 20% cash dividend and 10% bonus shares, for its ordinary shareholders.

Required
Describe how each of the above transactions should be accounted for in the financial statements
of Walnut Limited for the year ended 31 December 2015. Support your answer in the light of
relevant International Financial Reporting Standards.

14.10 ATTOCK TECHNOLOGIES LIMITED


Attock Technologies Limited (ATL) manufactures five hi-tech products, each on a different plant. It is
in the process of preparing its financial statements for the year ended June 30, 2015. As the CFO of
the company, the following matters are under your consideration:
(i) Inventory carried at Rs. 25 million on June 30, 2015 was sold for Rs. 15 million after it had
been damaged in a flood, in July 2015.
(ii) On July 5, 2015 one of ATL’s corporate customers declared bankruptcy. The liquidator
announced on August 25, 2015 that 20% of the debt would be paid on liquidation.
(iii) A new product introduced by a competitor on August 1, 2015 had caused a significant
decline in the market demand of one of ATL’s major products. As a result, ATL is considering
a reduction in price and a cut in production.
(iv) On August 18, 2015 the government announced a retrospective increase in the tax rate
applicable to the company.
(v) The directors of ATL declared a dividend of Rs. 3 per share on August 28, 2015.

Required
State how the above events should be treated in ATL’s financial statements for the year ended
June 30, 2015. You may assume that all the above events are material to the company.

© Emile Woolf International 84 The Institute of Chartered Accountants of Pakistan


Questions

14.11 SHAZAD INDUSTRIES LIMITED


Shazad Industries Ltd has recently acquired four large subsidiaries. These subsidiaries manufacture
products which are of different lines from those of the parent company. The parent company
manufactures plastics and related products whereas the subsidiaries manufacture the following:

Product Location

Subsidiary 1 Textiles Karachi

Subsidiary 2 Car products Lahore

Subsidiary 3 Fashion garments Peshawar

Subsidiary 4 Furniture items Multan

The directors have purchased these subsidiaries in order to diversify their product base but do not
have any knowledge of the information required in the financial statements regarding these
subsidiaries other than the statutory requirements.

Required
(a) Explain to the directors the purpose of segmental reporting of financial information.
(b) Explain to the directors the criteria which should be used to identify the separate reportable
segments. (You should illustrate your answer by reference to the above information)
(c) Critically evaluate IFRS 8, Operating segments, setting out any problems with the standard.

14.12 AZ
For enterprises that are engaged in different businesses with differing risks and opportunities, the
usefulness of financial information concerning these enterprises is greatly enhanced if it is
supplemented by information on individual business segments.

Required
(i) Explain why the information content of financial statements is improved by the inclusion of
segmental data on individual business segments.
(ii) Discuss how IFRS 8 requires that segments be analysed.

14.13 ROWSLEY
Rowsley is a diverse group with many subsidiaries. The group is proud of its reputation as a ‘caring’
organisation and has adopted various ethical policies towards its employees and the wider
community in which it operates. As part of its Annual Report, the group publishes details of its
environmental policies, which include setting performance targets for activities such as recycling,
controlling emissions of noxious substances and limiting use of non-renewable resources.
The finance director is reviewing the accounting treatment of various items prior to finalising the
accounts for the year ended 31 March 20X4. All items are material in the context of the accounts as
a whole. The accounts are due to be approved by the directors on 30 June 20X4.
Closure of factory
On 15 February 20X4, the board of Rowsley decided to close down a large factory in Derbytown.
The board is trying to draw up a plan to manage the effects of the reorganisation, and it is envisaged
that production will be transferred to other factories. The factory will be closed on 31 August 20X4,
but at 31 March 20X4 this decision had not yet been announced to the employees or to any other
interested parties. Costs of the reorganisation have been estimated at Rs. 45 million

© Emile Woolf International 85 The Institute of Chartered Accountants of Pakistan


Financial accounting and reporting II

Relocation of subsidiary
During December 20X3, one of the subsidiary companies moved from Buckington to Sundertown in
order to take advantage of government development grants. Its main premises in Buckington are
held under an operating lease, which runs until 31 March 20X9. Annual rentals under the lease are
Rs. 10 million. The company is unable to cancel the lease, but it has let some of the premises to a
charitable organisation at a nominal rent. The company is attempting to rent the remainder of the
premises at a commercial rent, but the directors have been advised that the chances of achieving
this are less than 50%.
Legal claim
During the year to 31 March 20X4, a customer started legal proceedings against the group, claiming
that one of the food products that it manufactures had caused several members of his family to
become seriously ill. The group’s lawyers have advised that this action will probably not succeed.
Environmental impact of overseas subsidiary
The group has an overseas subsidiary that is involved in mining precious metals. These activities
cause significant damage to the environment, including deforestation. The company expects to
abandon the mine in eight years’ time. The mine is situated in a country where there is no
environmental legislation obliging companies to rectify environmental damage and it is very unlikely
that any such legislation will be enacted within the next eight years. It has been estimated that the
cost of cleaning the site and re-planting the trees will be Rs. 25 million if the re-planting was
successful at the first attempt, but it will probably be necessary to make a further attempt, which will
increase the cost by a further Rs. 5 million.

Required
Explain how each of the items above should be treated in the consolidated financial statements for
the year ended 31 March 20X4

14.14 MULTAN PETROCHEM LTD


Multan Petrochem Limited (MPL) operates in the oil extraction and refining business and is
preparing its draft financial statements for the year ended 31 December 2016. The following
information has been collected for the preparation of the provisions and contingencies notes.
(1) A new site was acquired on 1 January 2015 and is being used as the site for a new oil
refinery. Initial preparation work was undertaken at the site at the start of 2015 and the oil
refinery was completed and ready for use on 31 December 2015. The new refinery was
expected to have a useful life of 25 years. MPL has a well-publicised policy that it will reinstate
any environmental damage caused by its activities. The present value of the estimated cost of
reinstating the environment is Rs. 1,300,000 for damage caused during the initial preparation
work. This amount is based on a discount rate of 8%.
(2) An explosion at one of MPL’s oil extraction plants on 1 July 2016 has led to a number of
personal injury claims being made by employees who were injured during the explosion. Five
claims have been made to date but if these claims are successful, it is likely that a further
three employees who were also injured will make a claim. MPL’s lawyers estimate that it is
probable that the claims will succeed and that the estimated average cost of each payout will
be Rs. 150,000. The lawyers have recommended that MPL settles the claims out of court as
quickly as possible at their estimated amount for all eight employees injured to avoid any
adverse publicity.
An additional two claims have been made by employees for the stress, rather than injury, that
the explosion has caused them. If these claims were to succeed the lawyers have estimated
that the likely payout would be around Rs. 10,000 per employee. However, the lawyers have
stated that they believe it to be unlikely that these employees will win such a case.
MPL made an insurance claim to try to recover the personal injury costs that it is probable that
it will incur. The claim is now in its advanced stages and the insurance company has agreed
to meet the cost of the claims in full. The insurance company will refund MPL once the claims
have been settled.

© Emile Woolf International 86 The Institute of Chartered Accountants of Pakistan


Questions

(3) The future of MPL’s business operations is in doubt following the explosion at the oil
extraction plant. The national press criticised MPL for the way that it handled the problem. To
address this, on 1 October 2016 MPL paid Rs. 12,000 to a risk assessment specialist who has
recommended introducing a new disaster recovery plan at an estimated cost of Rs. 500,000.
(4) MPL entered into an operating lease in the previous period for some office space. However,
the company’s plans changed and the office space was no longer required. At 1 January 2016
a correctly calculated provision had been made for the future outstanding rentals of Rs.
80,000 for the remaining five years. This was based on a discount rate of 8%. The rent paid
during the period was Rs. 15,000. In addition, MPP has signed a sub-lease to rent out the
space for the first six months of next year for total rental income of Rs. 6,000. No other
tenants are expected to be found for the office space.
Required
(a) Prepare the provisions and contingencies notes for inclusion in the financial statements of
MPP for the year ended 31 December 2016.
(b) List the amounts that should be recognised in the statement of profit or loss for the year
ended 31 December 2016.

14.15 VIOLET POWER LIMITED


Violet Power Limited is running a coal based power project in Pakistan. The Company has built its
plant in an area which contains large reserves of coal. The company has signed a 20 years
agreement for sale of power to the Government. The period of the agreement covers a
significant portion of the useful life of the plant. The company is liable to restore the site by
dismantling and removing the plant and associated facilities on the expiry of the agreement.
Following relevant information is available:
(i) The plant commenced its production on July 1, 2015. It is the policy of the company to
measure the related assets using the cost model;
(ii) Initial cost of plant was Rs. 6,570 million including erection, installation and borrowing costs
but does not include any decommissioning cost;
(iii) Residual value of the plant is estimated at Rs. 320 million;
(iv) Initial estimate of amount required for dismantling of plant, at the time of installation of plant
was Rs. 780 million. However, such estimate was reviewed as of June 30, 2016 and was
revised to Rs. 1,021 million;
(v) The Company follows straight line method of depreciation; and
(vi) Real risk-free interest rate prevailing in the market was 8% per annum when initial estimates
of decommissioning costs were made. However, at the end of the year such rate has dropped
to 6% per annum.
Required
Work out the carrying value of plant and decommissioning liability as of June 30, 2016.

14.16 GOHAR LIMITED


Gohar Limited (GL), a listed company, is engaged in chemicals, soda ash, polyester, paints and
pharma businesses. Results of each business segment for the year ended 31 March 2015 are as
follows:
Business Sales Gross Operating Assets Liabilities
segments profit expenses
------------------------- Rs. in million -------------------------
Chemicals 1,790 1,101 63 637 442
Soda Ash 216 117 57 444 355
Polyester 227 48 23 115 94
Paints 247 26 16 127 108
Pharma 252 31 12 132 98

© Emile Woolf International 87 The Institute of Chartered Accountants of Pakistan


Financial accounting and reporting II

Inter-segment sale by Chemicals to Polyester and Soda Ash is Rs. 28 million and Rs. 10 million
respectively at a contribution margin of 30%.
Operating expenses include GL’s head office expenses amounting to Rs. 75 million which have not
been allocated to any segment. Furthermore, assets and liabilities amounting to Rs. 150 million and
Rs. 27 million have not been reported in the assets and liabilities of any segment.

Required:
In accordance with the requirements of International Financial Reporting Standards:
(a) determine the reportable segments of Gohar Limited; and
(b) show how these reportable segments and the necessary reconciliation would be disclosed in
GL’s financial statements for the year ended 31 March 2015.

14.17 JAY LIMITED


(a) Specify the criteria for identification of operating segments, in accordance with the
International Financial Reporting Standards.
(b) Jay Limited is an integrated manufacturing company with five operating segments.
Following information pertains to the year ended 31 March 2012:

Internal External Total Profit / Assets Liabilities


Operating
revenue revenue revenue (loss)
segments
-----------------------Rs. in million-----------------------
A 38 705 743 194 200 130
B - 82 82 (22) 44 40
C - 300 300 81 206 125
D 35 - 35 10 75 60
E 38 90 128 (63) 50 25
Total 111 1,177 1,288 200 575 380

Required:
In respect of each operating segment explain whether it is a reportable segment.

14.18 GLADIATOR LIMITED


Faraz is a chartered accountant and employed as Finance Manager of Gladiator Limited (GL). He
has recently returned after a long medical leave and has been provided with draft financial
statements of GL for the year ended 30 June 2017. Following figures are reflected in the draft
financial statements:

------- Rs. in million -------

Profit before tax 125

Total assets 1,420


Total liabilities 925

While reviewing the financial statements, he noted the following issues:


As at 30 June 2017, dismantling cost relating to a plant has increased from initial estimate of Rs. 30
million to Rs. 40 million. Further, fair value of the plant on that date was assessed at Rs. 112 million
(net of dismantling cost). No accounting entries have been made in respect of increase in
dismantling liability and revaluation of the plant.

© Emile Woolf International 88 The Institute of Chartered Accountants of Pakistan


Questions

The plant had a useful life of 5 years when it was purchased on 1 July 2015. The carrying value of
plant and related revaluation surplus included in the financial statements are Rs. 135.4 million (after
depreciation for the year ended 30 June 2017) and Rs. 3.15 million (after transferring incremental
depreciation for the year ended 30 June 2017) respectively.

Required:
Determine the revised amounts of profit before tax, total assets and total liabilities after incorporating
the impact of above adjustments, if any.

14.19 BRAVO LIMITED


`The financial statements of Bravo Limited (BL) for the year ended 30 September 2013 are
under finalisation and the following matters are under consideration:
BL’s plant was commissioned and became operational on 1 April 2008 at a cost of Rs. 130 million.
At the time of commissioning its useful life and present value of decommissioning liability was
estimated at 20 years and Rs. 19 million respectively.
BL’s discount rate is 10%.
There has been no change in the above estimates till 30 September 2013 except for the
decommissioning liability whose present value as at 1 April 2013 was estimated at Rs. 25 million.

Required:
Compute the related amounts as they would appear in the statements of financial position and
comprehensive income of Bravo Limited for the year ended 30 September 2013 in accordance with
IFRS. (Ignore corresponding figures)

14.20 WASTE MANAGEMENT LIMITED


Waste Management Limited (WML) had installed a plant in 2005 for generation of electricity
from garbage collected by the civic agencies. WML had signed an agreement with the
government for allotment of a plot of land, free of cost, for 10 years. However, W ML has agreed
to restore the site, at the end of the agreement.
Other relevant information is as under:
i. Initial cost of the plant was Rs. 80 million. It is estimated that the site restoration cost would
amount to Rs. 10 million.
ii. It is the policy of the company to measure its plant and machinery using the revaluation
model.
iii. When the plant commenced its operations i.e. on April 1, 2005 the prevailing market based
discount rate was 10%.
iv. On March 31, 2007 the plant was revalued at Rs. 70 million including site restoration cost.
v. On March 31, 2009 prevailing market based discount rate had increased to 12%.
vi. On March 31, 2011 estimate of site restoration cost was revised to Rs. 14 million.
vii. Useful life of the plant is 10 years and WML follows straight line method of depreciation.
viii. Appropriate adjustments have been recorded in the prior years i.e. up to March 31, 2010.

Required:
Prepare accounting entries for the year ended March 31, 2011 based on the above information,
in accordance with International Financial Reporting Standards. (Ignore taxation.)

© Emile Woolf International 89 The Institute of Chartered Accountants of Pakistan


Financial accounting and reporting II

CHAPTER 15 – ETHICAL ISSUES IN FINANCIAL REPORTING


15.1 ETHICAL ISSUES
Waheed is a chartered accountant, recently employed by AA plc as deputy to the finance director,
Arif (also a chartered accountant). AA plc is listed on the Lahore stock exchange.
On Waheed’s first day on the job he met with Arif who said ‘Look, keep it to yourself but I’m having a
second interview next week for a new job. The first thing that I need you to do is to review the
financial statements before the auditors arrive. I qualified a few years ago and am not up to date on
all of the little technicalities in IFRS. You should now these better than me and you’ll know more
about what the auditors might focus on. We must do our best to present the financial statements in
the most favourable light as the bonus paid to employees (including me) depends on profit being
more than 10% bigger than last year’s and remember that you qualify for this too. Keep this in mind
when you carry out the review as we do not really want to find anything. Do well at this and I might
put in a good word for you when I leave as I’m sure you’ll be a great replacement for me.”

Required
Explain the ethical issues inherent in the above conversation and what Waheed should do about
them.

15.2 SINDH INDUSTRIES LTD


Jafar has recently been appointed as financial controller to Sindh Industries Ltd. Until a month ago,
Sindh Industries had a finance director, who resigned suddenly, due to ill health. Since Jafar joined
the company, he has learned that his resignation was related to stress caused by a series of
disagreements with the managing director about the performance of the business. The directors
have not yet appointed a replacement.
It is now March 2016 and you have been asked to finalise the financial statements for the year
ended 31 December 2015. The draft statement of profit or loss extract and statement of financial
position are shown below:
Draft statement of profit or loss for the year ended 31 December 2015

Rs. 000
Profit before tax 2,500

Draft statement of financial position at 31 December 2015

Rs. 000
Property, plant and equipment 12,000
Current assets 3,500
Total assets 15,500

Share capital 2,000


Retained earnings 6,000
Equity 8,000
Non-current liabilities 5,000
Current liabilities 2,500
Total equity and liabilities 15,500
During the year ended 31 December 2015 Sindh Industries entered into the following transactions.
(1) Just before the year end Sindh Industries signed a contract to deliver consultancy services for
a period of 2 years at a fee of Rs. 500,000 per annum. The full amount of this fee has been
paid in advance and is non-refundable.

© Emile Woolf International 90 The Institute of Chartered Accountants of Pakistan


Questions

(2) Sindh Industries has constructed a new factory. The construction has been financed from the
pool of existing borrowings. Land at a cost of Rs. 1.8 million was acquired on 1 February 2015
and construction began on 1 June 2015. Construction was completed on 30 September 2015
at an additional cost of Rs. 2.7 million. Although the factory was usable from that date, full
production did not commence until 1 December 2015. Throughout the year the company’s
average borrowings were as follows:

Amount Annual interest rate

Rs. %

Bank overdraft 1,000,000 9.75

Bank loan 1,750,000 10

Debenture 2,500,000 8

An amount of Rs. 450,000 has been included in property, plant and equipment in respect of
borrowing costs relating to the construction of the factory. The useful life of the factory has
been estimated at 20 years. No depreciation has been charged for the year. The reason for
this is that the factory has only been in use for one month and that the depreciation charge
would be immaterial.
(3) A blast furnace with a carrying amount at 1 January 2015 of Rs. 3.5 million has been
depreciated in the draft financial statements on the basis of a remaining life of 20 years. In
December 2015 the directors carried out a review of the useful lives of various significant
items of plant and machinery, including the blast furnace and came to the conclusion that the
useful life of the furnace was 20 years at 31 December 2015. The reasoning behind this
judgement was that the lining of the furnace had been replaced in the last week of December
20X6 at a cost of Rs. 1.4 million. Provided that the lining is replaced every five years, the life
of the furnace can be extended accordingly. You have found a report, commissioned by the
previous finance director and prepared by a firm of asset valuation specialists, which
assesses the remaining useful life of the main structure of the furnace at 1 January 2015 at 15
years and the lining of the furnace at 5 years. You have also found evidence that the
managing director has seen this report.
Jafar has had a conversation with the managing director who told him, “We need to make the
figures look as good as possible so I hope you’re not going to start being difficult. The
consultancy fee is non-refundable so there’s no reason why we can’t include it in full. I think
we should look at our depreciation policies. We’re writing off our assets over far too short a
period. As you know, we’re planning to go for a stock market listing in the near future and
being prudent and playing safe won’t help us do that. It won’t help your future with this
company either.”
Required
(a) Explain the required IFRS accounting treatment of these issues, preparing relevant
calculations where appropriate.
(b) Prepare a revised draft of the statement of profit or loss extract for the year ended
31 December 2015 and the statement of financial position at that date.
(c) Discuss the ethical issues arising from your review of the draft financial statements and the
actions that you should consider.

15.3 CHARMING LIMITED


Umer Sheikh, ACA is Manager Finance at Charming Limited (CL) and reports to Abid, FCA who is
the Chief Financial Officer of CL. Abid is also a close relative of the major shareholder of CL.
CL is negotiating an important financing arrangement with Union Standard Bank (USB) in order to
expand its business in foreign markets. The rate quoted by USB is comparatively higher than
existing rates being paid by CL.

© Emile Woolf International 91 The Institute of Chartered Accountants of Pakistan


Financial accounting and reporting II

During a meeting with the Executive Vice President (EVP) of USB, where Umer Sheikh was also
present, Abid revealed that his son has applied for a house financing in USB last month but has not
received any response from USB so far. Abid requested EVP to consider his application. EVP
agreed to look into the matter. On conclusion of the meeting, Abid asked Umer Sheikh to prepare a
note for the board of directors proposing the acceptance of the rate offered by USB.
Required
Briefly explain how Abid may be in breach of the fundamental principles of ICAP’s code of ethics.
Also state the potential threats that Umer Sheikh may face in the above circumstances and how he
should respond.

15.4 MEHRAN LIMITED


Usman is a Chartered Accountant and has been working as Finance Director in Mehran Limited
(ML) for the past one year. He reports to the CEO who is also a Chartered Accountant.
Recently, Usman has received a bill issued by an advertising agency which is duly approved for
payment by the Director Marketing. Usman believes that the amounts agreed to be paid under the
contract far exceed the value of services to be provided by the advertising agency and that the
payment would be redirected to obtain a sales contract. He has discussed the matter with CEO who
has advised him to process the payment in ML’s business interest. The CEO also informed Usman
that if the said contract is secured, the management staff will be entitled to a handsome bonus.

Required:
Briefly explain how CEO is in breach of the fundamental principles of ICAP’s code of ethics. Also
state the potential threats which Usman may face under the circumstances, along with available
safeguards (if any).

15.5 MNZ LIMITED


Atif is a chartered accountant and has been working as Manager – Accounts in an unlisted public
company MNZ Limited.
While preparing the financial statements for the year ended 31 December 2016, CFO of MNZ who is
also a chartered accountant informed Atif that the directors are considering to have the company
listed on Pakistan Stock Exchange.
Consequently, CFO wants to show higher profit and has asked Atif to identify areas where book
adjustments can be made. He has also informed that if MNZ is able to list the shares at a price of
Rs. 35 or more, all managerial staff would be given an additional bonus this year.

Required:
Briefly explain how the CFO is in breach of the fundamental principles of ICAP’s code of ethics. Also
state the potential threats that Atif may face under the above circumstances and how he should
respond.

15.6 UNIQUE ENGINEERING LIMITED


Zia is a Chartered Accountant and works as a financial controller in Unique Engineering Limited
(UEL). UEL is currently considering the acquisition of Top Storage Limited (TSL) and Zia is a
member of the team which is currently negotiating the acquisition with the management of TSL.
After becoming aware of the prospective acquisition, Zia purchased 1,000,000 shares of TSL in the
name of his wife and son.

Required
Briefly explain how Zia is in breach of the fundamental principles of ICAP’s code of ethics.
Also explain the potential threats that may be involved in the above situation.

© Emile Woolf International 92 The Institute of Chartered Accountants of Pakistan


Questions

15.7 ALI AND BASHIR


Ali and Bashir are chartered accountants and have been working as Managing Director (MD) and
Chief Financial Officer (CFO) in a listed company. In a recent meeting of the Board, the directors
have decided to expand the business within six months by opening 20 retail outlets. This expansion
would require financing of Rs. 300 million which may be arranged through bank loan.
The following information has been extracted from latest draft financial statements of the company:

Rs. in ‘000

Sales 1,700

Gross profit 545

Tax expense 23

Profit after tax 40

Total assets 2,500

Non-current assets 900

Inventories 850

Trade receivables 600

Share capital 800

Reserves 152

Long term debt @ 9% 750

Following additional information is also available:


 80% of the sales are on credit.
 Opening inventory was Rs. 100 million.
 40% of current liabilities comprise of trade payables.
MD has advised the CFO to arrange the loan from MN Bank. He has also informed that the
President of the bank is his good friend and the loan can be arranged on a fast track basis at a
mark-up of 15% per annum, subject to the following conditions:
 current ratio and quick ratio should be at least 2:1 and 1:1 respectively;
 gearing ratio should not exceed 40%; and
 interest cover should be at least 3.
CFO is not comfortable with this deal as the mark-up offered by the bank is much higher than the
rate on the existing loan and it is difficult for the company to meet the gearing requirements of the
bank. However, MD has asked him to make certain changes in the draft financial statements before
submission to the bank; which according to the CFO are not in accordance with the IFRSs.

Required
Briefly explain how the MD may be in breach of the fundamental principles of ICAP’s code of ethics.
Also state the potential threats that CFO may face under the circumstances, along with available
safeguards (if any).

© Emile Woolf International 93 The Institute of Chartered Accountants of Pakistan


Financial accounting and reporting II

15.8 FARAZ
On receiving the revised financial statements, the CEO called Faraz and briefed him in the following
manner:
“Since the position of the CFO is vacant, I intend to promote you as CFO. GL has been through a
rough year and has some disappointing results but a reasonable profit needs to be reported for the
mutual benefit of all stakeholders. Moreover, the financial statements would also be scrutinized by
the bank to ensure that the loan covenants are met which include maintaining total assets at 1.5
times the total liabilities.
Therefore, I want you to confirm the draft financial statements without making any adjustment for
presentation before the Board and submission to the bank.”

Required:
Briefly explain the potential threats that Faraz may face in the above situation and how he should
respond.

© Emile Woolf International 94 The Institute of Chartered Accountants of Pakistan


Certificate in Accounting and Finance

B
Financial accounting and reporting II

SECTION
Answers
CHAPTER 1 – LEGAL BACKGROUND TO THE PREPARATION OF FINANCIAL
STATEMENTS
There are no questions specific to chapter one because the learning outcomes in this area concern the
preparation of financial statements. The relevant questions have been given in chapter 2 of this question
bank.

© Emile Woolf International 95 The Institute of Chartered Accountants of Pakistan


Financial accounting and reporting II

CHAPTER 2 – IAS 1: PRESENTATION OF FINANCIAL STATEMENTS


2.1 LARRY
Statement of profit or loss
For the year ended 31 December 2015
Rs. in
million
Revenue 3,304
Cost of sales (2,542 + 118 – 127) (2,533)
Gross profit 771
Other income 20
Distribution costs (175)
Administrative expenses (342)
Profit before tax 274
Income tax expense (75)
Profit for the period 199
Statement of financial position
As at 31 December 2015
Assets Rs. in
million
Non-current assets
Property, plant and equipment (2,830 – 918) 1,912
Intangible assets (26 – 5) 21
1,933
Current assets
Inventories 127
Trade and other receivables 189
Cash (89 +2) 91
407
Total assets 2,340
Equity and liabilities
Equity
Share capital 400
Retained earnings (1,562 + 199) 1,761
2,161
Non-current liabilities
Long-term borrowings (18 x 2/3) 12
Current liabilities
Trade and other payables 86
Current portion of long-term borrowing (18 ÷ 3) 6
Current tax payable 75
167
Total equity and liabilities 2,340

© Emile Woolf International 96 The Institute of Chartered Accountants of Pakistan


Answers

2.2 MINGORA IMPORTS LIMITED


Statement of profit or loss for the year ended 31 December 2015
Rs. in
million
Revenue 1,740
Change in inventories of finished goods and work-in-progress (W3) 40
Staff costs (W3) (620)
Depreciation and other amortisation expense (W3) (42)
Other expenses (W3) (359)
Profit before tax 759
Income tax expense (120)
Profit for the period 639
Statement of financial position as at 31 December 2015
Assets
Non-current assets
Property, plant and equipment (W1) 368
Intangible assets (W2) 40
408
Current assets
Inventories (180 + 140) 320
Trade and other receivables (420 x 95%) 399
Cash 440
1,159
Total assets 1,567
Equity and liabilities
Equity
Share capital 600
Other reserves 120
Retaind earnings 635
1,355
Current liabilities
Trade and other payables 92
Current tax payable 120
212
Total equity and liabilities 1,567
Statement of changes in equity for the year ended 31 December 2015
Amounts in Rs. million
Share Revaluati Retained
capital on earnings Total
reserve
Balance at 31 December 2014 620 121 721
Dividends paid (125) (125)
Net revaluation surplus in the year (360 – (300 – 120 - 120
60))
Profit after tax for the period - - 639 639
Balance at 31 December 2015 620 120 635 1,355

© Emile Woolf International 97 The Institute of Chartered Accountants of Pakistan


Financial accounting and reporting II

Workings
(1) Property, plant and equipment
Rs. in
million
Cost brought forward
Leasehold 300
Computers 50
Revaluation 60
Cost carried forward 410
Accumulated depreciation brought forward (60 + 20) 80
Revaluation (60)
Charge for the year
Leasehold (360 ÷ 30) 12
Computers (50 ÷ 5) 10
Accumulated depreciation carried forward 42
Carrying amount carried forward 368

(2) Intangible assets

Cost 60
Amortisation (60 ÷ 3) (20)
Carried forward 40

(3) Allocation of costs


Amounts in Rs. million
Change in Depreciati Other
Staff costs
inventories on etc expenses
Work-in-progress (140 – 125) (15)
Staff costs 260
Finished goods (180 – 155) (25)
Consultancy fees 44
Directors’ salaries 360
Doubtful receivables (420  5%) 21
Sundry 294
Amortisation of patent (W2) 20
Depreciation (12 + 10) (W1) 22
(40) 620 42 359

© Emile Woolf International 98 The Institute of Chartered Accountants of Pakistan


Answers

2.3 BARRY
Barry
Statement of profit or loss
For the year ended 31st August 2015
Rs. in
million
Revenue 30,000
Cost of sales (W1) (19,650)
Gross profit 10,350
Distribution costs (W1) (1,370)
Administrative expenses (W1) (1,930)
Profit from operations 7,050
Finance costs (350)
Profit before tax 6,700
Tax (W2) (2,500)
Profit after tax 4,200
Barry
Statement of financial position
As at 31st August 2015
Rs. in
million
ASSETS
Non-current assets
Property, plant and equipment 39,600
Current assets
Inventory 4,600
Trade and other receivables (7,400 + 200) 7,600
Cash and cash equivalents 700
12,900
Total assets 52,500
EQUITY AND LIABILITIES
Capital and reserves
Equity shares 21,000
Share premium 2,000
Accumulated profits (W3) 11,800
Total equity 34,800
Revaluation reserve (W4) 4,700
Non current liabilities
Borrowings 5,200
Current liabilities
Trade and other payables 5,300
Taxation (2,100 + 400) 2,500
7,800
Total equity and liabilities 52,500

© Emile Woolf International 99 The Institute of Chartered Accountants of Pakistan


Financial accounting and reporting II

Reconciliation of opening and closing property, plant and equipment


Rs. in ‘000
Assets
Plant & Fixtures
under
Land Buildings machine & Total
construct
ry fittings
ion
Cost/ Valuation
At 1 Sept 2014 10,000 9,000 20,100 10,000 400 49,500
Additions - - - - 50 50
Reclassification - - 450 - (450) -
Revaluation 1,000 1,000 - - - 2,000
At 31 Aug 2015 11,000 10,000 20,550 10,000 - 51,550
Depreciation
At 1 Sept 2014 - 3,000 4,000 3,700 - 10,700
Revaluation - (3,000) - - - (3,000)
Charge for year - 1,000 2,550 700 - 4,250
At 31 Aug 2015 - 1,000 6,550 4,400 - 11,950
Net book value
At 31 Aug 2015 11,000 9,000 14,000 5,600 - 39,600
At 1 Sept 2014 10,000 6,000 16,100 6,300 400 38,800
Workings
1 Allocation of expenses Rs.in ‘000
Cost of Admin Distrib
sales
Raw materials consumed 9,500
Manufacturing overheads 5,000
Increase in inventories (1,400)
Staff costs (70%/20%/10%) 3,290 940 470
Distribution costs 900
Depreciation
Building (50%/50%) 500 500
Plant and machinery 2,550
Fixtures and fittings (30%/70%) 210 490
19,650 1,930 1,370
2 Tax charge
Rs. in ‘000
Current year 2,100
Under provision from previous year 400
2,500

© Emile Woolf International 100 The Institute of Chartered Accountants of Pakistan


Answers

3 Accumulated profits carried forward


Rs. in ‘000
Accumulated profits carried forward per question 14,000
Less tax charge
- Current year estimate 2,100
- Under-provision in previous year 400
(2,500)
Add transfer of excess depreciation on revalued building 300
11,800
4 Revaluation reserve carried forward
Revaluation reserve per question 5,000
Add transfer of excess depreciation on revalued building (300)
4,700

2.4 OSCAR INC


(a) Statement of profit or loss For the year ended 31 March 2015

Rs. in ‘000
Sales 2,010
Operating costs (140 + 960 – 150 + 420 + 210 + 16) (1,596)
————
Operating profit before interest 414
Income from investments 75
————
Profit before taxation 489
Income tax (49)
————
440
————
Statement of financial position As at 31 March 2015
Assets
Non-current assets
Tangible assets 530
Investments 560
————
1,090
Current assets
Inventory 150
Receivables 470
————
620
————
1,710
————
Equity and liabilities
Capital and reserves
Share capital 600
Retained earnings 500
————
1,100
Current liabilities 414
Provisions for liabilities and charges 196
————
1,710
————

© Emile Woolf International 101 The Institute of Chartered Accountants of Pakistan


Financial accounting and reporting II

Workings

(1) Income tax


Rs. in ‘000
Income tax (current year) 74
Over provision for tax in the previous year (25)
——
49
——
(2) Tangible assets – plant and machinery
Rs. in ‘000
Cost at 1 April 2014 and 31 March 2015 750
——
Accumulated depreciation
At 31 March 2014 188
Provided during the year (27 + 5) 32
——
At 31 March 2015 220
——

Net book value at 31 March 2015 530


——

(3) Current liabilities


Rs. in ‘000
Trade payables 260
Mainstream corporation tax 74
Bank overdraft 80
——
414
——

(4) Provisions for liabilities and charges


Rs. in ‘000
At 1 April 2014 180
Provided in the year 16
——
At 31 March 2015 196
——
(5) Retained earnings
Rs. in ‘000
Retained earnings 440
Opening retained earnings 180
Dividends (120)
———
Closing retained earnings 500
———

© Emile Woolf International 102 The Institute of Chartered Accountants of Pakistan


Answers

2.5 CLIFTON PHARMA LIMITED


(a) Clifton Pharma Limited
Statement of profit or loss for the year ended 30 September 2015
Rs. in ‘000
Revenue 338,300
Cost of sales: see working (1) (180,000)
Gross profit 158,300
Operating expenses: see working (2) (35,000)
Investment income 2,000
Finance costs: Loan notes – see working (3) (3,000)
Finance lease – see working (2) (1,700)
(4,700)
Profit before tax 120,600
Income tax expense: see working (4) (21,000)
Profit for the period 119,600
(b) Clifton Pharma Limited
Statement of financial position as at 30 September 2015
Non-current assets
Property, plant and equipment: see working (5) 358,000
Investments 94,000
452,000
Current assets
Inventory 23,700
Trade receivables 76,400
Bank 12,100
112,200
Total assets 564,200

Equity and liabilities


Capital and reserves
Share capital 280,000
Share premium 20,000
Retained earnings: see working (6) 138,900
418,900
Non-current liabilities
3% loan notes: see working (3) 51,500
Deferred tax: see working (4) 23,000
Finance lease obligation: see working (2) 11,700
86,200
Current liabilities
Trade payables 14,100
Accrued lease finance costs: see working (2) 1,700
Finance lease obligation: see working (2) 5,300
Income tax payable 18,000
39,100
Total equity and liabilities 564,200
Workings

© Emile Woolf International 103 The Institute of Chartered Accountants of Pakistan


Financial accounting and reporting II

(1) Cost of sales Rs. in ‘000


As given in the trial balance 134,000
Depreciation of plant and equipment: 20%  (197,000 – 47,000) 30,000
Depreciation of leased vehicles: 24,000/4 years 6,000
Amortisation of leasehold property: 250,000/25 years 10,000
180,000
(2) Finance lease
Fair value of leased assets 24,000
Less: First rental payment, paid in advance 1 October 2014 (7,000)
Remaining obligation, 1 October 2014 17,000
Interest at 10% to 30 September 2015 (current liability) 1,700
Lease payment due 1 October 2015 7,000
Capital repayment due (= balance, current liability) (5,300)
Remaining lease obligation = non-current liability 11,700
(3) Loan notes
The effective interest rate is 6%. Actual interest paid was Rs.1,500,000 (in trial balance);
therefore the balancing Rs.1,500,000 should be added to the loan notes obligation, to
make the total loan notes liability Rs.50 million + Rs.1,500,000 = Rs.51.5 million.
(4) Taxation
Deferred tax liability b/f 20,000
Deferred tax: credit in the statement of profit or loss 2,000
Deferred tax liability c/f (92,000  25%) 23,000
Tax expense
Income tax on profits for the year 18,000
Deferred tax movement 3,000
Tax charge in the statement of profit or loss 21,000
(5) Non-current assets and depreciation
Leasehold property Rs. in ‘000
Carrying value in the trial balance (250,000 – 40,000) 210,000
Amortisation charge for the year to 30 September 2015 (10,000)
200,000
Re-valued amount 220,000
Transfer to revaluation reserve 20,000
The annual depreciation charges for plant and equipment and the leased vehicles are
shown in workings (1) Rs. in ’000
Cost or Accumulated Carrying
valuation depreciation amount
Leasehold property 220,000 0 220,000
Plant and equipment 197,000 77,000 120,000
(non-leased)
Leased vehicles 24,000 6,000 18,000
441,000 83,000 358,000

© Emile Woolf International 104 The Institute of Chartered Accountants of Pakistan


Answers

(6) Retained profits


At 1 October 2014 (trial balance) 19,300
Profit for the year 119,600
Retained profits at 30 September 2015 138,900

2.6 SARHAD SUGAR LIMITED


(a) Sarhad Sugar Limited –
Statement of profit or loss for the year ended 30 September 2015
Revenue (300,000 – 2,500) 297,500
Cost of sales (w (i)) (225,400)

Gross profit 72,100


Distribution costs (14,500)
Administrative expenses (22,200 – 400 + 100 see note below) (21,900)
Finance costs (1,000)
Profit before tax 34,700
(Income tax expense (11,400 + (6,000 – 5,800 deferred tax)) (11,600)

Profit for the year 23,100

(b) Sarhad Sugar Limited


Statement of financial position as at 30 September 2015

Assets
Non-current assets (w (ii))
Property, plant and equipment (43,000 + 38,400) 81,400
Development costs 14,800

96,200
Current assets

Inventory 20,000
Trade receivables 43,100

63,100

Total assets 159,300


Equity and liabilities:
Equity
Share capital 70,000
Retained earnings (w (iii)) 41,600
117,100
Revaluation reserve (w (iii)) 5,500
Non-current liabilities
Deferred tax 6,000

© Emile Woolf International 105 The Institute of Chartered Accountants of Pakistan


Financial accounting and reporting II

Current liabilities
Trade payables (23,800 – 400 + 100 – re legal action) 23,500
Bank overdraft 1,300
Current tax payable 11,400
36,200
Total equity and liabilities 159,300

Note: As it is considered that the outcome of the legal action against Sarhad Sugar Limited is
unlikely to succeed (only a 20% chance) it is inappropriate to provide for any damages. The
potential damages are an example of a contingent liability which should be disclosed (at Rs.2
million) as a note to the financial statements. The unrecoverable legal costs are a liability (the
start of the legal action is a past event) and should be provided for in full.
Workings (figures in brackets in Rs.000)
(i) Cost of sales: Rs. in ‘000
Per trial balance 204,000
Depreciation (w (iii)) – leasehold property 2,500
– plant and equipment 9,600
Loss on disposal of plant (4,000 – 2,500) 1,500
Amortisation of development costs (w (iii)) 4,000
Research and development expensed (1,400 + 2,400 (w (iii)) 3,800
––––––––
225,400
══════
(ii) Non-current assets: Rs. in ‘000
Leasehold property
Valuation at 1 October 2014 50,000
Depreciation for year (20 year life) (2,500)
––––––––
Carrying amount at date of revaluation 47,500
Valuation at 30 September 2015 (43,000)
––––––––
Revaluation deficit 4,500
══════

Plant and equipment per trial balance (76,600 – 24,600) 52,000


Disposal (8,000 – 4,000) (4,000)
––––––––
48,000
Depreciation for year (20%) (9,600)
––––––––
Carrying amount at 30 September 2015 38,400
══════
Capitalised/deferred development costs
Carrying amount at 1 October 2014 (20,000 – 6,000) 14,000
Amortised for year (20,000 x 20%) (4,000)
Capitalised during year (800 x 6 months) 4,800
––––––––
Carrying amount at 30 September 2015 14,800
══════
Note: development costs can only be treated as an asset from the point where they
meet the recognition criteria in IAS 38 Intangible assets. Thus development costs from
1 April to 30 September 2015 of Rs.4·8 million (800 x 6 months) can be capitalised.
These will not be amortised as the project is still in development.

© Emile Woolf International 106 The Institute of Chartered Accountants of Pakistan


Answers

The research costs of Rs.1·4 million plus three months’ development costs of Rs.2·4
million (800 x 3 months) (i.e. those incurred before 1 April 2015) are treated as an
expense.
(iii) Movements on reserves
Revaluation Retained
surplus earnings
Rs. in ‘000
Balances at 1 October 2014 10,000 24,500
Dividend (6,000)
Comprehensive income 23,100
Revaluation loss (4,500)
Balances at 30 September 2015 5,500 41,600

2.7 BSZ LIMITED


Statement of financial position as at June 30, 2017
Note Rs. in
million
ASSETS
Fixed Assets
Property, plant & equipment 1 576
Intangible assets 2 8

584

Long term advances – considered good 4


Current assets

Stocks in trade 90
Accounts receivable 3 57
Advances, deposits, prepayments and other receivables 4 45
Cash at banks 5 29

221

809
EQUITY AND LIABILITIES
Share capital and reserves
Authorized share capital
50,000,000 shares of Rs. 10 each 500

Issued, subscribed and paid up capital


40,000,000 shares of Rs. 10 each 400
Unappropriated profit 65
465

© Emile Woolf International 107 The Institute of Chartered Accountants of Pakistan


Financial accounting and reporting II

Note Rs. in
million
Surplus on revaluation of fixed assets 120
Non-current liabilities
Deferred taxation 40

Current liabilities
Short term loan 85
Account and other payables 6 82
Provision for taxation 17
184
809
Notes
1. Property, plant and equipment
Operating assets 556
Capital work in progress – building 20
576

1.1 Operating assets Rs. in million


Cost/revalued amount Freehold land Building Machines Fixtures Total
As of July 01 2016 375.0 130.0 100.0 19.0 624.0
Additions - - - 8.0 8.0
Disposals - - (15.0) - (15.0)
As at June 30 2017 375.0 130.0 85.0 27.0 617.0

Accumulated depreciation
As of July 01 2016 - 19.5 22.5 5.9 47.9
For the year - 6.5 18.1
(105 × 85) + 10% × 15 × 8/12) 9.5
(105 × 19) + 10% × 8 × 3/12) 2.1
Disposals - - (5.0) - (5.0)
As at June 30 2017 - 26.0 27.0 8.0 61.0
Carrying amount 375.0 104.0 58.0 19.0 556.0
Depreciation rate - 5% 10% 10%

1.2 Revaluation
During the year 2013, the first revaluation of freehold land was carried out. The valuation was
carried out under market value basis by an independent valuer, Mr. Dee, Chartered Civil
Engineer of M/s SSS Consultants (Pvt.) Ltd., Islamabad. It resulted in a surplus of Rs. 120
million over book values which was credited to surplus on revaluation of fixed assets. Had
there been no revaluation, the value of freehold land would be Rs. 255 million.

© Emile Woolf International 108 The Institute of Chartered Accountants of Pakistan


Answers

1.3 Disposal of machine


Rs. in
million
Proceeds 13.0
Cost 15.0
Accumulated depreciation (5.0)

Carrying amount (10.0)

Profit on disposal 3.0

Note 2017
Rs. in
million
2. Intangible Assets
Cost of computer software/license 10.0

Accumulated Amortization as of July 1, 2016 1.0


Amortization for the year 1.0

Accumulated Amortization as of June 30, 2016 2.0

Carrying value as at June 30, 2017 8.0

Amortization rate 10%

3. Accounts Receivable
Considered good

- Secured 30
- Unsecured 27
57
Considered doubtful 3

60
Less: Provision for bad debts 3.1 3

57

3.1 Provision for bad debts


Balance as at July 1, 2016 3.4
Provision made during the year 1.0
Amount written off during the year (1.4)

Balance as at June 30, 2017 (Rs. 30 million x 10%) 3.0

© Emile Woolf International 109 The Institute of Chartered Accountants of Pakistan


Financial accounting and reporting II

Note 2017
Rs. in
million
4 Advances, Deposits, Prepayments and Other Receivables
Advances
- suppliers - considered good 12
- staffs 6

18
Deposits 11
Prepayments 4
Sales tax receivable 12

45

5 Cash at banks
Cash at banks - current accounts 7
saving accounts 5.1 22

29

5.1: It carries interest / mark up ranging from 3% to 7% per annum.


6 Accounts and other payables
Accounts payable 75
Accrued liabilities 7

82

2.8 YASIR INDUSTRIES LIMITED


Statement of Financial Position as at June 30, 2017

Rs. in
Assets million

Non-current assets
Property, plant and equipment (W2)
351.00
Intangible assets (20 – 12)
8.00
359.00
Current assets
Inventories (W6)
64.50
Trade receivables (W5)
39.00
103.50
462.50

© Emile Woolf International 110 The Institute of Chartered Accountants of Pakistan


Answers

Rs. in
million
Equity and Liabilities
Equity
Issued, subscribed and paid up capital 120.00
Retained earnings (W4) 87.10
207.10
Revaluation surplus 41.25
Non-current liabilities
Redeemable preference shares 40.00
Debentures 80.00
Deferred taxation (W 10) 9.00
129.00
Current liabilities
Trade payables 30.40
Accrued expenses (W3) 25.00
Taxation 16.50
Bank overdraft 13.25
85.15
Total equity and liabilities 462.50
Statement of profit or loss for the year ended June 30, 2017
Sales revenue (W5) 445.40
Cost of sales (W7) (250.72)
Gross profit 194.68
Distribution costs (W8) (20.05)
Administrative expenses (W8) (40.38)
Financial charges (W9) (9.10)
125.15
Loss due to fraud (30.00)
Profit before tax 95.15
Income tax expense (W10) (19.50)
Profit for the year 75.65
Workings
(W1) Leasehold property

Annual depreciation before the revaluation (230 ÷ 40 years) = Rs. 5.75 million per
annum.
Depreciation this year has been charged incorrectly on cost (whereas it should have
been on the revalued amount).
This year’s charge must be added back
Dr Cr
Accumulated depreciation 5.75
Cost of sales (50%) 2.88
Administrative expenses (30%) 1.72
Distribution costs (20%) 1.15

© Emile Woolf International 111 The Institute of Chartered Accountants of Pakistan


Financial accounting and reporting II

Rs. in
million
Carrying amount at the 30 June (as per trial balance)(230.00 – 40.25) 189.75
Add back depreciation incorrectly charged (see above) 5.75
Carrying amount of property at the start of the year 195.5

Revaluation surplus

Revalued amount of leasehold property 238.00


Less: WDV of leasehold property at revaluation 195.50
Revaluation surplus arising in the year 42.50
Transfer to retained earnings in respect of incremental depreciation (Rs.
7 million – Rs. 5.75 million) (1.25)
41.25
Depreciation of revalued property

Number of years depreciation by the year end: (40.25 ÷ 5.75) = 7 years.


Therefore, remaining useful life as at the year-end = 33 years
Revaluation was at the start of the year
Remaining useful life at the start of the year = 34 years
Depreciation charge based on the revalued amount (238/34 years) = Rs. 7 million
Dr Cr
Cost of sales (50%) 3.5
Administrative expenses (30%) 2.1
Distribution costs (20%) 1.4
Accumulated depreciation 7.00
(W2) Property, plant and equipment
Rs. in
million
Leasehold property (Rs. 238m – 7) 231
Machines (Rs. 168.6 – Rs. 48.6m) 120
351
(W3) Accrued Expenses
As per trial balance 15.00
Accrued interest on debentures (Rs. 80m × 12% × 6/ ) 4.80
12

Dividend on preference shares (Rs. 40m × 10%) 4.00


23.80
(W4) Retained earnings
Balance as per trial balance 10.20
Profit for the year 75.65
Transfer from revalution surplus 1.25
87.10

© Emile Woolf International 112 The Institute of Chartered Accountants of Pakistan


Answers

(W5) Sales and receivables


Sales. Rec.
Rs. in Rs. m
million
Given in the trial balance 478.40 66.00
Deduct revenue incorrectly recognised (sale or return) (27.00) (27.00)
Cost of sales 451.40 39.00
(W6) Closing inventory
Rs. in
million
Given in the question 42.00
Add back inventory held by customer on sale or return ( 100/120  27) 22.50
Cost of sales 64.50
(W7) Cost of sales
Rs. in
million
Opening inventory as of July 1, 2016 38.90
Purchases 175.70
Direct labour 61.00
Manufacturing overheads excluding incremental depreciation 39.00
Less: Closing inventory (64.50)
Deduct depreciation incorrectly charged on cost (2.88)
Add depreciation charged on revalued amount 3.50
Cost of sales 250.10
(W8) Administrative expenses and distribution costs
Admin. DIst.
Rs. in Rs. m
million
Given in the trial balance 40.00 19.80
Deduct depreciation incorrectly charged on cost (1.72) (1.15)
Add depreciation charged on revalued amount 2.10 1.40
Cost of sales 40.38 20.05
(W9) Financial charges
Rs. in
million
Balance as per trial balance 0.30
Accrued interest on debentures (Rs. 80m × 12% × 6/12) 4.80
Preference dividend for the year (Rs. 40m × 10%) 4.00
9.10

© Emile Woolf International 113 The Institute of Chartered Accountants of Pakistan


Financial accounting and reporting II

(W10) Taxation
Deferred taxation Rs. in
million
Balance b/f 6.00
Charge for the year (balancing figure) 3.00
Balance c/f (30%  Rs. 30 million temporary difference) 9.00
Tax expense
Current tax 16.50
Deferred tax (see above) 3.00
19.50

2.9 SHAHEEN LIMITED


Shaheen Limited
Statement of financial position
As of June 30, 2017
Assets Rs. in ‘000
Non-current assets
Property, plant and equipment (86,000  12,000  4,500) 69,500
Intangible assets (6,000  600) 5,400
74,900
Current assets
Stock in trade 30,000
Trade receivables (37,800  10,000) 27,800
Other receivables and prepayments (14,000 + 6,000) 20,000
Cash and bank balances 4,725
82,525
157,425
Equity and liabilities
Share capital and reserves
issued, subscribed and paid up capital 60,000
Unappropriated profit 35,372
95,372
Non-current liabilities
Long term borrowings (31,525  6,000) 25,525
Deferred taxation (5,000  1,470) 3,530
29,055
Current liabilities
Trade payables 12,000
Current portion of long term borrowings 6,000
Provision for litigation 5,000
Provision for taxation (2,000 + 9,988  2,000) 9,998
32,998
157,425

© Emile Woolf International 114 The Institute of Chartered Accountants of Pakistan


Answers

Shaheen Limited
Statement of profit or loss and other comprehensive income
As of June 30, 2017 Rs. in ‘000
Sales revenue 200,000
Cost of sales (W2) (104,708)
Gross profit 95,292
Selling and distribution expenses (W2) (36,275)
Administrative expenses (W2) (30,450)
(66,725)
Financial charges (5,000)
Profit before taxation 23,567
Taxation (W3) (6,528)
Profit after taxation 17,039
Other comprehensive income – net of tax -
Total comprehensive income 17,039

Shaheen Limited
Statement of changes in equity 2017
As of June 30, 2017 Rs.000
Issued,
subscribed Retained
& paid up earnings
capital
Balance July 1, 2016 60,000 32,000*
Correction of prior year error (10,000  20/120) (1,667)
Balance July 1, 2016 (restated) 60,000 30,333
Comprehensive income for the year 17,039
Dividend for the year ended June 30, 2016 (60,000*0.20) (12,000)
Balance June 30, 2017 60,000 35,372

*Retained earnings as at 01-07-11 = 20,000+ (20% of 60,000)=32,000

Workings
W1 Depreciation for the year
On building (36,000/20) 1,800
On plant and equipment (30,000  3,000)/10 2,700
Total 4,500

© Emile Woolf International 115 The Institute of Chartered Accountants of Pakistan


Financial accounting and reporting II

Selling and
Cost of Administrative
distribution
sales costs
W2 Costs costs
Opening inventory 23,000
Costs as per Trial balance 100,000 35,000 30,000
Closing inventory (30,000)
Depreciation (75%, 15%, and 10% of Rs. 4,500) 3,375 675 450
Adjustment for goods sent on sale or return,
erroneously booked as sales last year now
returned during the year. (10,000/1.2) 8,333
Note: It is assumed that sale on return
arrangement does not always result in sale of
goods
Amortization of export license (6,000/5*0.5) 600
104,708 36,275 30,450
W3:Taxation
Profit before tax 23,567
Disallowances and add backs 5,000
Taxable income 28,567
Current For the year (28,567*0.35) 9,998
For prior years (7,000  5,000) (2,000)
Deferred For the year (5,000  800)*0.35 (1,470)
6,528

2.10 MOONLIGHT PAKISTAN LIMITED


(a) Moonlight Pakistan Limited
Statement of Financial Position
As at December 31, 2017
Rs. in
million
ASSETS
Non-current assets
Property, plant and equipment (W2) 3,472
Current assets
Stocks in trade 758
Trade receivables 702
Cash and bank 354
1,814
5,286
EQUITY
Issued, subscribed and paid-up capital (W3) 1,750
Share premium (420 x 2/12) 70
Retained earnings (W3) 876
2,696
Surplus on revaluation of fixed assets 240

© Emile Woolf International 116 The Institute of Chartered Accountants of Pakistan


Answers

Rs. in
million
LIABILITIES
Non-current liabilities
Long term loan 1,600
Deferred tax (22 + 80 x 35%) 50
Provision for gratuity 23
1,673
Current liabilities
Creditor and other liabilities (544 + 96) 640
Income tax payable 37
677
5,286
(b) Moonlight Pakistan Limited
Statement of profit or loss
For the year ended December 31, 2017
Sales 3,608
Cost of sales (W1) (2,149)
Gross profit 1,459
Selling expenses (W1) 252
Administrative expenses (W1) 270
522
937
Financial charges (210 + 1,600 x 12% x 6/12) 306
Profit before taxation 631
Taxation (37 + 80 x 35%) 65
Profit after taxation 566

W1: Cost of sales/selling expenses/admin expenses


Cost of Selling Admin.
sales expenses expenses
Rs. in million
As per trial balance 1,784 220 250
Depreciation – building (60% : 25% : 15%) (W2) 69 29 17
Depreciation – plant 287 - -
Provision for gratuity (23-8) x 60%:20%:20% 9 3 3
2,149 252 270

© Emile Woolf International 117 The Institute of Chartered Accountants of Pakistan


Financial accounting and reporting II

W2: Property, plant and equipment


Land Building Plant Total
Rs. in million
Cost as at January 1, 2017 600 2,000 2,104 4,704
Accumulated depreciation - (400) (670) (1,070)

Revaluation (1,840 - (2,000 - 400 )) - 240 - 240


Current year depreciation - (287) (402)
(1,840/16) (115)
600 1,725 1,147 3,472

W3: Share Capital/Retained Earnings


Retained
Share capital
earnings
Rs. in million
As per trial balance 1,200 510
Bonus issue (1200 ÷ 6) 200 (200)
Right issue (420 x 10/12) 350 -
Profit for the year - 566
1,750 876

2.11 FIGS PAKISTAN LIMITED


Figs Pakistan Limited
Statement of profit or loss and other comprehensive income
For the year ended 31 December 2017
2017
Note Rs. in million
Sales 1 44,758
Cost of sales 2 (26,203)
Gross profit 18,555
Distribution costs 3 (6,431)
Administrative expenses 4 (752)
Other operating expenses 5 (399)
Other operating income 6 30
Profit from operations 11,003
Finance costs 7 (166)
Profit before tax 10,837
Taxation 8 (2,532)
Profit after tax 8,305
Other comprehensive income -
Total comprehensive income for the year 8,305

© Emile Woolf International 118 The Institute of Chartered Accountants of Pakistan


Answers

2017
Note Rs. in million
Figs Pakistan Limited
Notes to the financial statements
For the year ended 31 December 2017
1 Sales
Manufactured goods
Gross sales 56,528
Sales tax (10,201)
46,327
Imported goods
Gross sales 1,078
Sales tax (53)
1,025
Sales discounts (2,594)
44,758
2 Cost of sales
Raw material consumed (1,751 + 22,603 - 2,125) 22,229
Stores and spares consumed 180
Salaries, wages and benefits (2,367 × 55%) 2.1 1,302
Utilities (734 × 85%) 624
Depreciation and amortizations (1.287 × 70%) 901
Stationery and office expenses (230 × 25%) 58
Repairs and maintenance (315 × 85%) 268
25,562
Opening work in process 73
Closing work in process (125)
25,510
Opening finished goods (manufactured) 1,210
Closing finished goods (manufactured) (1,153)
25,567
Finished goods (imported)
Opening stock 44
Purchases 658
702
Closing stock (66)
636
26,203
2.1 Salaries, wages and benefits include Rs. 30 million (54 × 55%) and Rs. 24 million (44 × 55%)
in respect of defined contribution plan and defined benefit plan respectively.

© Emile Woolf International 119 The Institute of Chartered Accountants of Pakistan


Financial accounting and reporting II

3 Distribution costs Rs. in million


Advertisement and sales promotion 4,040
Outward freight and handling 1,279
Salaries, wages and benefits (2,367 × 30%) 3.2 710
Utilities (734 × 5%) 37
Depreciation and amortization (1,287 × 20%) 257
Stationery and office expenses (230 × 40%) 92
Repairs and maintenance (315 × 5%) 16
6,431

3.1 Salaries, wages and benefits include Rs. 16 million (54 × 30%) and Rs. 13 million (44×30%)
in respect of defined contribution plan and defined benefit plan respectively.

4 Administrative expenses Rs. in million


Salaries, wages and benefits (2,367 × 15%) 4.1 355
Utilities (734 × 10%) 73
Depreciation and amortization (1,287 × 10%) 129
Stationery and office expenses (230 × 35%) 80
Repairs and maintenance (315 × 10%) 31
Legal and professional charges 71
Auditor's remuneration 4.2 13
752

4.1 Salaries, wages and benefits include Rs. 8 million (54 × 15%) and Rs. 7 million (44×15%) in
respect of defined contribution plan and defined benefit plan respectively.

4.2 Auditor's remuneration Rs. in million


Audit fees 8
Taxation services 4
Out of pocket expenses 1
13
5 Other operating expenses
Donation 5.1 34
Worker's Profit Participation Fund 257
Worker Welfare Fund 98
Loss on disposal of property, plant and equipment 10
399
5.1 Donations

Donations include Rs. 5 million given to Dates Cancer Foundation (DCF). One of the
company’s directors, Mr. Peanut is a trustee of DCH.

Donations other than that mentioned above were not made to any donee in which a director
or his spouse had any interest at any time during the year.

© Emile Woolf International 120 The Institute of Chartered Accountants of Pakistan


Answers

6 Other operating income Rs. in million


Income from financial assets
Dividend income 12
Return on savings account 2
Income from non-financial assets
Scrap sales 16
30
7 Finance costs
Finance charges on short term borrowings 133
Exchange loss 22
Finance charges on lease 11
166
8 Taxation
Current - for the year 1,440
Deferred (3,120 × 35%) 1,092
2,532

2.12 WAH AGRIPROD LTD


(a) Wah Agriprod Ltd: Statement of profit or loss and other comprehensive income for the year
ended 31 December 2016.

Notes Rs.‘000
Revenue (wi) 851,400
Cost of sales (wii) (495,532)

Gross profit 355,868


Distribution cost (31,950)
Administrative expenses (wiii) (79,100)
Profit from operations 244,818
Investment income 18,250
Finance cost (1,020)
Fair value loss on financial instrument
(40,500 – 39,700) (800)
Profit before tax 261,248
Income tax expenses (86,750)

Profit for the year 174,498


Other comprehensive income:
Fair value gain on leasehold property 12,333

Total comprehensive income for the year 186,831

© Emile Woolf International 121 The Institute of Chartered Accountants of Pakistan


Financial accounting and reporting II

Working Notes:

Rs.
Revenue
(i) Rs. 855,000 – Rs. 3,600 value of returnable goods = 851,400
(ii) Cost of sales:
Opening inventory 85,075
Purchases 503,600
Depreciation of plant and equipment
(0.15 of Rs. 98,800 – Rs. 28,800) 10,500
Amortization of leased property
(Rs. 3,833 + Rs. 4/208 of Rs. 99,000) 5,237
Closing inventory
(Rs. 106,000 + 0.8 of Rs. 3,600) (note iv) (108,880)
495,532
(iii) Admin expenses .
Per the question 104,400
Less dividend = Rs. 115,00 x 2 x Rs. 2.2 x Rs.0.05 (25,300)
79,100
(iv) Closing inventory: As per question 106,000
Sales return at cost (80% of 3,600) 2,880
108,880

(b) Statement of changes in equity for the year ended 31 December 2016

Revaluati
Share Share on Retained
capital premium surplus earnings Total
Rs.’000 Rs.’000 Rs.’000 Rs.’000 Rs.’000
Balance as at 1/1/2016 105,000 6,400 - 55,600 167,000
Issue of shares 10,000 2,000 - - 12,000
Profit for the year - - - 174,498 174,498
Fair value gain - - 12,333 - 12,333
Realised during the year - - (237) 237 -
Dividend paid - - - (25,300) (25,300)
Balance 31/12/2016 115,000 8,400 12,096 205,035 340,531

Revaluation of leased property Rs.‘000 Rs.‘000


Cost as at 1/1/2016 125,000
Accumulated depreciation as at 1/1/2016 35,000
For the eight months to 1/9/2016:
8/12 of 125,000 ÷ 25 years 3,333 38,333
86,667
Value of leased property 99,000
(12,333)

© Emile Woolf International 122 The Institute of Chartered Accountants of Pakistan


Answers

CHAPTER 3 – CONSOLIDATED ACCOUNTS: STATEMENTS OF FINANCIAL


POSITION– BASIC APPROACH
3.1 P AND S (I)
Consolidated Statement of Financial Position
P S Adjustments Consolidated
Non current assets 50 40 - 90
Goodwill (W-3) 5
Current assets 30 40 70
80 80 - 165

Share capital 100 100


Group retained earnings (W-4) 35
Current liabilities 20 10 30
120 10 - 165

W1 Group Structure S
P 100%
NCI 0%

W2 Net assets of subsidiary


At Acquisition At Reporting Post acquisition
date date profit
Share capital 50 50
Retained earnings 15 20 5
65 70 5

W3 Goodwill
Purchase consideration 70
NCI -
70
Less: FV of Net assets (65)
Goodwill 5

W4 Group retained earnings


Parent's retained earnings 30
Post-acquisition share 5
35

© Emile Woolf International 123 The Institute of Chartered Accountants of Pakistan


Financial accounting and reporting II

3.2 P AND S (II)


Consolidated Statement of Financial Position
P S Adjustments Consolidated
Non current assets 50 40 - 90
Goodwill (W-3) 14
Current assets 30 40 70
80 80 - 174

Share capital 100 100


Group retained earnings (W-5) 30
Non-controlling interest (W-4) 14.00
Current liabilities 20 10 30
120 10 - 174

W1 Group Structure S
P 80%
NCI 20%

W2 Net assets of subsidiary


At Acquisition At Reporting Post acquisition
date date profit
Share capital 50.00 50.00
Retained earnings 15.00 20.00 5.00
65.00 70.00 5.00

W3 Goodwill
Purchase consideration 70.00
NCI 13.00
83.00
Less: FV of Net assets (65.00)
Goodwill 18.00
Less: Impairment (3.60)
Goodwill at reporting 14.40

W4 Non-controlling interest
NCI at acquisition 13.00
NCI post-acquisition share 1.00
14.00

© Emile Woolf International 124 The Institute of Chartered Accountants of Pakistan


Answers

W5 Group retained earnings


Parent's retained earnings 30.00
Post-acquisition share 4.00
Impairment loss (3.60)
30.40

3.3 BL AND FL
Consolidated Statement of Financial Position
BL FL Adjustments Consolidated
Non current assets 86,000 24,500 2,000 112,500
Goodwill (W-3) 4,640
Current assets 20,000 10,000 30,000
106,000 34,500 2,000 147,140

Share capital 100,000 100,000


Group retained earnings (W-5) 22,440
Non-controlling interest (W-4) 5,700.00
Current liabilities 11,000 8,000 19,000
111,000 8,000 - 147,140

W1 Group Structure FL
BL 80%
NCI 20%

W 2 Net assets of subsidiary


At Acquisition At Reporting Post acquisition
date date profit

Share capital 20,000 20,000


Retained earnings 6,000 6,500 500
FV adjustment
- Land 2,000 2,000
- Inventory (1,500) - 1,500
26,500 28,500 2,000

W 3 Goodwill
Purchase consideration 27,000
NCI 5,300
32,300
Less: FV of Net assets (26,500)
Goodwill 5,800
Less: Impairment (1,160)
Goodwill at reporting 4,640

© Emile Woolf International 125 The Institute of Chartered Accountants of Pakistan


Financial accounting and reporting II

W 4 Non-controlling interest
NCI at acquisition 5,300
NCI post-acquisition share 400
5,700

W 5 Group retained earnings


Parent's retained earnings 22,000
Post-acquisition share 1,600
Impairment loss (1,160)
22,440

3.4 ML AND ZL
Consolidated Statement of Financial Position
ML ZL Adjustments Consolidated
Non current assets 41,000 16,000 - 57,000
Goodwill (W-3) 2,000
Current assets 20,000 28,000 500 48,500
61,000 44,000 500 107,500

Share capital 50,000 50,000


Group retained earnings (W-5) 35,100
Non-controlling interest (W-4) 7,700
Current liabilities 10,000 5,000 (300) 14,700
60,000 5,000 (300) 107,500

W1 Group Structure ZL
ML 75%
NCI 25%

W 2 Net assets of subsidiary


At Acquisition At Reporting Post acquisition
date date profit
Share capital 10,000 10,000
Retained earnings 12,000 20,000 8,000
Expense incurred by ML (200) (200)
22,000 29,800 7,800

© Emile Woolf International 126 The Institute of Chartered Accountants of Pakistan


Answers

W 3 Goodwill
Purchase consideration 19,000
NCI at FV 6,000
25,000
Less: FV of Net assets (22,000)

Goodwill 3,000
Less: Impairment (1,000)
Goodwill at reporting 2,000

W 4 Non-controlling interest
NCI at acquisition 6,000
NCI post-acquisition share 1,950
Impairment loss (250)
7,700

W 5 Group retained earnings


Parent's retained earnings 30,000
Post-acquisition share 5,850
Impairment loss (750)
35,100

W 6 Current account reconciliation


ML current account 10,000
Cheque not received till 3 Jan, 19 (500)
9,500

ZL current account 9,000


Amount wrongly credited 300
Expense incurred on behalf of ML 200
9,500

3.5 FLAMSTEED LTD AND HALLEY LTD


(a) An impairment loss is the amount by which the carrying amount of an asset or a cash
generating unit exceeds its recoverable amount.
(b) The following external sources of information may indicate that an asset is impaired.
(i) There are observable indications that the assets value has declined during the period
significantly more than would be expected as a result of the passage of time or normal
use.

© Emile Woolf International 127 The Institute of Chartered Accountants of Pakistan


Financial accounting and reporting II

(ii) Significant changes with an adverse effect on the entity have taken place during the
period or will take place in the near future, in the technological, market, economic or
legal environment in which the entity operates or in the market to which the asset is
dedicated.
(iii) The carrying amount of the net asset of the entity is more than its market capitalization.
(iv) The carrying amount of the investment in the separate financial statements exceeds the
carrying amount in the consolidated financial statement of the investee’s net asset,
including associated goodwill, or the dividend exceeds the total comprehensive income
of the subsidiary, joint venture or associates in the period the dividend is declared.
(v) Market interest rate or other market rate of return on investment have increased during
the period and those increases are likely to affect the discount rate used in calculating
the asset value in use and decrease the assets recoverable amount materially.
(c) Flamsteed Ltd group: extract of consolidated statement of financial position as at 30 June
2016

Rs.‘000
Assets
Non-Current Assets
Property, Plant and Equipment (100,000 + 80,000) 180,000
Goodwill (WK) 13,468
Intangible-brand name 10,000
203,468
Current Assets
Inventory (6,000 + 16,000) 22,000
Receivables (32,000 + 14,000) 46,000
Cash (4,000 + 0 + 4,000) 8,000
76,000

Total Assets 279,468

Workings
(i) Goodwill

Rs.‘000 Rs.‘000
Consideration transferred 77,468
Fair value of NCI 18,000
Net Asset acquired as represented by: 95,468
Ordinary share capital 50,000
Revaluation surplus on acquisition 10,000
Retained earnings on acquisition 12,000
Intangible assets (brand name) 10,000 (82,000)
13,468

Note
The deferred consideration has been discounted at 7% for 2 years (1 st July 2015 – 1st July
2017).

© Emile Woolf International 128 The Institute of Chartered Accountants of Pakistan


Answers

CHAPTER 4 – CONSOLIDATED ACCOUNTS: STATEMENTS OF FINANCIAL


POSITION– COMPLICATIONS
4.1 HAIL
Consolidated statement of financial position as at 31 December 2015
Rs.000 Rs.000
Assets
Non-current assets
Property, plant and equipment 246,000
Investments (68,000 – 65,000) 3,000
Goodwill (W3) 6,500
Current assets
Cash at bank and in hand 39,900
Trade receivables 138,300
Inventories 92,400
———–
526,100
———–
Equity and liabilities
Capital and reserves
Share capital 100,000
Capital reserve (W6) 18,000
Retained earnings (W5) 210,480
———–
328,480
Non-controlling interest (W4) 11,420

Current liabilities
Trade payables 183,000
Proposed dividend – parent company 3,000
– non controlling interest 200
———– 3,200
———–
526,100
———–
WORKINGS
(1) Group structure

Hail

90%

Snow

© Emile Woolf International 129 The Institute of Chartered Accountants of Pakistan


Financial accounting and reporting II

(2) Net assets of Snow

Reporting date Date of acquisition Post-acquisition


Rs.000 Rs.000
Share capital 50,000 50,000
Share premium account 5,000 5,000
Revaluation reserve 20,000 
Retained earnings
Per question 41,200
Proposed dividend (2,000)
39,200 10,000 29,200
114,200 65,000

(3) Goodwill Rs.000


Cost of shares 65,000
Net assets acquired (90%  65,000) (W2) (58,500)
————
6,500
————
(4) Non-controlling interest
10%  114,200 (W2) 11,420
————
(5) Retained earnings
Hail 185,400
Proposed dividend (3,000)
Dividend receivable from Snow 1,800
Snow (90%  29,200 (W2)) 26,280
————
210,480
————
(6) Capital reserve
Snow (90%  20,000 (W2)) 18,000
————
4.2 HAIRY
Consolidated statement of financial position as at 31 December 2015 Rs.000
Assets
Non-current assets
Property, plant and equipment 180,000
Current assets
Cash at bank and in hand 15,500
Investments 3,000
Receivables 91,700
Inventory (17,000 + 11,000 – 800) 27,200
———–
317,400
———–

© Emile Woolf International 130 The Institute of Chartered Accountants of Pakistan


Answers

Equity and liabilities


Capital and reserves
Share capital 100,000
Share premium account 20,000
Capital reserve 23,000
Retained earnings (W5) 102,900
———–
245,900
Non-controlling interest (W4) 16,500

Current liabilities 55,000


———–
317,400
———–
WORKINGS
(1) Group structure
Hairy

80%

Spider

(2) Net assets of Spider

Reporting date Date of acquisition Post-acquisition

Rs.000 Rs.000

Share capital 60,000 60,000

Share premium account 16,000 16,000

Retained earnings

Per question 7,300

Unrealised profit (800)

6,500 2,300 4,200

82,500 78,300

(3) Goodwill Rs.000


Cost of shares 55,000
Less Net assets acquired (80%  78,300 (W2)) (62,640)
————
(7,640)
————

© Emile Woolf International 131 The Institute of Chartered Accountants of Pakistan


Financial accounting and reporting II

(4) Non-controlling interest Rs.000


Share of net assets (20%  82,500 (W2)) 16,500
————

(5) Retained earnings


Hairy 91,900
Spider (80%  4,200 (W2)) 3,360
Negative goodwill (W4) 7,640
————
102,900
————
4.3 HARD
Consolidated statement of financial position as at 31 December 2015
Assets Rs.000
Non-current assets
Property, plant and equipment (225 + 175 – 17.5 (W6)) 382,500
Goodwill (W3) 14,000
Current assets (271 + 157) 428,000
———–
824,500
———–
Equity and liabilities
Shareholders’ equity
Share capital 100,000
Share premium account 15,000
Retained earnings (W5) 260,500
———–
375,500
Non-controlling interest (W4) 76,000
Current liabilities 373,000
———–
824,500
———–

WORKINGS
(1) Group structure

Hard

60%

Soft

© Emile Woolf International 132 The Institute of Chartered Accountants of Pakistan


Answers

(2) Net assets of Soft Inc

31 Dec 2015 31 Dec 2014 Post-acquisition


Rs.000 Rs.000

Share capital 100,000 100,000


Share premium account 10,000 10,000
Retained earnings 80,000 50,000 30,000
190,000 160,000

(3) Goodwill Rs.000


Cost 110,000
Net assets acquired 60%  160,000 (W2) (96,000)
————
14,000
————
(4) Non-controlling interest
40%  190,000 (W2) 76,000

(5) Retained earnings


Hard 260,000
Less Adjustment re intra group transfer (17,500)
————
242,500
Soft (60%  (80,000 – 50,000 (W2)) 18,000
————
260,500
————
(6) PURP on non current assets
IS
Cost 50,000
Accumulated depreciation (12,500)
————
37,500
————
SHOULD BE
Cost 100,000
Accumulated depreciation (80,000)
————
20,000
————
Dr Retained earnings 17,500
Cr Non current assets 17,500

© Emile Woolf International 133 The Institute of Chartered Accountants of Pakistan


Financial accounting and reporting II

4.4 HALE
(a) Consolidated statement of financial position as at 31 December 2015
Rs.000
Assets
Non-current assets
Property, plant and equipment
(152,000 + 129,600 + 28,000 (W2)) 309,600
Goodwill (W3) 61,400
Current assets
Bank (41,000 + 8,000) 49,000
Receivables (104,000 + 84,000) 188,000
Inventory (112,000 + 74,400 – 3,200 (W6)) 183,200
————–
791,200
————–
Equity and liabilities
Capital and reserves
Share capital 100,000
Retained earnings (W5) 555,200
————–
655,200
Non-controlling interest (W3) 60,000
Current liabilities (52,000 + 24,000) 76,000
————–
791,200
————–
WORKINGS
(1) Group structure
Hale

128
160
= 80% ord
ordords
ords

Sowen
(2) Net assets of Sowen

Reporting Date of Post-


date acquisition acquisition

Rs.000 Rs.000

Share capital 160,000 160,000

Fair value adjustment on non-


current assets 28,000 28,000

Retained earnings 112,000 (11,000) 123,000

300,000 177,000

© Emile Woolf International 134 The Institute of Chartered Accountants of Pakistan


Answers

(3) Goodwill Rs.000


Cost of shares 203,000
Less Net assets acquired (80%  177,000 (W2)) (141,600)
————–
61,400
————–
(4) Non-controlling interest
Share of net assets (20%  300,000 (W2)) 60,000
————
(5) Retained earnings
Hale 460,000
PURP (W6) (3,200)
Sowen (80%  123,000 (W2)) 98,400
————–
555,200
————–
(6) Unrealised profits % Rs.000
SP 125 16,000
Cost (100) (12,800)
—— ———
GP 25 3,200
—— ———
4.5 HELLO
Consolidated statement of financial position as at 31 December 2015
Assets Rs.
Non-current assets
Property, plant and equipment (225 + 175 + 10 – 2) 408,000
Goodwill (W3) 8,000
Current assets (271 + 157) 428,000
———–
844,000
———–
Equity and liabilities
Shareholders’ equity
Called up share capital 100,000
Retained earnings (W5) 291,800
———–
391,800
Non-controlling interest (W4) 79,200
Current liabilities 373,000
———–
844,000
———–
WORKINGS
(1) Group structure
Hello

60%

Solong

© Emile Woolf International 135 The Institute of Chartered Accountants of Pakistan


Financial accounting and reporting II

(2) Net assets of Solong Inc


Reporting Date of Post-
date acquisition acquisition
Rs. Rs.
Share capital 100,000 100,000
Retained earnings
Per the question 90,000
Less: Fair value adjustment for
depreciation (2/10 × 10,000) (2,000)
88,000 60,000
Fair value adjustment 10,000 10,000
198,000 170,000
(3) Goodwill Rs.
Cost 110,000
Net assets acquired
60%  170,000 (W2) (102,000)
————
8,000
————
(4) Non-controlling interest
40%  198,000 (W2) 79,200
(5) Retained earnings
Hello 275,000
Solong (60%  (88,000 – 60,000 (W2)) 16,800
———–
291,800
———–
4.6 HASAN LIMITED
Hasan Limited
Consolidated statement of financial position as at 31 March 2015
Rs.000 Rs.000
Assets
Non-current assets
Property, plant and equipment (W1) 4,020
Goodwill (W4) 480
Software (W1) 1,440
Investments (65 + 210) 275
–––––––––––––
6,215
Current assets
Inventories (W2) 1,274
Trade receivables (524 + 328) 852
Cash and bank (20 + 55 cash in transit) 75
–––––––––––––
2,201
–––––––––––––

Total assets 8,416


–––––––––––––

© Emile Woolf International 136 The Institute of Chartered Accountants of Pakistan


Answers

Rs.000 Rs.000
Equity and liabilities
Capital and reserves
Equity capital 2,000
Reserves
Share premium 2,000
Retained earnings (W3) 2,420
–––––––––––––
4,420
–––––––––––––

6,420
Non-controlling interest (W5) 350
Non-current liabilities
Government grants (230 + 40) 270
Current liabilities
Trade payables (475 + 472) 947
Operating overdraft 27
Income tax liability (228 + 174) 402
–––––––––––––
1,376
–––––––––––––

Total equity and liabilities 8,416


–––––––––––––

Workings
(W1) Property, plant and equipment
Rs.000
Balance from question – Hasan Limited 2,120
Balance from question – Shakeel Limited 1,990
Fair value adjustment on acquisition (see below) (120)
Over-depreciation re fair value adjustment year to 31 March 2015 30
–––––––––––––
4,020
–––––––––––––

A fair value of the leasehold based on the present value of the future rentals (receivable in
advance) would be the next (non-discounted) payment of the rental plus the final three years
as an annuity at 10%:

Rs.000
PV of rental receipts: Rs.80,000 + (Rs.80,000  2.50) 280
Carrying value on acquisition is (400)
–––––––––––––
Fair value reduction of leasehold (120)
–––––––––––––

The depreciation of the leasehold in Shakeel Limited’s accounts would be Rs.100,000 per
annum. However in the consolidated accounts it should be Rs.70,000 (Rs.280,000/4). This
would require a reduction in depreciation of Rs.30,000 in the consolidated accounts for the
next four years.

© Emile Woolf International 137 The Institute of Chartered Accountants of Pakistan


Financial accounting and reporting II

Software:

Shakeel Consolidated Difference


Limited’s figures
accounts
Rs.000 Rs.000
Capitalised amount 2,400 2,400
Depreciation to 8 year
31 March 2014 (300) life (480) 5 year life
––––– –––––
Value at date of acquisition 180 fair
2,100 1,920 value adjustment
Depreciation to 180 additional
31 March 2015 (300) (480) amortisation
––––– –––––
Carrying value
31 March 2015 1,800 1,440
––––– –––––

(W2) Inventories

Rs.000
Amounts given in the question (719 + 560) 1,279
Unrealised profit in inventories (25  25/125) (5)
–––––––––––––
1,274
–––––––––––––

(W3) Retained earnings

Rs.000
Retained profits of Shakeel Limited, 31 March 2015 1,955
Adjustments:
Excess charge for leasehold depreciation 30
Insufficient charge for Software amortisation (180)
Unrealised profit in inventory (W2) (5)
–––––––––––––

Adjusted retained profits at 31 March 2015 1,800


Retained earnings of Shakeel Limited at 1 April 2014 2,200
–––––––––––––

Shakeel Limited: loss for the year (post-acquisition loss) (400)


–––––––––––––

Parent company share of post-acquisition loss (90%) (360)


Hasan Limited reserves at 31 March 2015 2,900
Goodwill impairment (120)
–––––––––––––

Consolidated retained profits at 31 March 2015 2,420


–––––––––––––

© Emile Woolf International 138 The Institute of Chartered Accountants of Pakistan


Answers

(W4) Goodwill
Rs.000
At acquisition date
Shares of Shakeel Limited 1,500
Share premium of Shakeel Limited 500
Retained earnings of Shakeel Limited 2,200
Fair value adjustments:
Leasehold (W1) (120)
Software (W1) (180)
–––––––––––––

3,900
–––––––––––––

Acquired by Hasan Limited (90%) 3,510


Cost of investment 4,110
–––––––––––––

Goodwill at acquisition 600


Impairment 120
–––––––––––––

Goodwill at 31 March 2015 480


–––––––––––––

(W5) Non-controlling interests


Rs.000
Share capital of Shakeel Limited 1,500
Share premium of Shakeel Limited 500
Adjusted retained earnings of Shakeel Limited, 31 March 2015 (W3) 1,800
Fair value adjustments:
Leasehold (120)
Software (180)
–––––––––––––
Total net assets at 31 March 2015 3,500
–––––––––––––

Non-controlling interests (10%) 350


–––––––––––––

(W6) Elimination of current accounts:

Rs.000
Shakeel Limited’s current account with Hasan Limited per question 75
Deduct cash in transit regarding this balance (15)
–––––––––––––
Adjusted figure to cancel 60
–––––––––––––

(W7) Elimination of intra-group loan:

Rs.000
Investment in Hasan Limited’s books 200
Deduct repayment in transit (40)
–––––––––––––
Non-current liability in Shakeel Limited’s books 160
–––––––––––––

© Emile Woolf International 139 The Institute of Chartered Accountants of Pakistan


Financial accounting and reporting II

4.7 GOLDEN LIMITED


Consolidated statement of financial position As on 31 December 2016
Rs. in million
Non current Assets
Building (W-2) 2,011.50
Plant & machinery (W-2) 2,151.00

Current Assets
Current assets [2068+780–(112–62)–(24–15)–(50/12×3)] 2,776.50

6,939.00
Equity & liabilities
Share capital 980.00
Share premium 730.00
Consolidated retained earnings (W- 4) 3,239.90

Non controlling interest (W-5) 241.60


Liabilities [600+1160–(50×3/12)] 1,747.50
6,939.00

W-1: Computation of goodwill


Issuance of shares (20×12) 240
Cash payment (87–15) 72
Total consideration paid 312
Add: FV of NCI (45 × 0.4 × 11) 198
Total consideration and FV of NCI 510
Less: FV of net assets acquired (W-3) (561)
Bargain purchase/Negative goodwill - charged to P & L (51)

W-2: Property, plant & equipment Building Plant & Machinery


2,100.00 2,155.00
GL & SL (1,600+500) (1,465+690)
Decrease in fair value of building (80.00) -
Reversal of gain on exchange [120–(240–130)] (10.00) -
Increase in depreciation on reversal of exchange (1.50) (4.00)
transaction (120×5% (200–120)
×3/12) ×20%× 3/12)
Reversal of depreciation on fair value adjustment
-
(80×5%×9/12) 3.00
2,011.50 2,151.00

© Emile Woolf International 140 The Institute of Chartered Accountants of Pakistan


Answers

W-3: Net Asset of SL on year end and on acquisition date 31-Dec-16 At acquisition
-------- Rs. in million --------
Share capital 450 450
Share premium 150 150
Retained earnings 210 100
Decrease in fair value of building (250–170) (80) (80)
Decrease in fair value of inventory (112–62) (50) (50)
Increase in provision for bad debts (24–15) (9) (9)
Reversal of depreciation on fair value adjustment
(80×5%×9/12) 3 -
674 561

Post acquisition profit 113

W-4: Consolidated retained earnings Rs. in million


GL 3,150.00
Wrongly capitalization of acquisition related cost (15.00)
Parent's share in SL's post acquisition profit (113(W-3)×60%) 67.80
Reversal of gain on exchange [120–(240–130)] (10.00)
Increase in depreciation on building due to reversal of exchange transaction
(120×5%×3/12) (1.50)
Increase in depreciation on plant & machinery on reversal of exchange
transaction [(200–120)×20%×3/12×60%] (2.40)
Negative goodwill on acquisition of SL (W-1) 51.00
3,239.90

W-5: Non-controlling interest

Fair value at acquisition (45×40%×11) 198.00


NCI's share in SL's post acquisition profit (113(W-3)×40%) 45.20
Increase in depreciation on plant & machinery on reversal of exchange
transaction [(200–120)×20%×3/12×40%] (1.60)
241.60

4.8 YASIR LIMITED AND BILAL LIMITED


Yasir Limited
Consolidated Statement of financial position As at 30 June 2016

Non-current Assets Rs. in million


Fixed assets [180 + 470 + (12 × 0.9)] 660.80
Goodwill (W-1) 190.35
851.15

© Emile Woolf International 141 The Institute of Chartered Accountants of Pakistan


Financial accounting and reporting II

Rs. in million
Current assets
Stock in trade [160 + 150 – 4.8(Working: 32×0.15) – 4.6(Working: 20÷1.3×0.3)] 300.60
Other current assets (71 + 50) 121.00
Cash and bank (63 + 151 + 5.92) 219.92
641.52
Total assets 1,492.67
Share capital & Reserves
Share capital 750.00
Retained earnings (W-3) 406.20
Non controlling interest (W-4) 210.47
1,366.67
Liabilities
Creditors and other liabilities (75 + 51) 126.00
Total equity & liabilities 1,492.67
W-1 : Computation of Goodwill and its impairment

Cash consideration (50 × 0.75 × 18) 675.00

Fair value of NCI (50 × 0.25 × 15) 187.50

862.50

Less: Net assets (W-2) (651.00)

Goodwill on acquisition date 211.50

Less: Impairment (10%) (21.15)

190.35

W-2 : Net Asset of BL on year end and on acquisition date

30-Jun-16 At acquisition
-------- Rs. in million ---------
Share capital 500.00 500.00
Retained earnings 258.00 *139.00
Increase in fair value of building 12.00 12.00
Depreciation adjustment – building (12 x 5% x 2) (1.20)
Inter company sales (32 x 0.15) (4.80)
764.00 651.00
Post acquisition profit (764-651) 113.00
* [258 – 168 – 11 + 60 (500 x 12%)]

© Emile Woolf International 142 The Institute of Chartered Accountants of Pakistan


Answers

W-3: Consolidated retained earnings

Rs. in million

YL as at June 30, 2016 (given) 340.00

Interest income/expense on loan (16 × 0.12) 1.92

Inter company sales (20 ÷ 1.3 × 0.3) (4.62)

Parent’s share in BL’s post acquisition profit [113(W-2)×75%] 84.75

Impairment of goodwill (21.15 × 75%) (15.85)

406.20
W-4: Non-controlling interest

Fair value at acquisition (50 × 0.25 × 15) 187.50

NCI’s share in BL’s post acquisition profit [113(W-2)×25%] 28.25

Impairment of goodwill (21.15 × 25%) (5.28)

210.47

4.9 JASMINE LIMITED


Consolidated statement of financial position
As on 31 December 2017

Rs. in million

Property, plant and equipment [880+330–8{12–(12÷2×8÷12)}] 1,202.00

Intangible asset (W-1) 203.00

Current assets (640+345+30–25 (150×20÷120)–15(20×75%)–52) 923.00

2,328.00

Share capital (Rs. 10 each) 700.00

Share premium 240.00

Consolidated retained earnings (W-4) 708.25

Non-Controlling Interest (W-5) 187.75

1,836.00

Current liabilities (324+235–15(20×75%)–52) 492.00

2,328.00

W-1: Intangible asset

JL 40.00

SL 50.00

Goodwill (W-S) 105.00

Increase in FV of brand – not of amortization [15 - 3 (15 ÷5)] 12.00

Impairment of brand [(40 ÷ 5 x 4) – 28] 203.00

© Emile Woolf International 143 The Institute of Chartered Accountants of Pakistan


Financial accounting and reporting II

W-2: Computation of goodwill Rs. in million

Cash consideration (280 – 10) 270.00

Issuance of shares (10 × 24) 240.00

Fair value of NCI (20 × 25% × 36) 180.00

690.00

Fair value of net assets (W-3) 585.00

Goodwill 105.00

W-3: Net assets of SL At acquisition At reporting


date date

-------- Rs. in million --------

Share capital 200.00 200.00

Retained earnings 370.00 410.00


(410 – 60 + 20)

Increase in fair value of brand (40 – 25) 15.00 15.00

Amortization of brand due to fair value adjustment (3.00)


(15÷5)

Impairment of brand (4.00)

Interest income (120 × 10% × 6 ÷ 12) 6.00

Unrealised gain on sale of machine (8.00)

585.00 616.00

Post-acquisition 31.00

W-4: Consolidated retained earnings Rs. in million

JL 720.00

Post acquisition of SL [31 (W – 3) × 75%] 23.25

Charge off consulting fee (10.00)

Unrealised profit on closing stock (150 ÷ 120% x 20%)

708.25

W-5: Non-controlling interest Rs. in million

At acquisition 180.00

Post-acquisition of SL [31(W-3) × 25%] 7.75

187.75

© Emile Woolf International 144 The Institute of Chartered Accountants of Pakistan


Answers

4.10 BRADLEY LTD


Consolidated statement of financial position as at 31 December 2016

Rs.’Million Rs.’Million
Non-current assets
Goodwill (working 1) 120
Land & building (630 + 556 + 140) 1,326
Machinery & equipment (570 + 440) 1,010
2,456
Current assets
Inventory (714 +504 – 24) 1,194
Trade receivables (1,050 + 252 – 50) 1,252
Cash/Bank (316 + 60) 376 2,822
5,278
Ordinary shares of Rs. 1 each 3,000.0
Retained earnings (Working 3) 1,323.2
Non-controlling Int. (Working 4) 376.8
4,700
Current liabilities
Trade payables (440 + 188 - 50) 578
5,278

Workings:
Rs. million
1. Calculation of goodwill:
Fair value of consideration 1,320
Plus fair value of NCI at acquisition 330
Less net acquisition – fair value of
Assets acquired & liability:
Share capital 1,200
Retained Earning 190
Fair value adj at acquisition 140 (1,530)
Goodwill 120
2. Group structure
960 million
 100
1,200 million 80%
3. Retained earnings:
As per question 1,160 424
Adjustment (unrealised profit) (24)
Pre-acquisition retained earnings (190)
234
Group share of post-acquisition retained earnings:
(80% x 234) 187.2
1,323.2

© Emile Woolf International 145 The Institute of Chartered Accountants of Pakistan


Financial accounting and reporting II

4. Non-controlling interest: Rs. million


Fair value of NCI at acquisition 330.0
Plus NCI’s share of post-acquisition
retained earnings (20% x 234) 46.8
376.8
Alternative workings:

(W1) Fair value adjustment:


Dr: Consolidated land & building Rs. 140 million
Cr: Revaluation reserve Rs. 140 million

(W2) Consolidation Schedule


Bliss Ltd Rs. ’M Bradley in NCI Post –
Bliss 80% 40% Acq
(W3)
Rupees in millions
Ordinary share capital 1,200 960 240
Revaluation Res. (W1) 140 112 28
Retained earnings 424 152 84.8 187
────
Net assets acquired 1,224
Cost of acquisition (1,320)
Goodwill (partial value) (96)
Goodwill attribute to NCI (W5) (24) 24
────
Goodwill (fair value) ( 120)
────
Unrealised profit on inventory - (24)
NCI (fair value) 376.8
────
Retained earnings of Bradley Ltd 1,160
Consolidated retained earnings 1,323

960𝑚 𝑠ℎ𝑎𝑟𝑒𝑠
(W3) Bradley in Bliss = × 100 = 80%
1,200𝑚 𝑠ℎ𝑎𝑟𝑒𝑠

(W4) Bradley’s share of Bliss’s pre-acquisition retained earnings = 80% x


= 80% x Rs. 190m = Rs. 152m

(W5) Goodwill attribute to NCI Rs. m


Fair value of NCI @ date of acquisition 330
Less: fair value of net assets attributable to
NCI (20% x (1,200 + 140 + Rs. 190m)) 306
Goodwill attributable to NCI 24

© Emile Woolf International 146 The Institute of Chartered Accountants of Pakistan


Answers

CHAPTER 5 – CONSOLIDATED ACCOUNTS: STATEMENTS OF COMPREHENSIVE


INCOME
5.1 HARRY
Consolidated statement of profit or loss
for the year ended 31 December 2015
Rs.000

Revenue 1,410
Cost of sales (733)
——–
Gross profit 677
Distribution costs (90)
Administrative expenses (100)
——–
Operating profit 487
Investment income 9
Finance costs (22)
——–
Profit before tax 474
Income tax expense (165)
——–
Profit after tax 309
Non-controlling interest (W3) (15)
——–
Profit 294
——–
Movement on consolidated retained earnings
for the year ended 31 December 2015
Retained earnings at 1 January 2014 (W4) 127
Retained earnings for the year 294
Dividends (50)
——–
Retained earnings at 31 December 2015 (W5) 371
——–
WORKINGS

(1) Group structure


Harry

75%

Sally

© Emile Woolf International 147 The Institute of Chartered Accountants of Pakistan


Financial accounting and reporting II

(2) Consolidated statement of profit or loss


Harry Sally Adj Consol
Rs.000 Rs.000 Rs.000 Rs.000
Revenue 1,120 390 (100) 1,410
C of S – per Q (610) (220) 100
– PURP (3) – – (733)
Distribution costs (50) (40) (90)
Administrative expenses (55) (45) (100)
Investment income (20 – 15) 5 4 9
Interest payable (18) (4) (22)
Tax (140) (25) (165)
—–
PAT 60
—–
(3) Non-controlling interest Rs.000
25%  60,000 (W1) or as per PAT in question 15
——
(4) Reserves brought forward
Harry 100
Sally (75%  (45 – 9)) 27
——
127
——
(5) Reserves carried forward (proof)
Harry 317
PURP (3)
Sally (75%  (85 – 9)) 57
——
371
——
(6) Inter-company dividend
Payable by Sally 20
—–
Receivable by Harry (75%  20) 15
—–
5.2 HORNY
Consolidated statement of profit or loss
for the year ended 31 December 2015
Rs.000
Revenue 362,000
Cost of sales (169,050)
———–
Gross profit 192,950
Operating costs (93,817)
———–
Operating profit 99,133
Investment income 13,100
Negative goodwill 3,800
———–
Profit before tax 116,033
Income tax (48,400)
———–
Profit after tax 67,633
Non-controlling interest (W3) (2,996)
———–
Profit 64,637
———–

© Emile Woolf International 148 The Institute of Chartered Accountants of Pakistan


Answers

Movement on consolidated retained earnings for the year ended


31 December 2015
Rs.000
Retained earnings at 1 January 2015 80,200
Retained profit for the year 64,637
Dividend (20,000)
———–
Retained earnings at 31 December 2015 124,837
———–
WORKINGS
(1) Group structure
Horny

75% (acq 31 August 2005)

Smooth
(2) Consolidation schedule
Horny Smooth Adj Consol
4
12
Rs.000 Rs.000 Rs.000 Rs.000
Revenue 304,900 65,100 (8,000) 362,000
Cost of sales (144,200) (32,850) 8,000 (169,050)

Operating costs (76,450) (17,367) (93,817)


Investment income
of H 10,500
of S (all of it) 2,600 13,100
Tax (42,900) (5,500) (48,400)
———
PAT 11,983
———
(3) Non-controlling interest @ 25%
——–
= 2,996
——–

(4) Consolidated retained earnings carried forward - proof


Rs.000

Horny 112,050
Simpson (11,983 – 2,996) 8,987
Negative goodwill 3,800
———
124,837
———

© Emile Woolf International 149 The Institute of Chartered Accountants of Pakistan


Financial accounting and reporting II

5.3 HANKS
Consolidated statement of financial position as at 31 December 2015
Assets Rs.000 Rs.000
Non-current assets
Property, plant and equipment
(32,000 + 25,000 + 20,000 + 6,000) 83,000
Goodwill 4,500
———–
87,500
Current assets
Cash at bank and in hand (9,500 + 2,000 + 4,000) 15,500
Receivables (20,000 + 8,000 + 17,000) 45,000
Inventory (30,000 + 18,000 + 18,000 – 2,100) 63,900
———–
124,400
———–
Total assets 211,900
═════
Equity and liabilities
Share capital 40,000
Share premium account 6,500
Retained earnings (W5) 88,300
———–
134,800
Non-controlling interest (W4) 28,100

Current liabilities
Trade payables (23,500 + 6,000 + 17,000) 46,500
Proposed dividends – to minority shareholders (2,500 – 2,000) 500
– to Hanks’s shareholders 2,000
———– 49,000
———–
Total equity and liabilities 211,900
═════
Consolidated statement of profit or loss for the
year ended 31 December 2015
Rs.000
Revenue (W6) 310,000
Cost of sales (W6) (159,100)
———–
Gross profit 150,900
Distribution costs (W6) (51,000)
Administrative expenses (W6) (29,500)
———–
Profit before taxation 70,400
Tax (W6) (24,000)
———–
Profit after taxation 46,400
Non-controlling interest (W6) (9,200)
———–
Profit 37,200
═════

© Emile Woolf International 150 The Institute of Chartered Accountants of Pakistan


Answers

Statement of movements on reserves for the year ended 31 December 2015


Share
Share premium Retained
Capital account earnings Total
Rs.000 Rs.000 Rs.000

At 1 January 2015 40,000 6,500 53,100 (W7) 99,600


Profit for the year – 37,200 37,200
Dividends (proposed) (2,000) (2,000)
——–— –––––– ––––––– –––––––
At 31 December 2015 40,000 6,500 88,300 134,800
═════ ═════ ═════ ═════
WORKINGS
(1) Group structure
Hanks

80% 60%

Streep Scott
(2) Net assets
Streep

Reporting Date of Post-


date acquisition acquisition
Rs.000 Rs.000
Share capital 10,000 10,000
Retained earnings
Per question 37,000
Proposed dividend (2,500)
34,500 7,500 27,000
44,500 17,500

Scott

Reporting Date of Post-


date acquisition acquisition
Rs.000 Rs.000
Share capital 15,000 15,000
Retained earnings 27,000 3,000 24,000
Revaluation reserve 6,000 6,000
48,000 24,000

(3) Goodwill on Streep


Rs.000
Cost of shares 20,500
Net assets acquired (80%  17,500) (W2) (14,000)
———
6,500
———

© Emile Woolf International 151 The Institute of Chartered Accountants of Pakistan


Financial accounting and reporting II

Rs.000
Of which:
Written off by start of the year (6,500 – 5,000) 1,500
Written off by end of the year (6,500 – 4,500) 2,000
———
Recognised as impairment during the year (balancing figure) 500
———
Goodwill on Scott
Cost of shares 13,000
Net assets acquired (60%  24,000 (W2)) (14,400)
———
(1,400)
———
(4) Non-controlling interest
Streep (20%  44,500 (W2)) 8,900
Scott (40%  48,000 (W2)) 19,200
———
28,100
———
(5) Consolidated retained earnings c/f Rs.000
Hanks 55,000
Dividend receivable from Streep (80% of 2,500) 2,000
Proposed dividend (2,000)
Streep (80%  27,000 (W2)) 21,600
Scott (60%  24,000 (W2)) 14,400
30
PURP ((5,200 + 3,900)  130 ) (2,100)

Goodwill impairment – Streep (2,000)


Negative goodwill – Scott 1,400
———
88,300
———
(6) Consolidation schedule
Hanks Streep Scott Adj Consol
Rs.000 Rs.000 Rs.000 Rs.000 Rs.000

Sales revenue 125,000 117,000 82,000 (14,000) 310,000


C of S – per Q (65,000) (64,000) (42,000) 14,000
– PURP (W5) (2,100)
(159,100)
Distrib (21,000) (14,000) (16,000)
(51,000)
Admin (14,000) (8,000) (7,000) (500)
(29,500)
Tax (10,000) (9,000) (5,000) (24,000)
——— ———
PAT 22,000 12,000
Non-controlling interest in
profit after tax @20% @40%
——– ——–
4,400 + 4,800 = 9,200
——– ——– ——–

© Emile Woolf International 152 The Institute of Chartered Accountants of Pakistan


Answers

(7) Consolidated retained earnings b/f Rs.000


Hanks 40,000
Share of post acquisition profits of Streep (80% (15,000 – 7,500)) 6,000
Share of post acquisition profits of Scott (60% (15,000 – 3,000)) 7,200
Goodwill impairment - Streep (1,500)
Negative goodwill credited 1,400
———
53,100
———
5.4 OSCAR LIMITED
For the year ended 31 December 2015

Rs. in million
Sales [835+(645×8÷12)]-(60×8÷12)–(30×8÷12)–3.2(W-1) 1,201.80
Cost of sales [525+(396×8÷12)]-(60×8÷12)–(30×8÷12)+4+1 (734.00)
Gross profit 467.80
Operating expenses [115+(102×8÷12)]+2-0.1(W-1) (184.90)
Profit before tax 282.90
Tax expense [65+(48×8÷12)] (97.00)
Profit after tax 185.90
Other comprehensive income -
Total comprehensive income 185.90

Total comprehensive income attributable to:


Owners of the parent – balancing figure 173.94
Non-controlling interest (W-4) 11.96
185.90
As on 31 December 2015
Rs. in million
Property, plant and equipment (W-1) 736.90
Brand/Intangibles (45-2) 43.00
Goodwill (W-3) 46.40
Stock-in-trade (125+115-4-1) 235.00
Trade receivables (140+125-30) 235.00
Cash and bank (105+103+30) 238.00
Total Assets 1,534.00
Share capital (@ Rs. 10 each) 600.00
Share premium 150.00
Consolidated retained earnings (265+173.94) 438.94
Non-controlling interest (W-4) 125.36
Current liabilities (115+105) 220.00
Total Equity and Liabilities 1,534.30

© Emile Woolf International 153 The Institute of Chartered Accountants of Pakistan


Financial accounting and reporting II

Workings:
W-1: Property, plant and equipment

Rs. in million

OL and UL (390 + 350) 740

Reversal of gain on disposal [24 – (26÷10×8)] (3.2)

Reversal of depreciation on gain amount 0.1


(3.2÷8×0.25)

736.9

W-2: Adjustment for management services


No adjustment for management services in the consolidated financial statements.
W-3: Computation of Goodwill

------ Rs. in million ------

Cost of investment 500

Less : Share in FV of UL's net assets at acquisition

Share capital 250

Share Premium 60

Retained earnings (179 + 99 × 4 ÷ 12) 212

FV of brand 45

(567)

Share in FV of UL's net assets at acquisition (567 × 0.8) (453.60)

Goodwill 46.40

W-4: Non-controlling interest

Rs. in million

UL's post acquisition profit (99 - 8 ÷ 12) 66.00

Gain on sale of machine to OL (W-1) (3.20)

Inventory held by OL (1.00)

Amortization of brand (2.00)

59.80

Total comprehensive income attributable to NCI (59.80 × 0.2) 11.96

NCI's share of net assets at acquisition date (20% of 567) 113.40

NCI share of net assets at consolidation date 125.36

© Emile Woolf International 154 The Institute of Chartered Accountants of Pakistan


Answers

5.5 PRESENT LIMITED AND FUTURE LIMITED


(a) Present Limited
Consolidated statement of profit or loss
For the year ended 30 June 2017

Rs. in million

Sales [2,060+(1,524×10/12)–(100×60%)] 3,270.00

Less: Cost of sales (W-1) (1,949.00)

Gross profit 1,321.00

Less: Selling and administrative expenses (W-2) (584.55)

Net profit before other income 736.45

Other income:

Investment income [190–(2600×65%×5%)] 105.50

Gain on disposal of fixed assets [35+(42–30)] 47.00

Total other income 152.50

Net profit before tax 888.95

Less: Taxation [80+(60×10/12)] (130.00)

Net profit after tax 758.95

Net profit after tax attributable to:

Owners of the parent - balancing figure 662.19

Non-controlling interest (W-3) 96.76

758.95

W-1: Cost of sales Rs. in million

PL 1,300

FL (846×10/12) 705

FL’s sales to PL (100×60%) (60)

Increase in dep. on manufacturing plant sold by PL to FL [42–30)/6×6/12)] 1

Unrealized profit included in PL’s closing stock


[(100×60%×20%)÷1.3333×0.3333] 3

1,949

© Emile Woolf International 155 The Institute of Chartered Accountants of Pakistan


Financial accounting and reporting II

W-2: Selling and administrative expanse Rs. in million


PL 350.00
FL (225×10/12) 187.50
Amortization of brand (90÷10×10/12)) 7.50
Impairment of goodwill [395.50(W-2.1)×10%] 39.55

584.55

W-2.1: Cost of sales Rs. in million

Cost of investments [(100×16)+900] 2,500.00

Fair value of NCI [260×35%×12] 1,092.00

3,592.00
Less : Fair value of net assets acquired [2,600+5 3,196.50
Goodwill 395.50

W-3: Non-controlling interest Rs. in million

FL's profit for ten months [(1,524–846–225–60)×10/12×35%] 114.63

Unrealized profit in closing stock [3×35%] (1.05)

Increase in depreciation for machine sold by PL [1×35%] (0.35)

Amortization of brand (7.5×35%) (2.63)

Impairment of goodwill (39.55×35%) (13.84)

96.76

(b) Consolidated retained earnings Rs. in million

PL's retained earning – 1 July 2016


[1996–555(i.e. 2,060–1,300–350+190+35–80)+(3500×5%)] 1,616.00

PL's dividend (3500×5%) (175.00)

Consolidated income attributable to parent (part a) 662.19

2,103.19

Non-controlling interest Rs. in million

FV of NCI at acquisition [260×35%×12] 1,092.00

NCI's share in FL' s dividend [2600×5%×35%] (45.50)

NCI for the year (part a) 96.76

1,143.26

© Emile Woolf International 156 The Institute of Chartered Accountants of Pakistan


Answers

5.6 FATIMA LIMTED AND ALI LIMITED


Statement of Financial Position
FL AIL Adj. Cons.
PPE 16,740.00 4,680.00 (32.40) 21,387.60
Goodwill 1,860.00
Other Investments 20,100.00 - 20,100.00
Stock in trade 14,460.00 5,680.00 (570.00) 19,570.00
Accounts receivables 6,240.00 6,580.00 (800.00) 12,020.00
Cash and bank balances 4,920.00 2,700.00 (600.00) 7,020.00
62,460.00 19,640.00 (2,002.40) 81,957.60

Share capital 30,000.00 30,000.00


GRE 39,115.60 39,115.60
NCI 3,442.00 3,442.00
Accounts payable 2,760.00 1,440.00 (800.00) 3,400.00
Dividend payable 6,000.00 - - 6,000.00
38,760.00 1,440.00 41,757.60 81,957.60
-
Statement of Comprehensive Income Statement
FL AIL Adj. Cons.
Sales revenue 57,600.00 33,800.00 (7,800.00) 83,600.00
Cost of Sales (49,200.00) (21,000.00) 7,251.60 (62,948.40)
Gross Profit 8,400.00 12,800.00 (548.40) 20,651.60
Operating expenses (3,600.00) (5,400.00) (9,000.00)
Gain on sale of fixed assets 540.00 - (54.00) 486.00
Dividend income 1,080.00 - (480.00) 600.00
Profit before tax 6,420.00 7,400.00 (1,082.40) 12,737.60
Taxation - - - -
Profit after tax 6,420.00 7,400.00 (1,082.40) 12,737.60

Profit attributable to:


P 6,420.00 5,998.00 (1,082.40) 11,335.60
NCI 1,402.00 - 1,402.00
6,420.00 (1,082.40) 12,737.60

Group Structure AL
Subsidiary
FL 80%
NCI 20%

© Emile Woolf International 157 The Institute of Chartered Accountants of Pakistan


Financial accounting and reporting II

Net assets of subsidiaries AL


At Acq. At Rep.
Share capital 6,000.00 6,000.00
Retained earnings - Opening 4,800.00 4,800.00
Profit for the year - 7,400.00
Dividend paid - (600.00)
Unrealized profit on stock-in-trade - (390.00)
10,800.00 17,210.00

Goodwill AL
Cost of investment 10,500.00
NCI share 2,160.00
12,660.00
Less: NA at fair value (10,800.00)
Goodwill 1,860.00

NCI
At acquisition 2,160.00
Post - acquisition profits - AL 1,282.00
3,442.00

Group retained earnings


P's retained earnings 33,780.00
Profit for the year 6,420.00
Post - acquisition profits - AL 5,128.00
Unrealized profit (180.00)
Gain on sale (54.00)
Reversal of excess depreciation on gain 21.60
Dividend payable (6,000.00)
39,115.60

NCI in income statement AIL


Profit after tax 7,400.00
Less:
Unrealized profit on stock in trade (390.00)
7,010.00
NCI% of share 1,402.00

© Emile Woolf International 158 The Institute of Chartered Accountants of Pakistan


Answers

CHAPTER 6 – IAS 28: INVESTMENT IN ASSOCIATES AND JOINT VENTURES


6.1 HELIUM
Consolidated statement of financial position as at 31 December 2016
Rs.000
Assets
Non-current assets
Property, plant and equipment 500
Interest in associate (W6) 51
Goodwill 15
Current assets 605
———
Total assets 1,171
———
Equity and liabilities
Capital and reserves
Share capital 100
Retained earnings (W5) 737
———
837
Non-controlling interest 84
Long-term liabilities 250
———
Total equity and liabilities 1,171
———

Workings

(1) Group structure

Helium

30%
60%

Arsenic
Sulphur

(2) Net assets


Sulphur
Balance Post
sheet date Acquisition acquisition
Rs.000 Rs.000 Rs.000
Share capital 30 30 –
Retained earnings 180 70 110
—— ——
210 100
—— ——

© Emile Woolf International 159 The Institute of Chartered Accountants of Pakistan


Financial accounting and reporting II

(3) Goodwill
Sulphuric
Rs.000

Cost of investment 75
Share of net assets acquired
(60%  100 (W2)) (60)
——
15
——
(4) Non-controlling interest
Sulphur (40%  210) 84
——
(5) Retained earnings
Helium 650
Sulphur (60%  110 (W2)) 66
Arsenic (30%  (100 − 30)) 21
——
737
——
(6) Investment in associate
Cost 30
Share of post-acquisition profit (30%  (100 − 30)) 21
——
51
——
6.2 HAMACHI LTD
(a)

Hamachi Ltd
Consolidated statement of financial position as at 31 March 2016
Rs.000 Rs.000
Non-current assets
Property, plant and equipment (8,050 + 3,600) 11,650
Goodwill (W2) 702
Licence (180 – 60) (W3) 120
12,472
Investments
Associate (W6) 717
Others (4,000 + 910 – 3,240 – 630 + 120 FV) 1,160
1,877
14,349
Current assets
Inventory (830 + 340) 1,170
Accounts receivable (520 + 290 – 40) 770
Bank (240 + 40) 280
2,220
Total assets 16,569
Equity and liabilities
Equity attributable to equity holders of the parent:
Ordinary shares of Rs. 1 each 5,000
Retained earnings (W5) 8,415
13,415
Non-controlling interest (W4) 374
13,789

© Emile Woolf International 160 The Institute of Chartered Accountants of Pakistan


Answers

Hamachi Ltd
Consolidated statement of financial position as at 31 March 2016
Rs.000 Rs.000
Non-current liabilities

10% Loan notes (500 + 240) 740

Current liabilities

Accounts payable (420 + 960) 1,380

Taxation (220 + 250) 470

Overdraft 190

2,040

Total equity and liabilities 16,569

Workings
(W1) Net assets in subsidiary

At At end of
acquisition reporting period

Rs.000 Rs.000

Share capital 1,200 1,200

Retained earnings 800 2,300

Fair value adjustment:

Investment property 120 120

Licence 180 180

Amortisation of licence 180/6 x 2yrs (60)

2,300 3,740

(W2) Goodwill

Rs.000

Cost of investment (Rs. 3  1,200  90%) 3,240

Net assets acquired (90%  2,300) (W1) 2,070

Goodwill 1,170

Less impairment (468)

702

© Emile Woolf International 161 The Institute of Chartered Accountants of Pakistan


Financial accounting and reporting II

(W3) Unrealised profit in inventory


((2/3 × 65,000) × 30/130) × 30% = Rs. 3,000
Parent sells to associate, therefore reduce group retained earnings and Investment
in associate

(W4) Non-controlling interest


10%  3,740 = Rs. 374

(W5) Retained earnings


Rs.000
Hamachi Ltd 7,500
Saba Ltd – group share post-acquisition

90%  (3,740 – 2,300) 1,296

Anogo Ltd – group share post-acquisition

30%  (600  6/12) 90

Unrealised profit (W3) (3)


Less impairment (468)

8,415

(W6) Investment in associate

Rs.000
Investment at cost 630
Post-acquisition profit (30%  (600  1/2)) 90
Unrealised profit in inventory (3)
717

(b) IAS 28 Investments in Associates and Joint Ventures defines associates. In order for an
investment to be classified as an investment in an associate the investor must have
‘significant influence’ over the investee. Significant influence is presumed to exist where
there is a holding of 20% or more of the voting power unless the investor can clearly
demonstrate that this is not the case. Conversely a holding of less than 20% is presumed
not to be an associate, unless it can be clearly demonstrated that the investor can exercise
significant influence. The voting rights can be held directly or through subsidiaries.
IAS 28 says that a majority holding by one investor does not preclude another investor
having significant influence. An investing company owning a majority holding in another
company normally has control over the investee and would thus class it as a subsidiary. In
normal circumstances it is difficult to see how a company could be controlled by one entity
and be significantly influenced by a different entity unless ‘control’ was passive. The 20%
test is not definitive and the following other evidence should be considered.
Does the investing company:
 have representation on the Board of the investee?
 participate in the policy making processes (operational and financial); have material
transactions with the investee?
 interchange managerial personnel with the investee; or provide technical expertise to
the investee?

© Emile Woolf International 162 The Institute of Chartered Accountants of Pakistan


Answers

6.3 HIDE
Hide
Consolidated statement of profit or loss for the year ended 30 June 2016
Rs.000
Revenue 15,131
Cost of sales and expenses (13,580)
———
Operating profit before tax 1,551
Tax (736)
———
Profit after tax 815
Share of profit of associate (30% of 594) 178
———
Profit for the year 993
———

Profit for the year attributable to members of Hide 963


Non-controlling interest (W2) 30
———
Profit for the year 993
———

Workings

(1) Group structure

Hide

30%
80%
Arrive

Seek

(2) Consolidation schedule


Hide Seek (5/12) Adjustment Total
Rs. 000 Rs. 000 Rs. 000 Rs. 000
Revenue 12,614 2,567 (50) 15,131
Cost of sales
Per question (11,318) (2,302) 50
Unrealised profit
50  (25/125) (10) (13,580)

Tax (621) (115) (736)


Profit for the year 150
Non-controlling interest (%) 20%
Non-controlling interest (Rs. 000) 30

© Emile Woolf International 163 The Institute of Chartered Accountants of Pakistan


Financial accounting and reporting II

6.4 HARK, SPARK AND ARK


Hark Group
Consolidated statement of financial position as at 31 March 2016
Rs.000 Rs.000
Non-current assets
Property, plant and equipment (working 1) 90,200
Goodwill (working 4) 23,000
Investment in associate (working 6) 9,500
Other investments 650
123,350
Current assets (working 5) 24,300
Total assets 147,650
Equity and liabilities
Equity shares of Rs. 1 each (working 3) 21,000
Share premium (working 3) 42,000
Retained earnings (working 8) 43,730
85,730
106,730
Non-controlling interests (working 7) 7,420
Total equity 114,150
Non-current liabilities
Deferred consideration for Spark shares 5,500
6% loan notes 10,000
7% loan notes 6,000
21,500
Current liabilities: 7,000 + 5,000 12,000
Total equity and liabilities 147,650
Workings
1 Property, plant and equipment (PPE)
Rs.000 Rs.000
Hark 60,000
Spark 31,000
Profit on transfer of machines (3 million – 2 million) 1,000
Less: Depreciation on this amount in accounts of Spark
(1,000/5 years) (200)
Unrealised profit in machines (800)
PPE in consolidated statement of financial position 90,200

2 Deferred consideration
The present value of the deferred consideration at 1 April 2015 is Rs. 6.05 million 
1/(1.10)2 = Rs. 5 million.
During the year to 31 March 2016 there is a finance charge of 10% (= Rs. 500,000) on this
amount, reducing the parent’s share of the consolidated profit.
The deferred consideration at 31 March 2016 is Rs. 5 million + Rs. 500,000 = Rs.
5,500,000. This is payable in just over 12 months and is included in the consolidated
statement of financial position as a non-current liability.

© Emile Woolf International 164 The Institute of Chartered Accountants of Pakistan


Answers

3 Share issues
The share issues to acquire the shares in Spark and Ark are not recorded in the summary
statement of financial position of Hark (as stated in the question).
Share Share
Total
capital premium
To acquire the shares in Spark Rs.000 Rs.000 Rs.000
Hark shares issued: (4 million at Rs. 9) 36,000 4,000 32,000
To acquire the shares in Ark
Hark shares issued: (1 million at Rs. 9) 9,000 1,000 8,000
Increase in share capital and share premium
of Hark 5,000 40,000
In summary statement of financial position 16,000 2,000
In consolidated statement of financial
position 21,000 42,000

4 Goodwill
Hark has acquired 4 million/5 million = 80% of the shares of Spark.
At 1 April 2015 the fair value of the net assets of Spark was (share capital plus reserves) =
Rs.(5 + 4 + 16) million = Rs. 25 million
Rs.000
Purchase consideration paid by the parent company
Issue of 4 million shares at Rs. 9 36,000
Deferred consideration 5,000
41,000

Fair value of parent company share of net assets


(80%  Rs. 25 million) 20,000
Purchased goodwill attributable to parent 21,000

Rs.000
Fair value of NCI at acquisition date (1 million shares  Rs. 7) 7,000
NCI share of net assets at this date (20%  Rs. 25 million) 5,000
Purchased goodwill attributable to NCI 2,000
There has been no impairment of goodwill during the year.
Rs.000
Purchased goodwill attributable to parent 21,000
Goodwill attributable to NCI 2,000
Total goodwill in consolidated statement of financial position 23,000
Alternatively, total goodwill could be calculated as follows:
Rs.000
Purchase consideration paid by the parent company 41,000
(see above)
Fair value of NCI at acquisition date 7,000
48,000
Net assets of the subsidiary at the acquisition date 25,000
(at fair value)
Total goodwill (parent and NCI) 23,000

© Emile Woolf International 165 The Institute of Chartered Accountants of Pakistan


Financial accounting and reporting II

5 Current assets
The cost of the goods sold by Spark to Hark was Rs. 3,600,000  100/150 = Rs. 2,400,000
and the profit was Rs. 1,200,000.
Since 75% of these goods are in closing inventory, the unrealised profit on intra-group sales
is 75%  Rs. 1,200,000 = Rs. 900,000. Current assets in the consolidated statement of
financial position (inventory) should be reduced by this amount.
The question states that the transaction costs of the acquisition of Spark have not yet been
recorded. These costs reduce the consolidated profit, and also (presumably) reduce the
current assets of Hark.
Current assets on consolidation Rs.000
Hark 18,200
Spark 8,000
Less: unrealised profit in closing inventory (900)
Less: expenses of acquisition of Spark (1,000)
Current assets in consolidated statement of financial position 24,300

6 Investment in associate (Ark)


Since Hark owns 25% of the equity of Ark, it is assumed that Ark is an associated entity.
Rs.000
Cost of investment: 25%  6 million shares  Rs. 6 9,000
Share of post-acquisition retained profit: 25%  Rs. 2 million 500
9,500
7 Non-controlling interests
Rs.000
Share of net assets of Spark at 31 March 2016 (20%  Rs. 28 million) 5,600
Goodwill attributable to NCI (working 4) 2,000
7,600
NCI share of unrealised profit in inventory (20%  Rs. 900,000) (180)
NCI at 31 March 2016: fair value method 7,420

8 Consolidated retained earnings


Rs.000 Rs.000
Hark retained earnings (36,000 + 8,000) 44,000
Spark
Profit for year ended 31 March 2016 3,000
Unrealised profit in closing inventory (900)
2,100
Parent company share (80%) 1,680
Share of post-acquisition retained profits of Ark 500
(25%  Rs. 2 million)
Costs of acquisition of Spark (expensed) (1,000)
Additional finance costs: deferred consideration (500)
Unrealised profit in machines (working 1) (800)
Loss on other (800 – 650) (150)
Consolidated retained earnings at 31 March 2016 43,730

© Emile Woolf International 166 The Institute of Chartered Accountants of Pakistan


Answers

6.5 P, S AND A
P Group
Consolidated statement of financial position as at 31 December Year 5
Assets
Non-current assets Rs.
Property, plant and equipment (450,000 + 240,000) 690,000
Goodwill (W3) 45,000
Investment in associates (W5) 168,800

903,800
Current assets
Inventory (70,000 + 90,000 – 10,000) 150,000
Other current assets (20,000 + 110,000 + 130,000) 260,000

Total assets 1,313,800

Equity and liabilities


Equity
Share capital 100,000
Share premium 160,000
Consolidated accumulated profits (W6) 711,300

Attributable to equity holders of the parent 971,300


Non-controlling interest in S (W4) 102,500

Total equity 1,073,800


Long-term liabilities (40,000 + 20,000) 60,000
Current liabilities (100,000 + 80,000) 180,000

Total equity and liabilities 1,313,800

Workings
P owns 75% of the equity of S and 30% of the equity of A. Therefore S is a subsidiary and A is
an associate.
W1: Net assets summary
Calculate the net assets of S and A at the acquisition date and at the end of the reporting period.
At this stage, make any fair value adjustments and eliminate the unrealised profit in inventory.
At date of At date of Post-
Net assets of S consolidation acquisition acquisition
Rs. Rs. Rs.
Equity shares 200,000 200,000 -
Share premium 80,000 80,000 -
Accumulated profits (per question) 140,000 60,000 80,000
410,000 340,000

© Emile Woolf International 167 The Institute of Chartered Accountants of Pakistan


Financial accounting and reporting II

W2: Unrealised profit on inter-company trading


Sale by S to P: Rs. 40,000 × 33.33/133.33 = Rs. 10,000.
Dr Cr
Consolidated accumulated profits (75%) 7,500
Non-controlling interest (25%) 2,500
Consolidated inventory 10,000
Sale by P to A: Rs. 16,000 × 33.33/133.33 = Rs. 4,000.
P’s share: Rs. 4,000  30% = Rs. 1,200
Dr Cr
Consolidated accumulated profits (75%) 1,200
Investment in associate 1,200
W3: Goodwill on acquisition of S
Rs.
Cost of the acquisition 320,000
Non-controlling interest at acquisition (25%  340,000 (W1)) 85,000
405,000
Less: Fair value of identifiable net assets at acquisition (W1) (340,000)
Goodwill at acquisition 65,000
Minus: Impairment to date (20,000)
Balance carried forward 45,000
W4: Non-controlling interest in S
Rs.
Non-controlling interest at acquisition (25%  340,000 (W1)) 85,000
Share of post-acquisition profits (25%  80,000 (W1)) 20,000
Unrealised profit (W2) (2,500)
102,500
W5: Investment in associate
Rs.
Investment at cost 140,000
P’s share of post-acquisition accumulated profits 30,000
(30%  (250,000 – 150,000)
Unrealised profit (W2) (1,200)
168,800
W6: Consolidated accumulated profits.
Rs.
Accumulated profits of P 650,000
P’s share of post-acquisition profits of S
(75% × Rs. 70,000 (W2)) 60,000
Unrealised profit (sale by S to P (W3)) (7,500)
P’s share of post-acquisition accumulated profits (W5) 30,000
Unrealised profit (W2) (1,200)
Impairment of goodwill (20,000)
Consolidated accumulated profits 711,300

© Emile Woolf International 168 The Institute of Chartered Accountants of Pakistan


Answers

6.6 BL, ML AND ZL


Statement of Financial Position
BL ML Adj. Consolidated
Goodwill 75.00 75.00
PPE 1,012.00 920.00 60.00 1,992.00
Intangible asset - 350.00 (87.00) 263.00
Investment in ZL 219.75 - 219.75
Stock in trade 620.00 1,460.00 (42.00) 2,038.00
Trade and other receivables 950.00 529.00 - 1,479.00
Cash and bank balances 900.00 510.00 - 1,410.00
3,701.75 3,769.00 6.00 7,476.75

Share capital 1,000.00 1,000.00


Share premium 200.00 200.00
GRE 1,786.75 1,786.75
NCI 310.00 310.00
Current liabilities 1,880.00 2,300.00 4,180.00
5,176.75 2,300.00 - 7,476.75

Statement of Comprehensive Income Statement


BL ML Adj. Consolidated
Sales revenue 4,480.00 4,200.00 (1,300.00) 7,380.00
Cost of Sales (2,690.00) (2,940.00) 1,241.00 (4,389.00)
Gross Profit 1,790.00 1,260.00 (59.00) 2,991.00
Distribution & administrative cost (620.00) (290.00) (87.00) (997.00)
Finance cost (50.00) (80.00) - (130.00)
Dividend income 260.00 - (260.00) -
Share of profit from ZL 30.75 - - 30.75
Profit before tax 1,410.75 890.00 (406.00) 1,894.75
Taxation (330.00) (274.00) - (604.00)
Profit after tax 1,080.75 616.00 (406.00) 1,290.75

Profit attributable to:


P 1,080.75 522.00 (406.00) 1,196.75
NCI 94.00 - 94.00
1,080.75 616.00 (406.00) 1,290.75

© Emile Woolf International 169 The Institute of Chartered Accountants of Pakistan


Financial accounting and reporting II

W-1 Group Structure ML ZL


Subsidiary Associate
BL 80% 25%
NCI 20%

W-2 Net assets of


subsidiaries ML ZL
Post Acq. Post Acq.
At Acq. At Rep. Profit At Acq. At Rep. Profit

Share capital 400.00 400.00 - 220.00 220.00 -

Share premium 140.00 140.00 - 83.00 83.00 -

Retained earnings 160.00 929.00 769.00 269.00 361.00 92.00

Fair value adjustment 100.00 100.00 - 80.00 80.00 -


Fair value depreciation
adjustment - (40.00) (40.00) - -
Intangible asset
expensed out (87.00) (87.00) - -
Unrealized profit on
stock - (42.00) (42.00) (25.00) (25.00)
800.00 1,400.00 600.00 652.00 719.00 67.00

W-3 Goodwill ML ZL
Cost of investment 765.00 203.00
NCI at fair value 210.00 -
975.00 203.00
Less: NA at fair value 800.00 163.00
Goodwill 175.00 40.00
Less: Impairment (100.00) -
75.00 40.00

W-4 NCI ML
At acquisition 210.00
Post - acquisition profits 120.00
Less: Impairment (20.00)
310.00

© Emile Woolf International 170 The Institute of Chartered Accountants of Pakistan


Answers

W-5 Group retained earnings


P's retained earnings 1,370.00
Post - acquisition profits - ML 480.00
Impairment loss (80.00)
Post - acquisition profits - ZL 16.75
1,786.75

W-6 Investment in Associate


Cost of investment 203.00
Share of profit/loss 16.75
219.75

W-7 NCI in Comprehensive income statement


Profit afte tax - ML 616.00
Less:
FV depreciation adjustment (10.00)
Intangible asset expensed out (87.00)
Unrealized profit on stock on closing stock (42.00)
Unrealized profit on stock on closing stock 18.00
Goodwill impairment (25.00)
470.00
NCI% of share 94.00

6.7 MANTO LIMITED AND HALI LIMITED


Consolidated statement of financial position
As on 31 December 2012
Rs. in million
ASSETS
Non-current assets
Property, plant and equipment W.1 5,546.90
Goodwill W.2 [110 – (37.5 × 80%)] 80.00
Investment in associates W.3 251.47
5,878.37
Current assets (5,480 + 400) 5,880.00
11,758.37

© Emile Woolf International 171 The Institute of Chartered Accountants of Pakistan


Financial accounting and reporting II

EQUITY AND LIABILITIES Rs. in million


Equity attributable to owners of QL
Ordinary shares capital 6,000.00
Shares to be issued (purchase consideration payable) (4 × 15) 60.00
Retained earnings W.4 2,828.99
8,888.99
Non-controlling interest [(500 + 100 – 3.1) × 20%] 119.38
9,008.37
Current liabilities (2,400 + 350) 2,750.00
11,758.37

W.1 Property, plant and equipment


QL and ML (5,000 + 550) 5,550.00
Unrealised gain on purchase of the machine from ML
- Unrealised gain as on 01-10-2012 [24 – (26÷10×8)] (3.2)
- Realised gain for 1-10-2012 to 31-12-2012 [3.2 × 0.25÷8)] 0.1
Unrealsied gain on 31-12-2012 (3.10)
5,546.90

W.2 Goodwill in ML and its impairment


Goodwill at the date of acquisition [630 – (80% × (500 + 150)] 110.00
Goodwill impairment:
Carrying value of ML's net assets on 31-12-2012 (950-350) 600.00
Gross-up of goodwill in ML (CGU) (110 ÷ 80%) 137.50
737.50
Estimated recoverable amount of ML on 31-12-2012 (700.00)
Total impairment 37.50

W.3 Investment in associates (HL)


Cost of investment [190 + (4 × 15)] 250.00
HL’s post acquisition profits [(240 – 224) × 40%] 6.40
Additional depreciation on fair value of HL’s
building exceeding its carrying amount [(28 ÷ 7 ÷ 12) × 40% ] (0.13)
Unrealized profit on inventories sold to HL [(52×30 ÷ 130) × 40%] (4.80)
251.47
W.4 Retained earnings QL
QL balance of retained earnings 2,900.00

© Emile Woolf International 172 The Institute of Chartered Accountants of Pakistan


Answers

Subsidiary ML:

ML post acquisition loss [(100 – 150) × 0.8] (40.00)

ML goodwill impairment W.2 (37.5 × 80%) (30.00)

Unrealized gain on purchase of the machine from ML W.1 (3.1 × 80%) (2.48)

Associate HL:

HL post acquisition profit W.3 6.40

QL’s share of additional depreciation on fair value


of HL’s building exceeding its carrying amount W.3 (0.13)

Unrealized profit on inventories sold to HL W.3 (4.80)

2,828.99

© Emile Woolf International 173 The Institute of Chartered Accountants of Pakistan


Financial accounting and reporting II

CHAPTER 7 – IAS 8: ACCOUNTING POLICIES, CHANGES IN ACCOUNTING


ESTIMATES AND ERRORS
7.1 WONDER LIMITED
2015 2014
(Restated)
Rs.m Rs.m
a) Wonder Limited
Extracts of Statement of financial position
For the year ended 30 June 2015
Property, plant and equipment 178.50 111.50
Retained earnings 158.65 95.05
Deferred tax liability 41.85 21.45
PPE: Year 2015: 189 - [20 - (20 × 10% × 1.75)] + [56/4 – 56/7] PPE: Year 2014: 130 - 18.5(Note X)
DTL: Year 2015: [(21.45 + (45 - 27) + {(6+2) × 30%}] DTL: Year 2014: 27 - 5.55 (Note X)

b) Wonder Limited
Extracts from the Statement of profit or loss for the year ended 30 June 2015
Profit before taxation 98.00 101.50
Taxation (34.40) (36.45)
Profit after taxation 63.60 65.05

PBT : Year 2015 : 90 + (20 × 10% ) + [(56/4) - (56/7)] PBT : Year 2014 : 120 - 18.5 (Note X)
Tax : Year 2015: 32 + [(6+2) × 30%] Tax : Year 2014 : 42 - 5.55 (Note X)

c) Wonder Limited
Extracts of statement of changes in equity for the year ended 30 June 2015

Retained
earnings
Rs.m
Balance as on 1 July 2013 (108-78) 30.00
Profit for the year ended 30 June 2014 (78 - 12.95 (Note X))- restated 65.05
Balance as at 30 June 2014 - restated 95.05
Profit for the year ended 30 June 2015 63.60
Balance as at 30-June 2015 158.65

d) Wonder Limited
Notes to the financial statements
For the year ended 31 December 2015
X Correction of error
During the year ended 30 June 2013, the repair works was erroneously debited to
machinery account. The effect of this error is as follows:

© Emile Woolf International 174 The Institute of Chartered Accountants of Pakistan


Answers

2014
Rs.m
Effect on the statement of profit or loss
(Increase) / decrease in expenses or losses
Repairs and maintenance (20.00)
Depreciation (20 × 10% × 9 ÷ 12) 1.50
Tax expenses (30% × (20-1.5)) 5.55
Decrease in profit for the year (12.95)

Effect on the statement of financial position


Increase / (decrease) in assets
Property, plant and equipment (20 – 1.5) (18.50)
(Increase) / decrease in liabilities
Deferred tax liability (Rs. 18.5 × 30%) 5.55
(Increase) / decrease in equity
Retained earnings (18.50 - 5.55) (12.95)

7.2 MOHANI MANUFACTURING LIMITED


Mohani Manufacturing (Private) Limited Retained
Statement of changes in equity Earnings
For the year ended December 31, 2015 Rs. in million
Balance at December 31, 2013 as previously reported (Rs. 89m – Rs. 21m) 68.00
Effect of change in accounting policy (Rs. 37m - Rs. 35.5m) (1.50)
Balance at December 31, 2013 – restated 66.50
Profit for the year ended December 31, 2014 - restated (W1) 39.70
Balance at December 31, 2014 – restated 106.20
Profit for the year ended December 31, 2015 (W2) 8.80
Balance at December 31, 2015 115.00

W1: Profit for the year ended December 31, 2014 (as restated)
Profit as previously reported 21.00
Incorrect recording of depreciation (Rs. 25 million – Rs. 10 million) 15.00

Reversal of FIFO method


Opening inventory 37.00
Closing inventory (42.30)
(5.30)
Application of weighted average method
Opening inventory (35.50)
Closing inventory 44.50
9.00
39.70

© Emile Woolf International 175 The Institute of Chartered Accountants of Pakistan


Financial accounting and reporting II

W2: Adjusted profit for year ended June 30, 2015


Profit as per draft financial statements 15.00
Adjustment in Opening Inventory
FIFO 42.30
Weighted average (44.50)
(2.20)
Adjustment in Closing Inventory
FIFO (58.40)
Weighted average 54.40
(4.00)
Adjusted profit 8.80

7.3 MARVELLOUS LIMITED


(a) Marvelous Limited
Extracts from statement of financial position
2016 2015
2017
Restated Restated
Assets ----------------- Rs. in million -----------------
Property, plant & equipment 714.63 630.37 677.56
2015: (650+36.75–9.19)
2016: (612+36.75–(9.19 ×2)
2017: 700+36.75–(9.19×3)+(15.95–10.5)
Equity & liabilities
Retained earnings 252.83 222.85 171.51
2015: 180 – (2.94+9.19 – 3.64)
2016: 240 – (60 – 51.34) – 8.49
2017: 275 – (35 – 29.98) – (8.66+8.49)
Deferred tax liability 48.50 44.65 36.36
2015: (40 –3.64)
2016: [52 – 3.64 – 3.71)
2017: [58 – 3.64 – 3.71 – 2.15)
Provision for decommission 46.30 42.87 39.69
(42.87+3.43) (39.69+3.18) (36.75+2.94)
Provision for taxation 12.00 16.00 24.00

(b) Extract from statement of profit or loss

2016
2017
Restated
Profit before tax 57.83 (65-3.43-9.19+5.45) 72.63 (85–3.18-9.19)
Taxation 27.85 21.29
(30-2.15) (25–3.71)
Profit after tax 29.98 51.34

© Emile Woolf International 176 The Institute of Chartered Accountants of Pakistan


Answers

(c) Correction of error note


It was identified in current year that the company did not recognise decommissioning liability
related to plant which was acquired on 1 July 2014. The effects of this error are as follows:

Effect on the statement of profit or loss 2016


Rs. in million
Increase/(decrease) in income:
Increase in finance cost (3.18)
Increase in depreciation (9.19)
Decrease in profit before tax (12.37)
Decrease in deferred tax liability (12.3 7×30%) 3.71
Decrease in pro fit after tax (8.66)

2016 2015
Effect on the statement of financial position -------- Rs. in million --------
Increase/(decrease) in retained earnings:
Increase in property, plant and equipment 18.38 (630.38–612) 27.56 (677.56–650)
Increase in provision for decommission (42.87) (39.69)
Decrease in deferred tax liability 7.35 (3.64 +3.71) 3.64
Decrease in retained earnings (17.14) (8.49)
Working:
Effects on Profit: 2017 2016 2015
------------------ Rs. in million ------------------
Correction of error:
Recording of Decommissioning liability
Increase in PPE (50÷1.084) 36.75
Increase in decommissioning liability (36.75)
Increase/(decrease) in income
(Increase) in finance cost (3.43) (3.18) (2.94)
(36.75+2.94+ (36.75+ 36.75×0.08
3.18)×0.08 2.94)×0.08
(Increase) in depreciation (36.75÷4) (9.19) (9.19) (9.19)
Change in Estimate
Reversal of dep. on RBM
(80×0.725×0.275) 15.95
Inclusion of dep. on SLM [(80×0.725–
16)÷4] (10.5)
Decrease in profit before tax (7.17) (12.37) (12.13)
Decrease in deferred tax (PBT×0.3) 2.15 3.71 3.64
Total effect on Profit (5.02) (8.66) (8.49)
Total effect on Retained earnings (22.17) (17.15) (8.49)

© Emile Woolf International 177 The Institute of Chartered Accountants of Pakistan


Financial accounting and reporting II

7.4 CHAND PAINTS LIMITED


(a) Chand Paints Limited
Notes to the financial statements for the year ended 30 June 2016
The effect of retrospective restatement on statement of comprehensive income is tabulated
below:

Increase / (decrease) in income 2015


Rs. in million

Increase in sales tax, commission and discounts (7,916 – 8,246) (330)


Decrease in cost of sales (44,633 – 44,606) 27

Increase in selling and distribution expenses (2,441 – 2,635) (194)


Increase in administration expenses (2,149 – 2,254) (105)
Decrease in operating income (602)
Decrease in other operating charges (515 – 467) 48
Decrease in other operating income ( 509 – 427 ) (82)
Decrease in profit for the year (636)

The effect of retrospective restatement on statement of financial position for 2015 is


tabulated below:

Decrease in trade debts (2,025 – 1,613) (412)


Increase in trade and other payables (8,894 – 8,670) 224
Decrease in un-appropriated profit (636)

(b) Chand Paints Limited


Statementcof changes in equity
for the year ended 30 June 2016

Share Retained
*Total
Description capital earnings
---------- Rs. in million ----------
Balance as on 1 July 2014 10,400 19,089 29,489
Interim dividend for the year ended 30 June 2015 (520) (520)
(10,400×5%)
Total comprehensive income for the year 2015 - 3,723 3,723
restated
Balance as at 30 June 2015 restated 10,400 22,292 32,692
Final dividend for the year ended 30 June 2015 (1,040) (1,040)
(10,400×10%)
Interim cash dividend for the year 2016 (1,040) (1,040)
(10,400×10%)
Total comprehensive income for the year ended 4,089 4,089
30 June 2016
Balance as at 30 June 2016 10,400 24,301 34,701

© Emile Woolf International 178 The Institute of Chartered Accountants of Pakistan


Answers

7.5 DAFFODIL LIMITED


Statement of changes in equity
For the year ended 31 December 2017

Ordinary
Share General Retained
share Total
premium reserves earnings
capital

-------------------- Rs. in million --------------------

Balance as at 31 December 2015 1,600.00 1,850.00 1,430.00 4,880.00


(As given)

Effect of correction of error (W-1) (54.69) (54.69)

Balance as at 31 December 2015 1,600.00 1,850.00 1,375.31 4,825.31


– Restated

Final cash dividend @ 7.5% - 2015 (120.00) (120.00)


(1,600×7.5%)

Right issue @ 25% 400.00 320.00 720.00

(1,600×25%) (160×25%×8)

Net profit - 2016 – Restated 331.67 331.67


[318+13.67(W-1)]

Transfer of incremental 49.00 49.00


depreciation

Balance as at 31 December 2016 - 2,000.00 320.00 1,850.00 1,635.98 5,805.98


Restated

Final bonus dividend @ 10% - 200.00 (200.00) -


2016 (2,000×10%)

Right issue 500.00 250.00 750.00

(50×10) (50×5)

Interim bonus dividend @ 15% - 405.00 (405.00) -


2017 (2,700×15%)

Net profit - 2017 [650 + 10.25 (W- 660.25 660.25


1)]

Transfer to general reserves 112.00 (112.00) -

Balance as at 31 December 2017 3,105.00 570.00 1,962.00 1,579.23 7,216.23

© Emile Woolf International 179 The Institute of Chartered Accountants of Pakistan


Financial accounting and reporting II

W-1: Correction of error

Correct Wrong
Increase/(decrease) in
depreciation @ depreciation @
depreciation
25% 25%
-------------------------- Rs. in million --------------------------
Cost 700 700
2014 160.42 87.50 72.92
(700 × 25% × 11 ÷ (700 × 25% × 6 ÷
12) 12)
2015 134.90 153.13 (18.23)
(700 – 160.42) × (700 – 87.50) ×
25% 25%
54.69
2016 101.17 114.84 (13.67)
(134.90 × 75%) (153.13 × 75%)
2017 75.88 86.13 (10.25)
(101.17 × 75%) (114.84 × 75%)

7.6 ASIF ENGINEERING LIMITED


(a) Asif Engineering Ltd.
Extracts from statement of financial position

2015 2014 2013


(Restated) (Restated)
----------------- Rs. in million -----------------
Property, plant & equipment (W-2) 2,498 2,461 2,149
Stores and spares 73 80 70
Retained earnings 421 283 145
(b) Extracts from statement of comprehensive income

2015 2014 (Restated)


--------- Rs. in million ---------
Net profit (W-1) 138 138

(c) Extracts from statement of changes in equity

Retained
earnings
Rs. in million
Balance as at 1 January 2014 101
Effect of retrospective change in accounting policy (W-1) 44
Balance at 1 January 2014 – restated 145
Total comprehensive income – 2014 (W-1) 138
Balance as at 1 January 2015 – (restated) 283
Total comprehensive income – 2015 138
Balance as at 31 December 2015 421

© Emile Woolf International 180 The Institute of Chartered Accountants of Pakistan


Answers

W-1: Computation of net profit

Depreciation expense for the year

2013 2014 2015

---------- Rs. in million ----------

Depreciation for 2013 (55÷5) 11 11 11

Depreciation for 2014 (39÷3) - 13 13

Depreciation for 2015 (44÷4) - - 11

11 24 35

Less: Amount already charged 55 39 44

Adjustment to be made in net profit 44 15 9

Profit for the year 123 129

Adjusted profit for the year 138 138

W-2: Property, plant and equipment

2013 2014 2015

---------- Rs. in million ----------

As given 2,105 2,402 2,430

Add: Stores issued 2013 55 55 55

Add: Stores issued 2014 0 39 39

Add: Stores issued 2015 0 0 44

Less: Accumulated depreciation as


calculated above 2014:
11 + 24; 2015: 11 + 24 + 35 (11) (35) (70)

Revised carrying value 2,149 2,461 2,498

© Emile Woolf International 181 The Institute of Chartered Accountants of Pakistan


Financial accounting and reporting II

CHAPTER 8 – IAS 12: INCOME TAXES


8.1 FRANCESCA
Rs. Rs.
Opening liability 1,340,600
Capital allowances during the year 50,000,000
Depreciation charged during the year (45,000,000)
–––––––––––
5,000,000  30% 1,500,000
–––––––––––
Interest receivable in statement of profit or loss 50,000
Interest received in tax computation (45,000)
–––––––––––
Receivable in statement of financial position 5,000  30% 1,500
–––––––––––
Interest payable in statement of profit or loss 32,000
Interest paid in tax computation (28,000)
–––––––––––
Payable in balance sheet 4,000  30% (1,200)
–––––––––––

Development costs as allowable expense 500,600  30% 150,180

Revaluation 6,000,000
Carrying value (4,900,500)
–––––––––––
Revaluation surplus 1,099,500 x 30% 329,850
––––––––––– ––––––––––
Closing liability 3,320,930
––––––––––

Rs.
Charged to the revaluation reserve 329,850
Charged in the statement of profit or loss (balancing figure) 1,650,480
––––––––––
Total movement on the provision of (3,320,930 – 1,340,600) 1,980,330
––––––––––
8.2 SHEP (I)
(a) Corporate income tax liability - year ended 31st December 2015
Rs.
Profit per accounts 121,000
Add Depreciation 11,000
————
133,000
Less tax depreciation (15,000)
————
Taxable profits 117,000
————
Tax payable @ 30% 35,100
————

© Emile Woolf International 182 The Institute of Chartered Accountants of Pakistan


Answers

(b) Deferred tax liability Rs.


Carrying amount (48,000 + 12,000 = 60,000 – 11,000) 49,000
Tax base (48,000 + 12,000 = 60,000 – 15,000) 45,000
———
Temporary difference (4,000)
———

Deferred tax liability required @ 30% (1,200)


———

(c) Movement on the deferred tax liability


Balance b/f 
Statement of profit or loss (balancing figure) 1,200
———
Balance c/f 1,200
———

(d) Statement of profit or loss note


Current tax expense 35,100
Deferred tax expense 1,200
————
Tax expense 36,300
————
8.3 SHEP (II)
(a) Corporate income tax liability - year ended 31st December 2016
Rs.

Profit per accounts 125,000


Add Depreciation 14,000
Interest payable 500
Provision 1,200
Fine 6,000
————
146,700
Less tax allowance (given) (16,000)
Interest receivable (150)
————
Taxable profits 130,550
————
Tax payable @ 30% 39,165
————

© Emile Woolf International 183 The Institute of Chartered Accountants of Pakistan


Financial accounting and reporting II

(b) Deferred tax liability


Carrying Tax Temporary
amount base difference
Rs. Rs. Rs.

Tangible assets
Carrying amount (49bf – 14) 35,000
Tax base (45bf – 16) 29,000 6,000
Interest payable (25,000 x 8% x 3/12) (500)  (500)
Interest receivable (4,000 x 15% x 3/12) 150  150
Provision (1,200)  (1,200)
——— ——— ———
33,450 29,000 4,450
——— ——— ———

Deferred tax @30% 1,335


———

(c) Movement on the deferred tax liability Rs.


Balance b/f 1,200
Statement of profit or loss (balancing figure) 135
———
Balance c/f 1,335
———

(d) Statement of profit or loss note


Current tax expense 39,165
Deferred tax expense 135
————
Tax expense 39,300
————

(e) Tax reconciliation


Accounting profit 125,000
————
Accounting profit @ 30% 37,500
Tax effect of the fine (6,000 @ 30%) 1,800
————
Tax expense 39,300
————

© Emile Woolf International 184 The Institute of Chartered Accountants of Pakistan


Answers

8.4 SHEP (III)


(a) Corporate income tax liability - year ended 31st December 2017
Rs.

Profit per accounts 175,000


Add Depreciation 18,500
Interest payable (note) 
Provision 2,000
Entertainment 20,000
————
215,500
Less tax allowance (given) (24,700)
Interest receivable (note) 
Development costs (17,800)
Provision (500)
————
Taxable profits 172,500
————

Tax payable @ 30% 51,750


————
Note
There is no adjustment to profit for the interest paid and the interest receivable.
Consider the interest payable. The tax authority will disallow the closing accrual but will allow last
year’s accrual (that has been paid in this year) as a deduction. These amounts are equal so there is
no net effect.
Similar comments can be made about the interest receivable.

(b) Deferred tax liability


Carrying Tax Temporary
amount base difference
Rs. Rs. Rs.
Tangible assets
Carrying amount (35bf – 18.5) 16,500
Tax base (29bf – 24.7) 4,300 12,200
Interest payable (500)  (500)
Interest receivable 150  150
Provision (2,700)  (2,700)
Development expenditure 17,800  17,800
——— —— ———
31,250 4,300 26,950
——— —— ———
Deferred tax @ 30% 8,085
———

© Emile Woolf International 185 The Institute of Chartered Accountants of Pakistan


Financial accounting and reporting II

(c) Movement on the deferred tax liability Rs.


Balance b/f 1,335
Statement of profit or loss (balancing figure) 6,750
———
Balance c/f 8,085
———
(d) Statement of profit or loss note
Current tax expense 51,750
Deferred tax expense 6,750
————
Tax expense 58,500
————
(e) Tax reconciliation
Accounting profit 175,000
————
Accounting profit @ 30% 52,500
Tax effect of the fine (20,000 @ 30%) 6,000
————
Tax expense 58,500
————
8.5 SHEP (IV)
(a) Corporate income tax liability - year ended 31st December 2017 Rs.
Taxable profits (as before) 172,500
————
Tax payable @ 34% 58,650
————

(b) Deferred tax liability


Temporary difference (as before) 26,950
———
Deferred tax @ 34% 9,163
———

(c) Movement on the deferred tax liability


Balance b/f 1,335
Adjustment due to change in rate 178
———
Opening balance restated to 34% (1,335 x 34/30) 1,513
Statement of profit or loss (balancing figure) 7,650
———
Balance c/f 9,163
———

(d) Statement of profit or loss note


Current tax expense 58,650
Deferred tax expense relating to origination and reversal
of temporary differences 7,650
Deferred tax expense resulting from increase in tax rate 178
————
Tax expense 66,478
————

© Emile Woolf International 186 The Institute of Chartered Accountants of Pakistan


Answers

(e) Tax reconciliation Rs.


Accounting profit 175,000
————
Accounting profit @ 34% 59,500
Tax effect of the fine (20,000 @ 34%) 6,800
Increase in opening deferred tax balances due to change in rate 178
————
Tax expense 66,478
————
8.6 WAQAR LIMITED
a) Computation of current period income tax liability
2015 2014
Rs.m Rs.m

Accounting profit before tax 40.00 30.00

Less: Admissible deductions


Capital Gain (10.00) (8.00)
Tax depreciation on furniture and fittings
Rs. 40.5 x 10% (4.05)
Rs. 40.5 (1-10%) x 10% (3.65)
Tax depreciation on Machinery
Rs. 90 x 10% (9.00)
Rs. 90 (1-10%) x 10% (8.10)

Add: Inadmissible deductions


Accounting depreciation on machinery 25.00 25.00
Accounting depreciation on furniture and fittings 5.00 5.00

Taxable profit 48.25 38.95


Tax rate 30% 35%

Tax payable (current tax) 14.48 13.63


b) Deferred taxation computation
Tax base Temporary Deferred
NBV (W1)
(W1) difference tax liability
Working 2 Rs.m Rs.m Rs.m Rs.m
At December 31,2013
Machinery 175.00 90.00 85.00 29.75
Furniture and fittings 40.00 40.50 (0.50) (0.18)
Deferred tax liability at December
31,2013 (35%) 29.57

© Emile Woolf International 187 The Institute of Chartered Accountants of Pakistan


Financial accounting and reporting II

Tax base Temporary Deferred


NBV (W1)
(W1) difference tax liability
Rs.m Rs.m Rs.m Rs.m
At December 31, 2014
Machinery 150.00 81.00 69.00 24.15
Furniture and fittings 35.00 36.45 (1.45) (0.51)
Deferred tax liability at December
31,2014 (35%) 23.64

WDV as at December 31, 2015


Machinery 125.00 72.90 52.10 15.63
Furniture and fittings 30.00 32.80 (2.80) (0.84)
Deferred tax liability at December
31,2015 (35%) 14.79

Working 1

Carrying amount and tax base of machinery NBV Tax base

Cost b/f 200.0 200.0

Accumulated depreciation b/f (25.0)

At 31 December 2013 175.0 90.0

Accounting depreciation (200/8 years) (25.0)

Tax depreciation (10% of WDV) (9.0)

At 31 December 2014 150.0 81.0

Accounting depreciation (200/8 years) (25.0)

Tax depreciation (10% of WDV) (8.1)

At 31 December 2015 125.0 72.9

Carrying amount and tax base of furniture and fittings

Cost b/f 50.0 50.0

Accumulated depreciation b/f (10.0)

At 31 December 2013 40.0 40.5

Accounting depreciation (10%  50) (5.0)

Tax depreciation (10% of WDV) (4.05)

At 31 December 2014 35.0 36.45

Accounting depreciation (10%  50) (5.0)

Tax depreciation (10% of WDV) (3.65)

At 31 December 2015 30.0 32.8

© Emile Woolf International 188 The Institute of Chartered Accountants of Pakistan


Answers

c) Movement on deferred taxation account (W2) 2015 2014


At January 1 23.64 29.57
Change due to change in rate (23.64  5/35) (3.38) -
20.26
Change due to origination and reversal of temporary
differences in the period (balancing figure) (5.47) (5.93)
At December 31 14.79 23.64

d) Tax expense
Current tax 14.48 13.63
Deferred tax: -
Due to origination and reversal of temporary differences in the
period (3.38)
Due to change in rate (5.47) (5.93)
Tax expense 5.63 7.7

e) Tax reconciliation
Accounting profit 40.0 30.0
Tax rate 30% 35%
12.0 10.5
Tax effect of untaxed gain:
30%  10.0 (3.0)
35%  8.0 (2.8)
Decrease in opening deferred tax balances due to change in
rate (with rounding adjustment) (3.37)
Tax expense 5.63 7.7

8.7 SHAKIR INDUSTRIES


COMPUTATION OF TAX EXPENSE
FOR THE YEAR ENDED DECEMBER 31, 2015
2015
Rs. in
million
Profit before tax 15.80
Add: Inadmissible expenses
Accounting depreciation (Rs. 1.1 million + Rs. 0.7 million) 1.80
Financial charges on finance lease 0.15
Penalty paid to SECP 0.70
Provision for gratuity 2.40
5.05

© Emile Woolf International 189 The Institute of Chartered Accountants of Pakistan


Financial accounting and reporting II

Less: Admissible expenses Rs.m


Tax depreciation 1.65
Lease payments 0.65
Payment of gratuity 1.60
Borrowing cost capitalised 2.30
6.20
Taxable profit for the year 14.65
Current tax expense @ 35% 5.13

COMPUTATION OF DEFERRED TAX EXPENSE


FOR THE YEAR ENDED DECEMBER 31, 2015
Carrying Tax Temp
amount base difference
Rs.m Rs.m Rs.m
Fixed assets – Owned 16.70 13.85 2.85
Fixed assets – Leased 1.80 - 1.80
Capital work in progress 2.30 - 2.30
Provision for gratuity (0.7 + 2.4 – 1.6) (1.50) - (1.50)
Obligation against assets subject to finance lease (1.20) - (1.20)
Total 4.25
Deferred tax expense @ 35% 1.49

Rs. in
million
Deferred tax liability (Opening) 0.55
Deferred tax expense for the year (balancing figure) 0.94
Deferred tax liability as at December 31, 2015 (Rs. 4.25 million x 35%) 1.49

8.8 MARS LIMITED


(a) Date Particulars Debit Credit
Rupees

01.07.2014 Motor Vehicle - Cost 1,600,000


Obligations under finance lease 1,600,000
Capitalisation of the lease

01.07.2014 Obligations under finance lease 480,000


Bank 480,000
First lease payment made in advance

© Emile Woolf International 190 The Institute of Chartered Accountants of Pakistan


Answers

(a) Date Particulars Debit Credit


Rupees

30.06.2015 Finance charges 153,451


Accrued finance charges 153,451

Finance charge accrual for the year ended June 30, 2015
Working: (Rs. 1,600,000  480,000)  13.701% = Rs. 153,451)

30.06.2015 Depreciation 400,000


Accumulated depreciation - Motor Vehicle 400,000

Depreciation charge for the year ended June 30, 2015


Working: Rs. 1,600,000 ÷ 4 = Rs. 400,000.
Assuming that there is no reasonable certainty about transfer of ownership at
the end of lease term.

30.06.2015 Tax expense (W1) 1,492,035


Tax payable 1,492,035
Recognition of tax expense for the year ended June 30, 2015)

30.06.2015 Tax expense 22,035


Deferred tax (W2) 22,035
Recognition of deferred tax asset.

W1 Tax computation
Rs.
Accounting profit before tax 4,900,000
Add: Depreciation on leased assets 400,000
Add: Finance charges 153,451
Less: Lease payment (480,000)
Taxable profit 4,973,451
Tax @ 30% 1,492,035

W2 Deferred tax computation


Carrying Tax
Difference
amount base
Taxable temporary difference
Leased assets 1,200,000 - 1,200,000
Deductible temporary difference
Obligations under finance lease (1,120,000) - (1,120,000)
Accrued finance charges (153,451) (153,451)
Net taxable temporary difference (73,451)
Deferred tax @ 30% (Asset) 22,035

© Emile Woolf International 191 The Institute of Chartered Accountants of Pakistan


Financial accounting and reporting II

(b) Liabilities against assets subject to finance lease (W3)


2015
Rs.
Present value of minimum lease payments 1,120,000
Less: Current maturity shown under current liabilities (326,549)
793,451
Minimum lease payments (W3)
Not later than 1 year 480,000
Later than 1 year and not later than 5 years (480,000 × 2) 960,000
1,440,000
Less: future finance charges on finance lease (320,000)
1,120,000
Present value of finance lease liabilities (W3)
Not later than 1 year 326,549
Later than 1 year and not later than 5 years (371,289 + 422,162) 793,451
1,120,000

The minimum lease payment has been discounted at an interest rate of 13.701% to arrive
at their present value. Rentals are paid in annual instalments.

W3: Repayment Schedule


Opening Principal Interest Annual Closing
Years Balance repayment 13.701% payment Balance
Rs.m Rs.m Rs.m Rs.m Rs.m
2015 1,600,000 480,000 480,000 1,120,000
2016 1,120,000 326,549 153,451 480,000 793,451
2017 793,451 371,289 108,711 480,000 422,162
2018 422,162 422,162 57,838 480,000 -
320,000

8.9 BILAL ENGINEERING LIMITED


(a) Computation of current taxation
Rs.m Rs.m
Profit before tax 50.000
Add: Accounting depreciation 10.000
Financial charges on lease liability (1.00 – 0.3) × 13.701% 0.096
Amortization of research and development cost for the year 1.000
Less: Tax depreciation (7.000)
Annual instalment of lease payment (0.300)

Amortization of research and development cost (15 ×


0.9/10) (1.350)
Current year taxable income 52.446

© Emile Woolf International 192 The Institute of Chartered Accountants of Pakistan


Answers

Rs.m Rs.m
Tax liability for the year (52.446 × 35%) 18.356

Tax liability for prior periods (0.100 × 35%) 0.035


18.391

Deferred taxation

Accounting depreciation 10.000


Tax depreciation (7.000) 3.000

Financial charges on finance lease liability(1.00 – 0.3) × 13.701% 0.096


Annual instalment of lease payment allowed under tax (0.300) (0.204)

Amortization charged in accounts 1.000


Amortization cost claimed in tax (1.350) (0.350)

Excess of taxable income over accounting profit due to time differences 2.446
Deferred tax credit at 35% (0.856)
Total tax expenses (current and deferred) 17.535

(b) Bilal Engineering Limited: Notes to the financial statements


for the year ended December 31, 2015

1.1 Relationship between tax expense and accounting profit 2015


Rs.m
Accounting profit before tax 50.000
Tax on accounting profit at 35% 17.500
Tax on expenses disallowed (Permanent Difference) 0.035
Effective tax rate/tax charge 17.535

(c) Journal entries Debit Credit


Rs.m Rs.m
1 Income tax expenses 18.391
Provision for taxation 18.391
(Tax provision for 2015)
2 Deferred tax asset 0.856
Tax expenses – deferred 0.856
(Deferred tax credit for 2015)

© Emile Woolf International 193 The Institute of Chartered Accountants of Pakistan


Financial accounting and reporting II

8.10 GALAXY INTERNATIONAL


28 : TAXATION 2015 2014
Rs.m Rs.m
Current - for the year (W – 1) 0.84 -
Deferred (W – 2) 6.95 (0.96)
7.79 (0.96)

28.1 : Relationship between tax expense and accounting profit


Profit/(Loss) before taxation 23.50 (1.75)

Tax at the applicable rate of 35% 8.23 (0.61)


Tax effect of exempt income (1.25 x 35%) (0.44) (0.35)
7.79 (0.96)

W1 : Computation of Current Tax


(Loss) / profit before tax as per books 23.50 (1.75)
Add: Allowable income / Disallowed expenses
Accounting depreciation 15.00 15.00
Provision for gratuity 2.20 1.70
Accrued expenses - 2.00

Less: Disallowed income / Allowable expenses


Tax depreciation (6.00) (45.00)
Interest income from SIBs (Exempt) (1.25) (1.00)
Accrued expenses (2.00)
Taxable income / (loss) 31.45 (29.05)
Tax liability (@ 35% 11.01 -
Tax loss to be brought forward (29.05 x 35%) (10.17) -
Tax payable 0.84 -

W -2: Computation of Deferred Tax


Timing differences (cumulative) on account of:
Depreciation (2015: 30-51, 2014: 15-45) 21.00 30.00
Accrued expenses - (2.00)
Provision for gratuity (3.90) (1.70 )
Tax losses - (29.05)
17.10 (2.75)

Deferred tax @ 35% 5.99 (0.96)


Add: Opening deferred tax (dr.) 0.96 -
Charge/(Reversal) for the year 6.95 (0.96)

© Emile Woolf International 194 The Institute of Chartered Accountants of Pakistan


Answers

8.11 APRICOT LIMITED


Taxation 2015 2014
Rs.m Rs.m
Current (W1) 20.48 10.76
Deferred (W2) (1.58) (21.35)
18.90 (10.59)

Relationship between tax expense and accounting profit 2015


Profit before taxation 60.00
Tax at the applicable rate of 35% 21.00
Less: Tax effect of exempt income (2.10)
18.90

W1: Computation of Current Tax


Profit before tax as per books 60.00 45.00
Add: Allowable income / Disallowed expenses
Accounting depreciation 10.00 9.00
Tax profit on sale of fixed assets 1.00 -
Bad debt expense 5.00 7.00
Less: Disallowed income / Allowable expenses
Tax depreciation (8.00) (7.00)
Accounting profit on sale of fixed assets (0.50) -
Capital gain (6.00) -
Bad debts written off (3.00) (4.00)
Taxable income 58.50 50.00
Tax liability (@ 35%) 20.48 17.50

W2: Computation of Deferred Tax


Fixed assets (2014: 95-90, 2015: 82.5-80) (W2.1) 0.87 1.75
Provision for bad debts (2014: 12×35%, 2015: 14×35%) [W2.2] (4.90) (4.20)
Closing balance of deferred tax (4.03) (2.45)
Less: Opening balance (2.45) (18.90)
Charge for the year (1.58) (21.35)

W2.1 Movement of Fixed Assets Accounting Tax


Opening balance 95.00 90.00
Disposal during the year (2.50) (2.00)
Depreciation for the year - 2015 (10.00) (8.00)
Closing balance 82.50 80.00

W2.2 Movement of provision for bad debts 2015 2014


Opening balance 12.00 9.00
Provision for the year 5.00 7.00
Write off during the year (3.00) (4.00)
Closing balance 14.00 12.00

© Emile Woolf International 195 The Institute of Chartered Accountants of Pakistan


Financial accounting and reporting II

8.12 ROSE LIMITED


Computation of deferred tax liability / asset
As on 31 December 2017

(Deductible)
/Taxable Deferred tax
Carrying value Tax base
Description Temporary Tax rate (Asset)/Liability
difference
---------- Rs. in million ---------- Rs. in million
Advertising cost 0 12 (12) 30% (3.60)

Trade & other payable


- Unearned commission (10) (10) - 30% -
- Other (30) (25) (5) 30% (1.5)

Other receivables
- Dividend receivable 8.00 8.00 - 0% -
- Other 9.00 6.00 3.00 30% 0.9
Interest receivable 3.00 - 3.00 15% 0.45
40×10%×9÷12
Machine 48.82*1 -

Finance lease liability (48.60) *2 -


Interest accrued on (4.86) -
finance
lease (48.6×10%)
(4.64) - (4.64) 30% (1.39)

Plant 234.59 225.00 9.59 30% 2.88


(250+31.5) (250×90%)
×(5÷6) -
Provision for (34.03)
decommissioning (50÷1.085) (34.03) 30% (10.21)

Buildings (300×25÷30) 250 177.147 72.85 30% 21.86


(300÷30×25) (300×0.95)
Deferred tax liability - net 9.39
*1
97.63[28×{1+(1–1.1-3)÷0.1}]×2÷4
*2
[28×{(1–1.1-2)÷0.1}]

© Emile Woolf International 196 The Institute of Chartered Accountants of Pakistan


Answers

8.13 FLOOR AND TILES LIMITED


2015
Rs. in million
20 – Taxation
Current
- for the year [83.11(W-1) × 32%] 26.60
- for prior year (6 × 32%) 1.92
Deferred (W-2) (2.28)
26.24
Reconciliation of tax charge for the year
Accounting profit before tax 80.00

Applicable tax rate 32%

Tax on accounting profit at 32% 25.60


Add: Effect of permanent difference (1 × 32%) 0.32
Add: Effect of prior year taxation (6 × 32%) 1.92
Less: Effect of exempt income (5 × 32%) (1.60)
26.24
Workings
W-1: Computation of current tax expense for the year

Rs. in million
Profit before tax 80.00
Add: Inadmissible expenses / admissible income
Accounting depreciation (W-3) 17.50
Depreciation on leased assets (120 ÷ 5) 24.00
Tax profit on disposal [35 – 24.3(W-4)] 10.70
Finance charges on leases [(120 – 30) × 12.59%] 11.33
Provision for bad debts 10.00
Provision for gratuity 12.00
Penalty paid 1.00
86.53
Less: Admissible expenses / inadmissible income
Tax depreciation (W-4) (29.92)
Accounting profit on disposal [35 – 26.5 (W-3)] (8.50)
Gratuity paid (10.00)
Lease rental (30.00)
Capital gain (exempt) (5.00)
(83.42)
Taxable profit 83.11

© Emile Woolf International 197 The Institute of Chartered Accountants of Pakistan


Financial accounting and reporting II

W-2: Computation of deferred tax liability / (assets)

Carrying
Tax Base Difference
Amount
Building - owned (W-3) & (W-4) 311.00 269.28 41.72
Machine - lease [120-(120÷5)] 96.00 - 96.00
Provision for bad debts (2 + 10) 12.00 - (12.00)
Liabilities against assets subject to finance lease 90.00 - (90.00)
(120-30)
Accrued finance charges on lease (120– 11.33 - (11.33)
30)×12.59%
Provision for gratuity (13+12-10) 15.00 - (15.00)
Total differences 9.39

Closing deferred tax liability (9.39×32%) 3.00


Less: Opening deferred tax liability as given (5.28)
Deferred tax income for the year (2.28)

W-3: Accounting depreciation

Opening Acc. Depreciation


for the year Carrying
Cost Depreciation
amount
(cost×5%× 2) (cost×5%)
A B C D=A-B-C
Building (350-30) 320 32 16.00 272.00
Purchased 40 1.00 39.00
Disposal 30 3 0.50 26.50
Owned assets 390 35 17.50 337.50
Less: disposed off asset (30) (26.50)
Owned assets at year 360 17.50 311.00
end
W-4: Tax depreciation

Depreciation for
Opening Acc. Carrying
Cost the year
Depreciation amount
(cost×5%)
B = [A-
A (A×0.9×0.9)] C = (A - B)×0.1 D=A-B-C
Building (350-30) 320 60.8 25.92 233.28
Purchased 40 4.00 36.00
Disposal 30 5.7 24.30
Owned assets 390 66.5 29.92 293.58
Less: disposed off asset (30) (24.30)
Owned assets at year-
end - as per tax rules 360 29.92 269.28

© Emile Woolf International 198 The Institute of Chartered Accountants of Pakistan


Answers

8.14 HADI LIMITED


(a) Hadi Limited
Statement of comprehensive income
For the year ended 31 december 2016

Sales 201,407
Less: Cost of sales (W-1) (88,164)
Gross profit 113,243
Less: Administrative expenses (W-2) (47,382)
Less: Loss on sale of fixed assets [20,000-(25,000-3750)] (1,250)
Net profit before tax 64,611
Taxation: Current [req (b)] (17,527)
Deferred [req (b)] 2,643
Net profit after tax 49,727
Other comprehensive income -
Total comprehensive income 49,727

W-1: Cost of sales


Given 78,664
Less: R & D wrongly charged to cost of sales (20,000)
Depreciation expense [(305,000–25,000–15,000)×10%)
+(25,000×10%×6/12)+(35,000×10%×6/12)] 29,500
88,164

W-2: Administrative expenses


As given 37,636
Add: Bad debts expense (W-3) 6,204
Provision for warranty (W-4) 1,542
Research expense (20,000×4/10) 8,000
Amortization of development cost [20,000×6/10=12,000/20×2/12)] 100
Impairment of development cost (11,900– 10,000) 1,900
Less: Payment against warranty claim of the products sold during previous year
and wrongly charged to admin expense (8,000)
47,382

W-3: Bad debts expense


Provision for doubtful debts - closing balance [(61,400– 8,000)×6%] 3,204
Amount written off (10,000×80%) 8,000
Less: Provision for doubtful debts - opening balance (5,000)
Bad debts expense for the year 6,204

© Emile Woolf International 199 The Institute of Chartered Accountants of Pakistan


Financial accounting and reporting II

W-4: Provision for warranty Rs. in '000


Charge for the year (201,407×3%) 6,042
Less: Warranty expired (10,000 – 8,000) (2,000)
Payment of current year already charged off (2,500)
Net impact to be taken to SOCI 1,542

(b) Computation of current tax


Profit before tax [req. (a)] 64,611
Add: Inadmissible expense/Admissible income
Accounting depreciation [req. (a)] 29,500
Accounting loss on disposal [req. (a)] 1,250
Tax gain on disposal (20,000–(25,000×0.75×0.85) 4,063
Accounting amortization [req. (a)] 100
Impairment on development expense [req. (a)] 1,900
Bad debts expense [req. (a)] 6,204
Provision for warranty (6,042–2,000) 4,042
Research expense [req. (a)] 8,000

Less: Admissible expense


Tax depreciation [{153–(25×0.75×0.85)} ×0.15]+[35×0.25]+[35×0.75×0.15] (33,247)
*Tax amortization (20,000/10) (2,000)
Bad debts written off (8,000)
Finance cost on long term loan (75,000×10%) (7,500)
Payment against warranty (2,500+8,000 ) (10,500)
Taxable profit 58,423

Current tax @ 30% 17,527


(b)

Computation of deferred tax Carrying


Tax base Difference
(Balance sheet approach) amount
---------------- Rs. in ‘000 ----------------
Plant & machinery (W-1) 221,000 138,815 82,185
Development cost 10,000 18,000 (8,000)
Provision for bad debts (61,400– 8,000)×6% (3,204) - (3,204)
Finance cost capitalized 7,500 - 7,500
Provision for warranty (6,042–2,500) (3,542) - (3,542)
Total difference 74,939

Closing deferred tax liability (74,939×30%) 22,482


Opening deferred tax liability (25,125)
Deferred tax (income)/expense (2,643)

© Emile Woolf International 200 The Institute of Chartered Accountants of Pakistan


Answers

W-1: Plant and machinery Rs. in '000


Carrying amount
Cost – given 305,000
Disposal of machine to be recorded (25,000)
Capitalisation of machine to be recorded 20,000
Less: Accumulated depreciation (53,250+29,500–3,750) (79,000)
221,000
Tax base
WDV - opening balance 153,000
WDV of machine disposed off (25,000×0.75×0.85) (15,938)
Cost of machine acquired 35,000
Tax depreciation for the year (33,247)
138,815

© Emile Woolf International 201 The Institute of Chartered Accountants of Pakistan


Financial accounting and reporting II

CHAPTER 9: IAS 21: FOREIGN CURRENCY TRANSACTIONS


9.1 DND LIMITED
Date Description Dr. Cr.
Rs. Rs.
1-Jul-16 Advance to suppliers 1,210,000
Cash 1,210,000
(Amount paid on signing the contract. Exchange
rate was Rs. 60.5/US$)
30-Sep-16 Advance to suppliers 3,050,000
Cash 3,050,000
(Amount paid on delivery. Exchange rate was Rs.
61/US$)
30-Sep-16 PPE in transit/ CWIP 6,090,000
Advance to suppliers 4,260,000
Payable to suppliers 1,830,000
(Recording of asset on the delivery date as risk
and rewards are transferred to the company)
31-Dec-16 Exchange loss 6,000
Payable to suppliers 6,000
(Adjustment of exchange rate as of reporting date.
Exchange rate was Rs. 60.5/US$)
31-Jan-17 Property, plant and Equipment 6,090,000
PPE (In transit/ in progress) 6,090,000
(Transfer the new plants and machineries to
Property, Plant and Equipment)
31-Jan-17 Payable to suppliers 1,836,000
Exchange loss (Bal.) 9,000
Cash 1,845,000
(Final payment to supplier. Exchange rate was Rs.
61.5/US$1)

9.2 ORLANDO
(a) Year to June Year 4
The revenue and the receivable for the sale of €96,000 should be translated at the spot rate
of 0.8 = $120,000
The capital expenditure of €1m should also be translated at the spot rate of 0.8:
Debit Property, plant and equipment $1,250,000
Credit: Payables $1,250,000.
The receipt on 12 June relating to the receivable is translated at the rate at that date of 0.9.
This generates cash of $106,667 to settle a receivable of $120,000. Hence an exchange
loss of $13,333 is recognised in profit or loss.

© Emile Woolf International 202 The Institute of Chartered Accountants of Pakistan


Answers

The non-current asset is not re-translated at the year end, but the outstanding payable (a
monetary item) must be re-stated to the year end exchange rate of 0.7. This gives a year-
end payable balance of $1,428,571. This has increased from the initial $1,250,000; therefore
an exchange loss of $178,571 will be recognised in profit or loss.

(b) Year to June Year 5


When the payable is settled after the year end at the spot rate of 0.8, it results in a payment
of $1,250,000. There is an exchange gain of $178,571 compared with the carrying value at
the end of Year 4.
9.3 OMEGA LIMITED
Omega Limited
Extract from Statement of comprehensive income for the year ended 31 December 2013
 Profit for the year: Rupees
Dividend received from AWL
(IFRS 9,B5.7.5.1) (20,000*10*15%*26.5) 795,000
Transfer of FV gain reserve of 31-12-2012, on derecognition of
AWL investment W.1 500,000
FV / exchange gains on valuation of AWL shares on 1-6-2013 W.1 2,124,000
Loss on de-recognition of AWL' shares W.1 ( 308,000)
 Other comprehensive income:
FV gain/(loss) on investment available for sale W.1 693,000
Exchange gain on investment available for sale W.1 225,225

W-1 FV per Gain /


No. of Investment
Date share (loss) Remarks
shares
AED AED Conv.@ Rupees Rupees
1-May-2012 20,000 12.00 240,000 25.00 6,000,000
31-Dec-2012 20,000 13.00 260,000 25.00 6,500,000 500,000 FV gain
1-Jun-2013 22,000 14.00 308,000 28.00 8,624,000 2,124,000 Gain on
(20,000x1.1) valuation of
AWL on its
acquisition
by HL
1-Jun-2013 16,500 18.00 297,000 28.00 8,316,000 (308,000) Loss on de-
(22,000/4*3) recognition
of
AWL shares
31-Dec-2013 16,500 19.50 321,750 28.00 9,009,000 693,000 FV gain
31-Dec-2013 16,500 19.50 321,750 28.70 9,234,225 225,225 Exchange
gain
3,234,225

© Emile Woolf International 203 The Institute of Chartered Accountants of Pakistan


Financial accounting and reporting II

9.4 KANGAROO LIMITED


Kangaroo Limited
Statement of Financial Position
As on 31 December 2017

Assets Rs. in million

Investment property (W-1) 290.00

Investments (105 + 130) (W-2) 235.00

Liabilities

Unearned rent (0.24 × 8 ÷12 × 110) 17.60

Statement of comprehensive income


For the year ended 31 December 2017

Profit and Loss account Rs. in million

Exchange loss on 20% payment (2.6 × 20% × (105 – 108) (1.56)

Increase in fair value of investment property (W-1) 18.30

Rent income (0.24 × 4÷12 × 110) 8.80

Transaction cost – Investment-A (2.00)

Dividend income (12 + 9) 21.00

Realised gain on investment-A [(23 × 0.98 – (100 × 20%)] 2.54

Unrealised Gain – Investment-A (W-2) 25.00

Other comprehensive income

Unrealized gain- Investment-B (W-2) 22.90

Realised gain on investment-B [(50 × 0.98 – (153 × 0.3)] 3.10

W-1: Investment company Rs. in milllion

Advance payment (2.6 × 10% × 100) 26.00

Initial recognition (2.6 × 70% × 105) 191.10

(2.6 × 70% × 105) 54.60

Total cost 271.70

Fair value (2.5 x 116) 290.00

Gain (P & L) 18.30

© Emile Woolf International 204 The Institute of Chartered Accountants of Pakistan


Answers

W-2: Investments Investment A Investment B


--------- Rs. in million ---------
Purchase price 100.00 150.00
Transaction cost 3.00
Total cost 100.00 153.00
Cost of shares held at 31 Dec 2017 (100×80%) 80.00 (153×70%)107.10
Fair value - 31 Dec 2017 105.00 130.00
Gain 25.00 22.90

9.5 MZA LIMITED


a) Dr. Cr.
Date Description
$ $
4-Mar-18 Equipment 200,000.00
Accounts payable 200,000.00
Purchase of equipment

25-Aug-18 Accounts payable 200,000.00


Exchange gain 60,000.00
Bank 260,000.00
Payment of accounts payable

b) 27-Feb-18 Accounts receivable 68,000.00


Sales 68,000.00
Revenue recognition

27-Feb-18 Cost of sales 46,000.00


Inventory 46,000.00
Cost recognition

25-May-18 Bank 71,044.78


Accounts receivable 68,000.00
Exchange loss 3,044.78
Receipt of accounts receivable

c) 2-Sep-18 Accounts receivable 24,000.00


Sales 24,000.00
Revenue recognition

© Emile Woolf International 205 The Institute of Chartered Accountants of Pakistan


Financial accounting and reporting II

Dr. Cr.
Date Description
$ $
2-Sep-18 Cost of sales 17,000.00
Inventory 17,000.00
Cost recognition

31-Dec-18 Exchange gain 277.33


Accounts receivable 277.33
Year end closing rate

7-Feb-19 Bank 23,146.57


Exchange gain 576.09
Accounts receivable 23,722.67

Receipt of accounts receivable

d) 25-May-18 Bank 284,000.00


Loan 284,000.00
Receipt of loan

31-Dec-18 Loan 47,333.33


Exchange loss 47,333.33
Year end closing rate

© Emile Woolf International 206 The Institute of Chartered Accountants of Pakistan


Answers

CHAPTER 10 – IAS 38: INTANGIBLE ASSETS


10.1 FAZAL
In accordance with IAS 38, expenditure on intangible assets must be expensed unless it meets the
recognition criteria for capitalisation. These criteria require the demonstration that future benefits will
arise from the incurred costs. It would be difficult to prove that this is the case in relation to training
costs and IAS 38 specifically states that training costs should always be expensed as they are
incurred and not treated as an intangible asset.
Hence the treatment adopted by Fazal is not correct and the costs being carried forward must be
expensed to the year’s profits.
10.2 HENRY
Property, plant and equipment
Plant and machinery
Cost Rs.
On 1 January 2015 X
Additions 30,000
–––––––
On 31 December 2015 X
–––––––
Accumulated depreciation
On 1 January 2015 X
Charge for the year (30,000  9/12 ÷ 5) 4,500
–––––––
On 31 December 2015 X
–––––––
Carrying amount
On 31 December 2014 X
–––––––
On 31 December 2015 25,500
–––––––
Intangible assets

Internally generated research and development expenditure


Cost Rs.
On 1 January 2015 412,500
Additions 45,000
––––––––
On 31 December 2015 457,500
––––––––

Accumulated amortisation
On 1 January 2015 -
Charge for the year (W) 68,750
––––––––
On 31 December 2015 68,750
––––––––
Carrying amount
On 31 December 2014 412,500
––––––––
On 31 December 2015 388,750
––––––––

© Emile Woolf International 207 The Institute of Chartered Accountants of Pakistan


Financial accounting and reporting II

Working
Amortisation charge (Project A)

Rs.
Total savings (100,000 + 300,000 + 200,000) 600,000
2015 amortisation charge (100,000/600,000  412,500) 68,750

Tutorial notes
The costs in respect of Project B cannot be capitalised as there are uncertainties surrounding the
successful outcome of the project – but the machine bought may be capitalised in accordance with
IAS16.
The 2015 costs in respect of Project C can be capitalised as the uncertainties have now been resolved.
However, the 2014 costs cannot be reinstated.

10.3 TOBY
Intangible assets
Goodwill Patents Brands Total
Rs. Rs. Rs. Rs.
Cost
On 1 January 2015 - - - -
Additions (W1) 10,000 20,000 50,000 80,000
––––––– ––––––– ––––––– –––––––
On 31 December 2015 10,000 20,000 50,000 80,000
––––––– ––––––– ––––––– –––––––
Accumulated amortisation/impairment
On 1 January 2015 - - - -
Written off/amortised during the year
(W1 and W2) 3,000 2,500 7,500 13,000
––––––– ––––––– ––––––– –––––––
On 31 December 2015 3,000 2,500 7,500 13,000
––––––– ––––––– ––––––– –––––––
Carrying amount
On 31 December Year 0 - - - -
––––––– ––––––– ––––––– –––––––
On 31 December 2015 7,000 17,500 42,500 67,000
––––––– ––––––– ––––––– –––––––
Workings
(1) Goodwill on acquisition of George

Rs.
Cost of acquisition 105,000
Minus fair value of net assets acquired (100,000 – 5,000) (95,000)
––––––––
Goodwill 10,000
Recoverable value (7,000)
––––––––
Impairment write off 3,000
––––––––
(2) Amortisation of patent
20,000 ÷ 8 = Rs.2,500

© Emile Woolf International 208 The Institute of Chartered Accountants of Pakistan


Answers

(3) Amortisation of brand


50,000 ÷ 5 × 9/12 = Rs.7,500
Tutorial note
IAS38 Intangible assets prohibits the recognition of internally generated brands (3) or internally-
generated goodwill (4).

10.4 BROOKLYN
1 Development expenditure
IAS 38 on intangibles requires that research and development be considered separately:
 research – which must be expensed as incurred
 development – which must be capitalised where certain criteria are met.
It must first be clarified how much of the Rs.3 million incurred to date (10 months at
Rs.300,000) is simply research and how much is development. The development element will
only be capitalised where the IAS 38 criteria are met. The criteria are listed below together
with the extent to which they appear to be met.
 The project must be believed to be technically feasible. This appears to be so as the
feasibility has been acknowledged.
 There must be an intention to complete and use/sell the intangible. Completion is
scheduled for June 2016
 The entity must be able to use or sell the intangible. Interest has been expressed in
purchasing the knoWhow on completion
 It must be considered that the asset will generate probable future benefits. Confirmation
is required from Brooklyn as to the extent of interest shown by the pharmaceutical
companies and whether this is of a sufficient level to generate orders and to cover the
deferred costs.
 Availability of adequate financial and technical resources must exist to complete the
project. The financial position of Brooklyn must be investigated. A grant is being
obtained to fund further work and the terms of the grant, together with any conditions,
must be discussed further.
 Able to identify and measure the expenditure incurred. A separate nominal ledger
account has been set up to track the expenditure.
If all of the above criteria are met, then the development element of the Rs.3m incurred to
date must be capitalised as an intangible asset. Amortisation will not begin until commercial
production commences.
2 Provision
Although the claim was made after the reporting period, IAS 10 considers this to be an
adjusting event after the reporting period. The employment of the individual dates back to
20X2 and so the lawsuit constitutes a current obligation for the payment of damages as a
result of this past event (the employment).
The amount and the timing are not precisely known but the likelihood of payment of damages
by Brooklyn is probable and so a provision should be made for the estimated amount of the
liability, as advised by the lawyer. Disclosure, rather than provision, would only be appropriate
if the expected settlement was possible or remote, and the lawyer’s view is that a payment is
more likely than not.
It is not appropriate to calculate an expected value where there is only one event, instead a
provision should be made for the most likely outcome. The lawyer has various views on the
possible payout, but the most likely payout is Rs.500,000 as this has a 50% probability. As
settlement of the provision is not anticipated until 2018, the provision should be discounted
back at 8% to give a liability of Rs.476,280.

© Emile Woolf International 209 The Institute of Chartered Accountants of Pakistan


Financial accounting and reporting II

Provided that the payment from the insurance company is virtually certain, this should be
shown as an asset, also at its discounted value of Rs.47,628, being 10% of the provision.
In both cases the discounting should be unwound over the coming three years through profit
or loss.
3 Revaluation
IAS 16 on Property, Plant and Equipment does not impose a frequency for updating
revaluations. It simply requires a revaluation where it is believed that the fair value of the
asset has materially changed. Hence, if in the past there have been material differences
between the carrying amount and fair value at the 5 yearly review then Brooklyn should
consider having more frequent valuations following on from this year’s valuation.
Revaluations should be regular and not timed simply when property prices are at a peak. It is
not acceptable for Brooklyn to defer its next revaluation while values are low. If property prices
do fall in 2016, then it may be necessary to perform an impairment test in accordance with
IAS 36 Impairment of assets.
If it is believed that an asset value has moved materially, then all assets in that class must be
revalued. Hence it is not sufficient for Brooklyn to just revalue the London property.
IAS 16 does not require the valuation to be performed by an external party, and so the use of
the property manager to conduct the valuations is acceptable. Notes to the financial
statements will disclose that he is not independent of the company.

10.5 ZOUQ INC


(a) (i) The depreciable amount of an intangible asset with a finite useful life shall be allocated on
a systematic basis over its useful life.

(ii) Amortization shall begin when the asset is available for use

(iii) Amortization shall cease at the earlier of the date that the asset is classified as held for
sale and the date that the asset is derecognised.

(iv) The amortization method used shall reflect the pattern in which the asset's future economic
benefits are expected to be consumed by the entity.

(v) The amortization charge for each period shall be recognised in statement of profit or loss.

(b) Goodwill Account


Rupees Rupees
Goodwill recognised Impairment
01.01.2014 (W1) 270,000,000 31.12.2014 of goodwill 50,000,000
31.12.2014 Balance b/d 220,000,000

270,000,000 270,000,000
01.01.2015 Balance b/d 220,000,000
31.12.2015 Balance b/d 220,000,000

220,000,000 220,000,000

Brand Account
Rupees Rupees
01.01.2014 Brand recognised 100,000,000 31.12.2014 Amortization 10,000,000

31.12.2014 Balance c/d 90,000,000

100,000,000 100,000,000

© Emile Woolf International 210 The Institute of Chartered Accountants of Pakistan


Answers

Brand Account
Rupees Rupees
01.01.2015 Balance b/d 90,000,000 31.12.2015 Amortization 10,000,000
Impairment of
31.12.2015 Brand 13,500,000
- 31.12.2015 Balance c/d 68,000,000
90,000,000 90,000,000

W1: Value of goodwill


Rupees
Purchase price (50,000,000 x Rs. 30 x 90%) 1,350,000,000
Less: Fair value of net identifiable assets and liabilities
(Rs. 1,100,000,000 x 90%) (990,000,000)
Less: Value of brand (Rs. 100,000,000 x 90%) (90,000,000)
Goodwill recognised 270,000,000

10.6 RAISIN INTERNATIONAL


(a) Following are the criteria that should be used while recognizing intangible assets from research
and development work.

(i) No intangible asset arising from research shall be recognised.

(ii) An intangible arising from development shall be recognised if, and only if , an entity can
demonstrate all of the following:

 the technical feasibility of completing the intangible asset so that it will be available
for use or sale.

 its intention to complete the intangible asset and use or sell it.

 its ability to use or sell the intangible asset.

 how the intangible asset will generate probable future economic benefits. Among
other things, the entity can demonstrate the existence of a market for the output of
the intangible asset or the intangible asset itself or, if it is to be used internally, the
usefulness of the intangible asset.

 the availability of adequate technical, financial and other resources to complete the
development and to use or sell the intangible asset.

 its ability to measure reliably the expenditure attributable to the intangible asset
during its development.

(b) (i) Since the product met all the criteria for the development of the product, it should be
recognised as an intangible in the statement of financial position (SOFP) of the company.
However, RI should capitalise only the development work (i.e. Rs. 9 million) as intangible
asset. IAS-38 does not allow capitalization of cost relating to the research work, training
of staff and cost of trial run.
Since the product has a useful life of 7 years, the amortization expense amounting to Rs.
0.32 million (Rs. 9 million × 3/12 ÷ 7 years) should be recorded in the statement of profit
or loss.

© Emile Woolf International 211 The Institute of Chartered Accountants of Pakistan


Financial accounting and reporting II

(ii) This purchasing of right to manufacture should be recognised as an intangible in the


SOFP because:
 it is for an established product which would generate future economic benefits.

 cost of the patent can be measured reliably.

Since there is a finite life, the patent must be amortised over its useful life. The useful life
will be shorter of its actual life (i.e. 10 years) and its legal life (i.e. 5 years. The amortization
to be recorded in SOCI is Rs. 2.83 million (Rs. 17 million × 10/12 ÷ 5).

(iii) The acquired brand should be recognised as an intangible in the SOFP because
acquisition price is a reliable measure of its value. The amortization to be recorded in
SOCI is Rs. 0.12 million (Rs. 2 million ÷ 10 years x 7/12).
(iv) The carrying value of the intangible asset should be increased to Rs. 10 million in the
SOFP. Since there is an indefinite useful life of the intangible assets, it should not be
amortised. Instead, RI should test the intangible asset for impairment by comparing its
recoverable amount with its carrying amount.

10.7 INTERNATIONAL ASSOCIATES LIMITED


Notes to the financial statements for the year ended 30 June 2016

Brands Software License Development Total


Note : Intangible assets
----------------------- Rs. in million -----------------------

Gross carrying amount–opening 200 80 15 - 295


balance

Accumulated amortization (40) (48) - - (88)

Net carrying amount–opening 160 32 15 - 207


balance

Additions - 120*1 - 30.00 *2 150

Less : Amortization for the year (20) (15) *3 (0.75) (0.75) (36.50)

Impairment loss (P/L) - (20) *4 - (20)

Net carrying amount–closing 140 117 14.25 29.25 300.50


balance

Gross carrying amount–closing 200 200 15 30.00 445


balance

Accumulated amortization and


impairment loss (60) (83) (0.75) (0.75) (144.50)

Net carrying amount–closing 140 117 14.25 29.25 300.50


balance

Useful life ( in years) 10 10 10 10


*1 115 + 5 = 120
*2 45 × 6 ÷ 9 = 30
*3 (80÷5 × 9/12)+(120÷10×3/12)=15

*4 32 – (80÷5×9/12 = 20

© Emile Woolf International 212 The Institute of Chartered Accountants of Pakistan


Answers

10.8 TULIP LIMITED


Total assets
Description Profit

---------- Rs. in million ----------


As per question 398.00 2,700.00
Development cost of January and February wrongly
capitalized [332(692–360)÷9×2] (73.78) (73.78)
Depreciation of plant to be charged off 72(360÷5)×5÷12 (30.00) (30.00)
Amortization of development cost for 3 months (7.51) (7.51)
[258.22 (332÷9×7)+42(72×7÷12)]÷(10×3÷12)]
286.71 2588.71

10.9 ZEBRA LIMITED


Since a part of the payment for the license has been deferred beyond normal credit terms so the
license will be initially recognised at cash price equivalent of Rs. 80 million i.e. Rs. 50 million plus
Rs. 30 million (i.e. present value of Rs. 36.3 million discounted at 10% for 2 years.)
The advertisement cost of Rs. 10 million incurred on launching of the channel cannot be included in
the cost of the license and will be charged to Profit and loss account.
Since the renewal cost is significant so the useful life of the license will be restricted to the original 5
years only.
The residual value of the license will be assumed to be zero since there is no active market for the
license and there is no commitment by 3rd party to purchase the license at the end of useful life.
The amortization for the year will be Rs. 12 million [(80 – 0) × 1/5 ×9/12] calculated from 1 April 2017
when the license was available for use:
Unwinding of interest expense of Rs. 2.25 million (30 × 10% × 9/12) shall be recorded with
increasing the liability of payable for license with same amount.
10.10 SUNSHINE LIMITED
For the year ended 30 June 2017 Rs. in million
Amount to be recognised in SOFP
Intangibles – Licenses (170+300+65+55) 590
Revaluation surplus (W-1) 93

Amount to be recognised in SOPL


Amortization (W-1) 63
Impairment (W-1) 20

W-1: A B C D Total
-------------------------- Rs. in million --------------------------
Cost of licenses 200 230 90 60 580
Amortization for the year (20) (23) (15) (5) (63)
(200÷10) (230÷10) (90÷6) (60÷12)
Cost less amortization 180 207 75 55 517
Active market value No active
170 300 65 market
Impairment (10) - (10) (20)
Revaluation surplus - 93 - - 93

© Emile Woolf International 213 The Institute of Chartered Accountants of Pakistan


Financial accounting and reporting II

CHAPTER 11: IAS 41: AGRICULTURE


11.1 GUJRANWALA FOODS LIMITED
(a)
Chickpea Apricot Dates Onion Total
Rs.’000 Rs.’000 Rs.’000 Rs.’000 Rs.’000
Sales: 200 300 750 250 1,500
Add: Subsidies 40 80 60 60 240
Own consumption 45 40 75 20 180
285 420 885 330 1920
Cost of sales
Opening inventory 40 40 70 - 150
Add: Purchases 75 150 300 80 605
Closing inventory (60) (110) (300) (60) (530)
55 80 70 20 225
Gross output value 230 340 815 310 1,695

Less expenses:
Casual labour 16
Regular workers 24
Land preparation 64
Hire of tractors 48
Depreciation: irrigation 80
Depreciation: farms equipment 60
(292)
1,403

Workings:
Casual labour 12/ x Rs. 20,000 = Rs. 16,000
15

Regular workers 12/ x Rs. 30,000 = Rs. 24,000


15

Land preparation 12/ x Rs. 80,000 = Rs. 64,000


15

Hire of tractors 12/ x Rs. 60,000 = Rs. 48,000


15

Depreciation:
Rs. 800,000
Irrigation cost
10
Farm equipment 20% x 400,000 x 9/12 = Rs. 80,000

(b) (i) Biological assets are living plants and animals.


(ii) Biological transformation relates to the process of growth and degeneration that
can cause changes of a quantitative or qualitative nature in a biological asset.
(iii) Harvest is the detachment of produce from a biological asset or cessation of a
biological asset’s life process.

© Emile Woolf International 214 The Institute of Chartered Accountants of Pakistan


Answers

11.2 HELIOS GROUP


(a) Helios Ltd
Consolidated statement of financial position as at 31 December, 2016
Non-current assets Rs.’000 Rs.’000

Biological assets - Dairy livestock: Immature 40,000


- Dairy livestock: mature 50,000
- Plantation 20,000 110,000
─────
Property, plant and equipment
(600,000 + 450,000 + 60,000 – 40,000 – 50,000) 1,020,000
Investments (800,000 – 600,000) 200,000
Goodwill (W3) 260,000
───────
1,590,000
Current assets
Inventories (160,000 + 150,000 – 20,000) 290,000
Trade receivables (120,000 + 280,000) 400,000
Cash and cash equivalents (20,000 + 50,000) 70,000 760,000
───── ───────
2,350,000
───────
Equity and liabilities
Equity
Ordinary shares capital 160,000
Share premium 40,000
Group reserves (W5) 730,000
───────
Parent equity 930,000
Non-controlling interests (W4) 220,000
───────
Total equity 1,150,000

Non-current Liabilities
Loan notes (600,000 + 170,000) 770,000

Current liabilities
Trade payables (310,000 + 120,000) 430,000 1,200,000
───── ───────
Total equity and liabilities 2,350,000
───────
Workings (W)

(1) Group Structure


Helios Ltd
70%
NCI = 30%
Sol Ltd

© Emile Woolf International 215 The Institute of Chartered Accountants of Pakistan


Financial accounting and reporting II

(2) Net assets of Sol Ltd


Acquisition Reporting Post-
date date acquisition
Rs.’000 Rs.’000 Rs.’000
Ordinary shares 120,000 120,000 -
Share premium 20,000 20,000 -
Reserves 300,000 500,000 200,000
Fair value adjustment (land) 60,000 60,000
Fair value of net assets 500,000 700,000 200,000
(3) Goodwill in Sol Ltd
Rs.’000
Purchase consideration 600,000
Fair value of NCI at acquisition 160,000
760,000
Fair value of net assets at acquired (500,000)
Goodwill 260,000
(4) NCI at reporting date
Rs.’000
Fair value of NCI at acquisition 160,000
Share of post-acquisition reserves
(30% x Rs. 200,000) 60,000
220,000
(5) Group reserves
Rs.’000
Helios Ltd 590,000
Share of post-acquisition (70% x Rs. 200,000) 140,000
730,000
(b) Measurement of harvested agricultural products. Agricultural products harvested from an
entity’s biological assets shall be measured at its fair value less costs to sell at the point of
harvest. Such measurement is the cost at that date when applying it as inventories or
another applicable standard.
11.3 THE DAIRY COMPANY
(i) “Biological asset” is a living animal or plant.
An entity shall recognize a biological asset if all the following conditions are met:
 The entity controls the asset as a result of past event;
 It is probable that future economic benefits associated with the asset will flow to
the entity;
 The fair value or cost of the asset can be measured reliably.
(ii) The new born cows are biological assets and should be measured at fair value less costs
to sell, both on initial recognition and at each reporting period.
 The gains on initial recognition and the gains from change in this value should be
recognized in profit or loss for the period in which it arises. The total gains to be
recognized in the year ended 30 June 2015 is as follows:
Rupees
New born [26,000 × 225 × (100%-2%)] 5,733,000
9 month old [53,000 × 75 × (100% - 2%)] 3,895,500
9,628,500

© Emile Woolf International 216 The Institute of Chartered Accountants of Pakistan


Answers

11.4 FATIMA LIMITED


Fatima Limited
Extracts of Statement of Financial Position
As at 31 December, 2018
Rs. in million
Current assets
Biological assets - Freestanding trees 2,500
Non-current assets
Land under trees 500
Roads in forests 400
3,400

11.5 ZOHA LIMITED

Zoha Limited

Extracts of Statement of Financial Position

As at 31 December, 2019

2019 2018

Rs. in million

Current assets

Biological assets 1,310 1,300

2018: (600 + 700 =1,300)

2019: (1300 +100-90=1,310)

Zoha Limited

Extracts of Statement of Comprehensive Income

For the year ended 31 December, 2019

2019 2018

Rs. in million

Fair value gain on biological assets 100 700

Loss on biological assets due to decrease in harvest (90) -

10 700

© Emile Woolf International 217 The Institute of Chartered Accountants of Pakistan


Financial accounting and reporting II

11.6 MISHALL LIMITED


Biological assets should be measured at each reporting date at fair value less estimated point-of-
sale costs unless fair value cannot be measured reliably. The Standard encourages companies to
separate the change in fair value less estimated point-of-sale costs between those changes due to
physical reasons and those due to price.
Fair value of cattle excluding Lahore region: Rs. 000
Fair value at January 1, 2018
Cows (210,000 – 60,000) × 40 6,000
Heifers (30,000 – 20,000) × 30 300
Purchase 75,000 heifers × 30 2,250
8,550
Increase due to price change
150,000 × (45 – 40) 750
10,000 × (32 – 30) 20
75,000 × (32 – 30) 150
920
Increase due to physical change
150,000 × (50 – 45) 750
10,000 × (45 – 32) 130
75,000 × (36 – 32) 300
1,180 10,650

Fair value less estimated point-of-sale costs at October 31, 20X4


150,000 × 50 7,500
10,000 × 45 450
75,000 × 36 2,700
10,650
Lahore region—fair value of cattle:
This region has an inventory of cattle of 60,000 cows and 20,000 heifers. Fair value is difficult to
ascertain because of the region’s problems. However, according to IAS 41, if fair value was used on
initial recognition, then it should be continued to be used. The cattle in this region would have been
fair valued at January 1, 2018, under the Standard. Therefore, the cattle must be valued at fair value
less estimated point-of-sale costs as at December 31, 2018. Although Rs.3 million has been offered
for these animals, this may be an onerous contract as rival companies are likely to wish to take
advantage of the problems in this region. The future discounted income is again an inappropriate
value as the cattle are healthy and could be moved to another region and sold.
The cattle in this region would
Rs.000
therefore be valued at
60,000 cows × 50 3,000
20,000 heifers × 45 900
3,900

© Emile Woolf International 218 The Institute of Chartered Accountants of Pakistan


Answers

CHAPTER 12: FINANCIAL INSTRUMENTS: RECOGNITION AND MEASUREMENT


12.1 AJI PANCA LTD
Capital and reserves Rs.
Share capital (Rs. 1 ordinary shares) (W2) 1,625,000
Share premium (W3) -
Retained earnings 6,116,812
7,741,812
Liabilities (W5) 164,751

Workings
(1) Profit for the year
Rs.
Original 508,500
Minus: Finance charges (W5) (14,988)
493,512
(2) Ordinary share capital
Rs.
At 1 January 1,000,000
Issue at full price on 31 March 300,000
1,300,000
Bonus issue on 30 June (1,300,000 ÷ 4) 325,000
1,625,000
(3) Share premium
Rs.
At 1 January 200,000
Issue at full price on 31 March ((300,000  0.30) – 20,000) 70,000
270,000
Bonus issue on 30 June (270,000)
NIL
(4) Retained earnings
Rs.
At 1 January 5,670,300
Minus: Bonus issue on 30 June (325,000 (W2) – 270,000 (W3) (55,000)
Add: Profit for the year (W1) 493,512
Add back: Preference dividends charged to retained earnings (W5)
8,000
6,116,812
(5) Redeemable preference shares
Rs.
Liability at beginning of year
Year 1 ((100,000  Rs. 1.60) – 2,237)) 157,763
Finance charge at 9.5% 14,988
Interest paid at 4% (8,000)
Liability at end of year 164,751

© Emile Woolf International 219 The Institute of Chartered Accountants of Pakistan


Financial accounting and reporting II

12.2 PASSILA LTD


(a) The face value of the debentures
Rs. 100 X 20,000 = Rs. 2,000,000
The amount accrued to the company as proceeds = Rs. 97.5 X 20,000 = Rs. 1,950,000
(b) The difference between the face value and the market value of the debentures is Rs. 50,000.
This is as a result of discount allowed on the issue on the debentures. Discount on
debentures attracts investors.
(c) Nominal interest rate is the rate based specifically on the face value of the loan capital. In
case of Passila Ltd., the nominal interest rate on the debentures is 8% per annum on Rs.
2,000,000.
The effective interest is the rate based on the market value. This is the actual value collected
on issue which can be at par, discount or premium. For Passila Ltd., the effective interest
rate will be 8% of Rs. 1,950,000
(d) The nominal interest payable
Rs. 2,000,000 X 8% X 6 months ÷ 12 months
= Rs. 80,000
(e) (i) The face value of Rs. 2,000,000 will be the most appropriate valuation
to be disclosed in the Statement of financial position. The management may be
interested in the quoted market value or the proceeds, but for the sake of outside
investors who would only be interested in the company having good reputations devoid
of trading losses, it is advisable that the face value be adopted.
(ii) Disclosing the debentures’ liability at face value plus interest payment for five years
may seem proper in the eyes of external investors and credit institutions, but principally,
it would be wrong to credit debentures’ account with both the face value and the
interest payments. An interest payment on debentures is a revenue item which is
debited to the Profit and Loss Account.
(iii) Disclosing debentures’ liability at market value on the Statement of financial position will
amount to disclosure at replacement value. The market value should be disclosed.
12.3 RASHID INDUSTRIES LIMITED
Investment in KL
Initial measurement
According to IFRS 9, at initial recognition, RIL may make irrevocable election to present subsequent
changes in fair value in equity investment in other comprehensive income instead of profit or loss
account.
If RIL opted as above, investment in KL would initially be recognized at fair value plus transaction
costs i.e. Rs. 20 million.
However, if RIL opted to measure the investment at fair value through profit and loss (FVTPL),
investment should initially be measured at Rs. 19.96 million (20/1.002) and transaction costs of Rs.
0.04 million (20–19.96) should be charged to profit and loss account.
Subsequent measurement
On 31 December 2016, if fair value through other comprehensive income has been opted,
investment in KL should be measured at fair value of Rs. 12.4 million and a loss of Rs. 7.6 million
[20–12.4(155,000×80)] (instead of Rs. 5 million) should be booked through other comprehensive
income.
According to IFRS 9, amount presented in other comprehensive income shall not be subsequently
transferred to profit or loss. However, the entity may transfer the cumulative gain / (loss) within
equity.

© Emile Woolf International 220 The Institute of Chartered Accountants of Pakistan


Answers

If fair value through profit or loss has been opted, then RIL should account for the loss of Rs. 7.56
million (20–0.04(transaction cost)–12.4) through profit and loss account.
Investment in BL
Initial measurement
The investment in BL should be recognized as held for trading at fair value of Rs. 64.87 million
(65÷1.002) and transaction cost of Rs. 0.13 million should be charged to profit and loss account.
Subsequent measurement
As at 30 November 2016, the investment should be re-measured to fair value at the market price of
Rs. 83.835 million (135,000×621) and a gain of Rs. 18.965 million (83.835–64.87) shall be booked
in the profit and loss account.
Reclassification of asset
On 30 November 2016 when RIL decided to hold the shares for a longer period, investment in BL
should be reclassified from held for trading to non-trading investment. Further, RIL may make
irrevocable election that investment in BL would be re-measured at fair value through other
comprehensive income, as discussed in the case of KL above. Similarly, treatment on 31 December
2016 would depend on whether RIL opted to re-measure at fair value through OCI or not.

© Emile Woolf International 221 The Institute of Chartered Accountants of Pakistan


Financial accounting and reporting II

CHAPTER 13 – IFRS 16: LEASES


13.1 X LTD
(a)

A B C D E
Period Opening Fin. Charge Rentals Closing Balance
Balance at 15% of B (B – (D - C)
Rs.’000 Rs.’000 Rs.’000 Rs.’000
2016 11,420 1,713 4,000 9,133
2017 9,133 1,370 4,000 6,503
2018 6,503 975 4,000 3,478
2019 3,478 522 4,000
──── ────
4,580 16,000
──── ────
(b)
Statement of Financial Position (Extract) as at 31 December 2016

Rs.’000
Non-Current assets (Rs.11,420,000 – Rs.2,855,00) 8,565
Non-Current Liabilities (Obligation under lease) 6,503
Current Liabilities Obligation under lease
(Rs.9,133,000 – Rs.6,503,000) 2,630

11,420,000
Note: Annual Depreciation = = Rs.2,855,000
4
13.2 PROGRESS LTD
(a) Annuity method

Year 1 Year 2 Year 3


Rs. Rs. Rs.
Cash flow 3,200,000 - -
Outstanding - 1,920,000 1,350,400
Capital repayment 1,280,000 569,600 637,952

Balance 1,920,000 1,350,400 712,448

Interest @ 12% of balance 230,400 162,048 85,494


Capital repayment 569,600 637,952 714,506
800,000 800,000 800,000

© Emile Woolf International 222 The Institute of Chartered Accountants of Pakistan


Answers

(b) Journal entries

Dr Cr
Rs. Rs.
2016
Jan. 3 Right of use - Plant and machinery 3,200,000
Fine Rentals Limited 3,200,000
Initial recognition of machine
Jan. 3 Fine Rentals Limited 1,280,000
Bank 1,280,000
Payment of initial deposit under lease
Dec. 31 Fine Rentals Limited 569,600
Interest expense 230,400
Bank 800,000
Apportionment of annual installment
between Principal repayment and
interest
Dec. 31 Profit and Loss Account 230,400
Interest Expense 230,400
Write-off of FL interest expense to
Profit and loss account
2017
Dec. 31 Fine Rentals Ltd 637,952
Interest expense 162,048
Bank 800,000
Apportionment of annual installment
for the year between Principal
repayment and interest
Dec. 31 Profit and Loss Account 162,048
Interest Expense 162,048
Write-off of FL interest expense to
Profit and loss account
2018
Dec. 31 Fine Rentals Limited 714,506
Interest expense 85,494
Bank 800,000
Apportionment of annual installment
for the year between Principal
repayment and interest
Dec. 31 Profit and Loss Account 85,494
Interest Expense 85,494
Write-off of FL interest expense to
Profit and loss account

© Emile Woolf International 223 The Institute of Chartered Accountants of Pakistan


Financial accounting and reporting II

13.3 MIRACLE TEXTILE LIMITED


Miracle Textile Limited
Statement of financial position (extracts) as at 30 June 2016
Note 2016 2015
ASSETS Rs. Rs.
Non-current assets
Right of use - Machinery 4 16,000,000 18,000,000

LIABILITIES
Non-current liabilities
Obligation under lease 9 6,505,219 10,633,074

Current liabilities
Current portion of obligation 9 4,127,856 3,566,925

Miracle Textile Limited


Notes to the financial statements (extracts) for the year ended 30 June 2016
4- Right of use assets 2016 2015
Leased assets
Cost
Opening balance 20,000,000 -
Addition during the year - 20,000,000
20,000,000 20,000,000
Accumulated depreciation
Opening balance (2,000,000) -
Depreciation for the year (2,000,000) (2,000,000)
(4,000,000) (2,000,000)
Balance as at 30 June 16,000,000 18,000,000

W1: Lease Schedule


Opening Principal Interest @ Closing
Payment date Instalment
principal repayment 15.73% principal
01-Jul-14 20,000,000 5,800,000 5,800,000 - 14,200,000
01-Jul-15 14,200,000 5,800,000 3,566,925 2,233,075 10,633,075
01-Jul-16 10,633,075 5,800,000 4,127,856 1,672,144 6,505,219
01-Jul-17 6,505,219 5,800,000 4,776,997 1,023,003 1,728,222
30-Jun-18 1,728,222 2,000,000 1,728,222 271,778 -
20,000,000 5,200,000
20- MATURITY ANALYSIS – CONTRACTUAL UNDISCOUNTED CASH FLOWS
Rs.
Not later than one year 5,800,000
Later than one year but not later than five years 7,800,000
13,600,000

© Emile Woolf International 224 The Institute of Chartered Accountants of Pakistan


Answers

13.4 ACACIA LTD


Relevant extracts
Statements of profit or loss for the year ended 31 March 2016 (extracts)
Rs.
Depreciation (272,850 ÷ 6) 45,475
Lease payments 6,000*
Finance costs (W) 19,460
* Considering low value item as described in IFRS16
Statement of financial position as at 31 March 2016 (extracts)
Rs.
Non-current assets
Right of use(272,850 – 45,475) 227,375
Non-current liabilities
Lease liabilities 135,810
Current liabilities
Lease liabilities 78,250
Statement of cash flows for the year ended 31 March 2016 (extracts)
Cash flows from financing activities
Payment of lease liabilities (78,250)
Notes to the financial statements (extracts)
(1) Maturity Analysis – Contratual undiscounted cash flows
Rs.
One year 78,250
Two to five years (2 x 78,250) 156,500
234,750

13.5 SHOAIB LEASING LIMITED


(a) Entries in the books of Lessor
Date Particulars Dr. Cr.
1-Jul-16 Lease payments receivable (W1) 2,680,000
Machine 2,100,000
Unearned finance income (W1) 580,000
30-Jun-17 Bank 860,000
Lease payments receivable 860,000
30-Jun-17 Unearned finance income 272,941
Finance income (W2) 272,941
30-Jun-18 Bank 860,000
Lease payments receivable 860,000
30-Jun-18 Unearned finance income 196,640
Finance income (W2) 196,640
30-Jun-19 Bank 960,000
Lease payments receivable 960,000
30-Jun-19 Unearned finance income 110,419
Finance income (W2) 110,419

© Emile Woolf International 225 The Institute of Chartered Accountants of Pakistan


Financial accounting and reporting II

W1: Total finance income Rs.


Total future lease payments (Rs.860,000 x 3) 2,580,000
Add: Purchase bargain option 100,000
Gross investment in finance lease 2,680,000
Less: Cost of assets 2,100,000
Total finance income 580,000

W2: Amortization schedule


Principal Principal
Instalment Interest Principal
Date Opening Closing
Rs.
30-Jun-17 2,100,000 860,000 272,941 587,059 1,512,941
30-Jun-18 1,512,941 860,000 196,640 663,360 849,581
30-Jun-19 849,581 960,000 110,419 849,581 nil
580,000 2,100,000

(b) Shoaib Leasing Limited


Extracts from the statement of financial position as at June 30, 2017
Note 2017
Rs.
Non-current Assets
Net investment in leases 10 849,578
Current Assets
Current portion of net Investment in leases 663,360

10 Net investment in leases


Lease payments receivables 10.1 1,720,000
Add: Residual value of leased assets (part
of LP) 100,000
Gross Investments in leases 1,820,000
Less: Unearned lease income (307,062)
Net investment in leases 10.2 1,512,938
Less: Current portion of net investment in leases (663,360)
849,578

Maturity Analysis – Contractual


10.1 undiscounted cash flows
Less than one year 860,000
More than one year and less than 5 years 960,000
1,820,000

© Emile Woolf International 226 The Institute of Chartered Accountants of Pakistan


Answers

13.6 SHALIMAR INDUSTRIES


(a)
Date Description Debit (Rs.) Credit (Rs.)
1-Jan-15 Lease receivable [Rs. 20,000,000 + Rs. 26,501,816
6,501,817(W-1)]
Cost of Goods sold
15,364,482
Sales (Lower of FV i.e. Rs. 20m or PV
of MLP i.e. 6,375,454 × 3.03735) 19,364,481
Inventory (Rs. 1.6m × 10) 16,000,000
Unearned finance income (W-1) 6,501,817
31-Dec-15 Bank 6,375,454
Lease Receivable 6,375,454

31-Dec-15 Unearned finance income 2,400,000


Finance income 2,400,000

W-1: Amortization Schedule

Interest Net Gross


Lease
Principal investment in investment
Year ended installment @ 12 lease in lease

---------------------------------- Rupees ----------------------------------

12% 20,000,000 26,501,816

31-Dec-15 6,375,454 2,400,000 3,975,454 16,024,546 20,126,362

31-Dec-16 6,375,454 1,922,946 4,452,508 11,572,038 13,750,908

31-Dec-17 6,375,454 1,388,645 4,986,809 6,585,228 7,375,454

31-Dec-18 6,375,454 790,227 5,585,227 1,000,000 1,000,000

6,501,817

(b) Disclosure in the financial statements

1 Net investment in lease

2015
Rupees
Lease receivable (Rs. 6,375,454 × 3) 19,126,362
Add: Unguaranteed residual amount 1,000,000
Gross investment in lease 20,126,362
Less: Unearned finance income (6,501,817 – 2,400,000) (4,101,817)
Net investment in finance lease 16,024,545

© Emile Woolf International 227 The Institute of Chartered Accountants of Pakistan


Financial accounting and reporting II

1.1 Maturity analysis (contractual undiscounted cash flows)

2015
Rupees
Not later than one year 6,375,454
Later than one year but not later than five years 13,750,908
20,126,362

1.2 The minimum lease payment has been discounted on interest rate of 12% to arrive at
their present value. Rentals are paid annually in arrears.
13.7 GUAVA LEASING LIMITED (GLL)
Guava Limited
Notes to the financial statements
For the year ended 30 June 2018
Rs. in million
Net investment in lease:
Lease payments receivables [(48×5)+(15×3)] 285.00
Residual value of machinery 20.00
Gross investment in lease 305.00
Unearned lease income (Bal.) (51.65)
Net investment in lease (W-1) 253.36
Current portion of net investment in lease (Bal.) (72.43)
(W-1) 180.92
Maturity analysis - contractual undiscounted cash flows
Less than one year (48×2) 96.00
One to two years (48×2) 96.00
Two to three years (48+15) 63.00
Three to four years [(15×2)+20] 50.00
305.00
W-1: Amortization Schedule
Installment Interest Closing
Date
--------------------- Rs. in million ---------------------
1-Jul-17 319.06
31-Dec-17 48.00 (15.95) (287.01)
30-Jun-18 48.00 (14.35) (253.36)
31-Dec-18 48.00 (12.67) (218.03)
30-Jun-19 48.00 (10.90) (180.92)
W-2: Net investment in lease on 1 July 2017
Rs. in million
PV of Rs. 48 million over 7 installment [48×5.7865{(1–1.05–7)÷0.05}] 277.75
PV of Rs. 15 million over 3 installment [15×{(1–1.05–3)÷0.05}×1.05–7] 29.03
PV of Rs. 20 million of UGRV [20×1.10–5] 12.28
319.06

© Emile Woolf International 228 The Institute of Chartered Accountants of Pakistan


Answers

CHAPTER 14 – OTHER AREAS OF IFRSs (IFRS 8, IAS 10, IAS 37)


14.1 BADAR
Decommissioning costs
IAS 37 Provisions, Contingent Liabilities and Contingent Assets only permits a provision to be made
if three conditions are met:
(i) The company has a present obligation, either legally or constructively, as a result of a past
event;
(ii) Probable outflow of resources is required to settle the obligation; and
(iii) A reliable estimate is available.
Although there is no legal requirement to restore the site, the company has established a
constructive obligation by setting a valid expectation in the market, due to its published policies and
past practice, from which it cannot realistically withdraw.
It therefore appears probable that Badar will have to pay money to improve the site and so a
provision should be created for the expected amount. As the expected payment of Rs.100,000 will
not be settled for three years, the provision should be discounted and entered at its net present
value of Rs.75,131 (Rs.100,000/(1.1)3). Over the three years, the discounting should be unwound
and charged to profit or loss as finance costs, resulting in a provision of Rs.100,000 by the end of
the third year.
The cost of the construction work has been correctly capitalised. The cost of the future
decommissioning work should be added to this asset so that the total costs of the site can be
matched to the revenue from the copper over the period of mining. This will result in an asset of
Rs.575,131 which should be depreciated over the three year life in line with anticipated revenues.

14.2 GEORGINA
(1) Litigation for damages
Under IAS37, a provision should only be recognised when:
 an entity has a present obligation as a result of a past event
 it is probable that an outflow of economic benefits will be required to settle the
obligation
 a reliable estimate can be made of the amount of the obligation.
Applying this to the facts given:
 Georgina’s legal advisors have confirmed that there is a legal obligation. This arose
from the past event of the sale, on 1 September 2015 (i.e. before the year end).
 Probable is defined as ‘more likely than not’. The legal advisors have confirmed that it is
likely that the claim will succeed.
 A reliable estimate of Rs.500,000 has been made.
Therefore a provision of Rs.500,000 should be made.
Counter-claim
IAS37 requires that such a reimbursement should only be recognised where receipt is ‘virtually
certain’. Since the legal advisors are unsure whether this claim will succeed no asset should be
recognised in respect of this claim.

© Emile Woolf International 229 The Institute of Chartered Accountants of Pakistan


Financial accounting and reporting II

(2) Claim for unfair dismissal


In this case, the legal advisers believe that success is unlikely (i.e. possible rather than
probable). Therefore this claim meets the IAS37 definition of a contingent liability:
 a possible obligation
 arising from past events
 whose existence will be confirmed only by the occurrence or non-occurrence of one
or more uncertain future events.
The liability is a possible one, which will be determined by a future court case or tribunal. It did
arise from past events (the dismissal had taken place by the year end).
This contingent liability should be disclosed in the financial statements (unless the legal advisors
believe that the possibility of success is in fact remote, and then no disclosure is necessary).
(3) Returns
Applying the IAS37 conditions in (1) to the facts given:
 Although there is no legal obligation, a constructive obligation arises from Georgina’s
past actions. Georgina has created an expectation in its customers that such refunds
will be given.
 As at the year end, based on past experience, an outflow of economic benefits is
probable.
 A reliable estimate can be made. This could be 1% × 400,000 but since the returns are
now all in the actual figure of Rs.3,500 can be used.
Therefore a provision of Rs.3,500 should be made.
(4) Closure of division
Applying the above IAS37 conditions in (1) to the facts given:
 A present obligation exists because at the year end there is a detailed plan in place and
the closure has been announced in the press.
 An outflow of economic benefits is probable.
 A reliable estimate of Rs.300,000 has been made.
However, IAS37 specifically states in respect of restructuring that any provision should include
only direct expenses, not ongoing expenses such as staff relocation or retraining. Therefore a
provision of Rs.250,000 (300,000 – 50,000) should be made.

14.3 EARLEY INC


(a) IAS 10 (revised) Events After the Statement of financial position Date states that assets and
liabilities should be adjusted for events occurring after the statement of financial position date
that provide additional evidence relating to conditions existing at the statement of financial
position date. It specifically includes the example of bad debts, where evidence of bankruptcy
of a debtor occurs after the year end.
In this case, Nedengy appears to have recovered part of the debt and as such only
Rs.200,000 needs to be provided. It may be argued that the receivership has occurred as a
result of events occurring after the statement of financial position date, as a result of a change
in legislation for example, but this is unlikely.
IAS 18 Revenue states that when uncertainty arises about the collectability of an amount
already included in revenue, the amount should be recognised as an expense.
(b) It is likely that the fall in the value of the property will fit the IAS 10 (revised) definition of
adjusting events noted in (a) above, unless, again, it can be argued that the decline in the
property market occurred after the year-end.

© Emile Woolf International 230 The Institute of Chartered Accountants of Pakistan


Answers

IAS 36 Impairment of assets and IAS 16 Property, Plant and Equipment require that the
carrying amount of property, plant and equipment should be reviewed periodically in order to
assess whether the recoverable amount has fallen below the carrying amount. Where it has,
the property, plant and equipment should be written down to the recoverable amount, either
through the statement of profit or loss as an expense, or though other comprehensive income
to revaluation reserve in shareholder’s equity, but only to the extent that the balance on the
revaluation reserve relates to a previous revaluation surplus on the same asset.
(c) IAS 2 Inventories requires that inventories be stated at the lower on cost and net realisable
value. Net realisable value is the estimated selling price in the ordinary course of business
less the estimated costs of completion and the estimated costs necessary to make the sale.
Unless Earley was making a significant margin on the tricycles, it is likely that the reduction in
selling price of 30% will necessitate a write- down to net realisable value, especially
considering the transportation costs to Iraq which must be included. If the Iraqi option is
unlikely to proceed, it may be necessary to write the tricycles down to scrap value.
(d) Under IAS 10, the nationalisation is likely to be regarded as a non-adjusting event that merely
requires disclosure in the financial statements. IAS 27 Consolidated Financial Statements
and Accounting for Investments in Subsidiaries, requires that an investment in a enterprise
should be accounted for as an investment (under IAS 39: Financial Instruments: Recognition
and Measurement) from the date that it ceases to fall within the definition of a subsidiary and
does not become an associate. It seems here that Earley has neither control nor significant
influence, nor even an investment as the assets have been in fact, expropriated. The loss of
the investment should be accounted for in the year in which it occurred, but disclosed in the
current year.
If the loss of the subsidiary results in Earley no longer being a going concern, then the event
becomes an adjusting event and both of the events described are non-adjusting event which
should be disclosed, but not adjusted for in the current year financial statements.

14.4 ACCOUNTING TREATMENTS


(a) IAS 37 Provisions contingent liabilities and contingent assets states that contingent gains
should not be recognised as income in the financial statements. The company has a debit
balance already in its books which indicates that it must be reasonably certain that at least
part of the claim will be paid. This element of the claim then is probably not a contingency at
all. The remaining part (the difference between the Rs.15,000 and the Rs.18,600) is, and
should be disclosed and not accrued.
(b) IAS 16 Property, Plant and Equipment requires that the carrying amount of property, plant
and equipment should be reviewed periodically in order to assess whether the recoverable
amount has fallen below the carrying amount. Where it has, the property, plant and equipment
should be written down to the recoverable amount through the statement of profit or loss as
an expense. In this case this would result in the recognition of an expense of Rs.200,000.
(280,000 – 80,000).
It may be the case that the amounts involved are so significant as to warrant separate
disclosure in the statement of profit or loss under IAS 8 Net Profit of Loss for the Period,
Fundamental Errors and Changes in Accounting Policies.
(c) IAS 37 states that contingent liabilities should not be recognised. Though a provision should
be made for amounts where the company has an obligation to pay them.
The question in this case is whether or there is an obligating event within the context of IAS
37. On balance it seems inappropriate to recognise a provision in respect of this amount but
the possible liability should be disclosed as a contingent liability.
(i) the nature of the contingency
(ii) the uncertainties surrounding the ultimate outcome
(iii) the likely effect, ie Rs.500,000 loss less likely tax relief.

© Emile Woolf International 231 The Institute of Chartered Accountants of Pakistan


Financial accounting and reporting II

(d) IAS 2 Inventories requires that inventories be stated at the lower on cost and net realisable
value. Net realisable value is the estimated selling price in the ordinary course of business
less the estimated costs of completion and the estimated costs necessary to make the sale.
In this case, cost is Rs.1,800 and net realisable value is Rs.1,600
(e) The company should set up a provision for Rs.100,040, ie should accrue for the 10% probable
liability. It should disclose the possible liability under contingent liabilities. The disclosure is as
noted in (c) except that the financial effect is Rs.300,120 (30%  Rs.1,000,400). The balance
should be ignored as it is a remote contingent liability.
Tutorial note
In (c) above it is not appropriate to provide for 20% receivable Rs.500,000, ie Rs.100,000. This
would only be appropriate where the event is recurring many times over.
In (e) it is appropriate to use the percentages provided, as warranty work is provided for.

14.5 J-MART LIMITED


(a) Adjusting events:
Adjusting events are events that provide further evidence of conditions that existed at the
reporting date.
Examples of adjusting events include:
(i) The subsequent determination of the purchase price or of the proceeds of sale of non-
current assets purchased or sold before the year end.
(ii) The renegotiation of amounts owing by customers or the insolvency of a customer
(iii) Amounts received or receivable in respect of insurance or the insolvency of a
customer.
(iv) The settlement after the reporting date of a court case that confirms that the entity had
a present obligation at the reporting date.
(v) The receipt of the information after the reporting date indicating that an asset was
impaired at the reporting date.
(vi) The discovery of fraud or errors that show that the financial statements are incorrect.

Non-adjusting events:
Non-adjusting events are indicative of conditions that arose subsequent to the reporting date.
Examples of non-adjusting events might be:
(i) Losses of non-current assets or inventories as a result of a catastrophe such as fire
or flood
(ii) Closing a significant part of the trading activities if this was not begun before the year
end
(iii) The value of an investment falls between the reporting date and the accounts are
authorised
(iv) Announcement of dividend after year end.
(b) (i) The conditions attached to the sale give rise to a constructive obligation on the
reporting date.
A provision for the sales return should be recognised for 5% of June 2015 sales. The
related cost should also be reversed.
(ii) Since the law suit was already in progress at year-end and the amount of
compensation can also be estimated, it is an adjusting event.
A provision of Rs. 400,000 should be made.

© Emile Woolf International 232 The Institute of Chartered Accountants of Pakistan


Answers

(iii) There is no obligating event at the year end either for the costs of fitting the smoke
detectors or for fines under the legislation.
No provision should be recognised in this regard.
(iv) The obligating event is the communication of decision to the customers and
employees, which gives rise to a constructive obligation from that date, because it
creates a valid expectation that the division will be closed.
Since no communication has yet been made, no provision is required in this regard.
(v) The obligating event is the signing of the lease contract, which gives rise to a legal
obligation.
A provision is required for the unavoidable rent payments.
(vi) Since the declaration was announced after year-end, there is no past event and no
obligation at year-end and is therefore non-adjusting event.
Details of the dividend declaration must, however, be disclosed.

14.6 AKBER CHEMICALS LIMITED


(a) The event is an accident, and since it happened before the year end, it is a past event.
However, there is no present obligation since:
(i) there is no law requiring the company to clean the canal.
(ii) there is no constructive obligation to clean the river since:
 a public statement has not been made;
 there is no established pattern of past practice as this was the first time the
company faced such a situation.
Although the company has decided to clean up the river and even has a reliable estimate of
the costs thereof, no liability or provision should be recognised in the current year because:

 the decision was taken after year end; and

 the decision was not yet made public.


(b) It is a non-adjustable event because the event due to which the net realizable value (NRV) of
stock has fallen, arose after the reporting date.
However, if this event is material, the company should disclose the decline in NRV in its
financial statement for the year ended June 30, 2015.
(c) The company should make the provision because:
(i) the company has a present obligation because of past event
(ii) the claim of the customer is valid and is confirmed by the company's inspection team
which shows that an outflow will be required to settle the obligation.
(iii) the amount of outflow is reliably estimated i.e. Rs. 2 million.
Since the company is certain of recovery from the vendor, it should:
(i) disclose it as a separate asset.
(ii) recognise a receivable but the same should not exceed the amount of the related
provision i.e. rs. 2.0 million.

© Emile Woolf International 233 The Institute of Chartered Accountants of Pakistan


Financial accounting and reporting II

14.7 QALLAT INDUSTRIES LIMITED


(i) Provision must be made for estimated future claims by customers for goods already sold.
The expected value i.e. Rs. 10 million ([Rs. 150m x 2%] + [Rs. 70m x 10%]) is the best estimate
of the provision.
(ii) Warehouse A: It is an onerous contract. as the warehouse has been sublet at a loss of Rs.
200,000 per month. QIT should therefore create a provision for the onerous contract that arises
on vacating the warehouse. This is calculated as the excess of unavoidable costs of the
contract over the economic benefits to be received from it. Therefore, QIL should immediately
provide for the amount of Rs. 13.2 million. [5.5 years x 12 month x Rs. 200,000] in its financial
statements i.e. for the year ended June 30, 2015.
Warehouse B: It is not an onerous contract because the warehouse has been sublet at profit.
Hence this would require no adjustment.
(iii) A provision is to be made by QIL against a contingent liability as:
(a) There is a present obligation (legal or constructive) as a result of a past event; i.e.
accident occurred on June 15, 2015.
(b) It is probable that outflow of resources will be required to settle the obligation; and
(c) A reliable estimate can be made of the amount of the obligation.
The amount of provision shall be Rs. 2.0 million i.e. the most probable amount as determined
by the lawyer.
(iv) A provision of Rs. 0.4 million is required in relation to penalty for March 1 to June 30, 2015
because at the reporting date there is a present obligation in respect of a past event.
The reimbursement of penalty amount from the vendor shall be recognised when and only
when it is virtually certain that reimbursement will be received if the entity settles the obligation.
The reimbursement should be treated as a separate asset in the statement of financial position.
However, in profit and loss statement, the expense relating to a provision may be netted off
with the amount recognised as recoverable, if any.

14.8 SKYLINE LIMITED


(i) Although the debt owing by the customer existed at the reporting date, the customer’s inability
to pay did not exist at that point. This condition only arose in January 2016 after the fire.
Thus, this is a non-adjusting event. However, if it is material for the financial statements, the
following disclosure should be made.
 Nature of the event
 An estimate of its financial effect
(ii) The amount withdrawn before year end i.e. Rs. 1.5 million is an adjusting event as although it
was discovered after year end it existed at the year end. However, since 60% has been
recovered subsequently, Rs. 0.6 million would be provided.
The further withdrawal of Rs. 6.0 million is a non-adjusting event as it occurred after year end.
However, if the events are considered material the following disclosures should be made:
 Nature of the event
 The gross amount of contingency
 The amount recovered subsequently
(iii) SL should not recognise the contingent gain until it is realised. However, if recovery of damages
is probable and material to the financial statements, SL should disclose the following facts in the
financial statements:
 Brief description of the nature of the contingent asset
 An estimate of the financial effect.

© Emile Woolf International 234 The Institute of Chartered Accountants of Pakistan


Answers

(iv) SL should make a provision of the expected amount i.e. Rs. 1.2 million (Rs. 1.0 million x 60% +
Rs. 1.5 million x 40%) because
 it is a present obligation as a result of past event;
 it is probable that an outflow of resources embodying economic benefits will be required
to settle the obligations; and
 a reliable estimate can be made of the amount.
In addition, SL should disclose the following in the notes to the financial statements:
 Brief nature of the contingent liability
 The amount of contingency
 An indication of the uncertainties relating to the amount or timing of any outflow.

14.9 WALNUT LIMITED


(i) This is an adjusting post reporting event as it provides evidence of conditions that existed at the
end of the reporting period. The reasons for the competitor’s price reduction will not have arisen
overnight, but will normally have occurred over a period of time, may be due to superior
investment in technology.
An inventory write down of Rs. 2.5 million should be recognised and the amount included as
inventory on the Statement of Financial Position reduced to Rs. 12.5 million.
(ii) The provision should be recognised because the obligating event is the communication of event
to the public which creates a valid expectation that the division will be closed.
However, the provision should only be recognised to the extent of redundancy costs. IAS
prohibits the recognition of future operating losses, staff training and profits on sale of assets.
(iii) This is a non-adjusting event because the burglary and theft of consumable stores occurred
after reporting date. However, if the event is material, it should be disclosed in the financial
statements unless the loss is recoverable from the insurance company.
(iv) The drop in value of investment in shares is a non-adjusting event. Since the legislation was
announced after the reporting date, the event is not a past event. However, if the amount is
material, it should be disclosed in the financial statements.
(v) This is an adjusting event as it provides evidence of conditions that existed at the end of the
reporting period. The insolvency of a debtor and the inability to pay usually builds up over a
period of time and it can therefore be assumed that it was facing financial difficulty at year-end.
A bad debts expense of Rs. 1.5 million should be recognised in SOCI.
(vi) It is a non-adjusting event because the declaration was announced after the year-end and there
was no obligation at year end. Details of the bonus shares declaration must, however, be
disclosed.

14.10 ATTOCK TECHNOLOGIES LIMITED


(i) Since the event which caused the inventory to be sold at a loss occurred after the year end, it
is non-adjusting event. However, the effect of the event should be disclosed in the financial
statements for the year ended June 30, 2015.
(ii) It is an adjusting event in accordance with the requirement of IAS-10. The debtor’s balance
should be written down by 80% amount.
(iii) It is non-adjusting event as the subsequent reduction in price is due to an event, introduction
of competitive product, occurred after the reporting period.
(iv) Since this change was not enacted before the reporting date, it is a non-adjusting event.
However, a disclosure should be made for this change.
(v) Since the declaration was announced after the year-end and there was no obligation at year-
end it is a non-adjusting event. Details of the dividend declaration must, however, be disclosed.

© Emile Woolf International 235 The Institute of Chartered Accountants of Pakistan


Financial accounting and reporting II

14.11 SHAZAD INDUSTRIES LTD


(a) The purposes of segmental information are:
(i) to provide users of financial statements with sufficient details for them to be able to
appreciate the different rates of profitability, different opportunities for growth and
different degrees of risk that apply to an entity’s classes of business and various
geographical locations.
(ii) to appreciate more thoroughly the results and financial position of the entity by
permitting a better understanding of the entity’s past performance and thus a better
assessment of its future prospects.
(iii) to create awareness of the impact that changes in significant components of a business
may have on the business as a whole.
b) IFRS 8 defines an operating segment as a component of an entity:
 that engages in business activities from which it may earn revenues and incur
expenses (including revenues and expenses relating to transactions with other
components of the same entity).
 whose operating results are regularly reviewed by the entity’s chief operating decision-
maker to make decisions about resources to be allocated to the segment and assess its
performance.
 for which discrete financial information is available.
In order to identify the separate reportable segments, the following criteria should be
adopted:
(i) The reported revenue of the segment in Shazad Industries Ltd, including both
sales to external customers and inter-segment sales, is ten percent or more of
the combined revenue of its four operating segments.
(ii) The Assets of the segment in Shazad Industries Ltd are ten percent or more of
the combined assets of its four operating segments.
(iii) The reported profit or loss of the segment in Shazad Industries Ltd should be ten
percent or more of the greater, in absolute amount, of:
 the combined reported profit of all its operating segments that did not report a loss and
 the combined reported loss of all operating segments that reported losses.
(c) IFRS 8 lays down some very broad and inclusive criteria for reporting segments. Unlike
earlier attempts to define segments in more quantitative terms, segments are defined largely
in terms of the breakdown and analysis used by management. This is, potentially, a very
powerful method of ensuring that preparers provide useful segmental information.
There will still be problems in deciding which segments to report, if only because management
may still attempt to reduce the amount of commercially sensitive information that they
produce.
The growing use of executive information systems and data management within businesses
makes it easier to generate reports on an ad hoc basis. It would be relatively easy to provide
management with a very basic set of internal reports and analyses and leave the individual
managers to prepare their own more detailed information using the interrogation software
provided by the system.
If such analyses become routine then they would be reportable under IFRS 8, but that would
be very difficult to check and audit.
There are problems in the measurement of segmental performance if the segments trade with
each other. Disclosure of details of inter-segment pricing policy is often considered to be
detrimental to the good of a company. There is little guidance on the policy for transfer
pricing.
Different internal reporting structures could lead to inconsistent and incompatible segmental
reports, even from companies in the same industry.

© Emile Woolf International 236 The Institute of Chartered Accountants of Pakistan


Answers

14.12 AZ
(i) Usefulness of segmental data

Many entities carry out several classes of business and operate in a number of
countries across the world. Each of these businesses and geographical segments
carries with it different opportunities for growth, different rates of profit and varying
degrees of risk. Some business segments may be strongly influenced by the health of
the economy whereas other segments may be unaffected by recession. One country
may be experiencing growth; another country may be less stable because of political
events. Awareness of these cultural and environmental differences is important to
investors in order to allow them to fully understand the performance and position of the
entity over the past, its prospects for the future and the risks that it faces.

IFRS 8 requires that segmental information should be provided to enable investors to


understand the impact that the different segments of a business may have on the
business as a whole. If the user of financial statements is only provided with figures for
the entity as a whole, this might hide the risks and problems or profits and opportunities
of the underlying business segments. The disaggregated financial information provided
by segmental reporting allows for analytical review on a segment by segment basis
which will provide greater understanding of the entity’s position and performance and
allow a better assessment of its future.

(ii) Analysing segments

IFRS 8 defines an operating segment as a component of an entity that engages in


business activities from which it may earn revenues and incur expenses, whose
operating results are reviewed regularly by the chief operating decision maker in the
entity and for which discrete financial information is available.

Not every part of a business is necessarily an operating segment or part of an operating


segment. Head office is an example, since head office does not usually earn revenues.
Generally an operating segment has a segment manager who is directly accountable to
and maintains regular contact with the chief operating decision-maker, to discuss the
performance of the segment.

IFRS 8 requires that entities should report information about each operating segment
that is identified and that exceeds certain quantitative thresholds for size of revenue,
operating profit or loss or assets. Financial information about operating segments with
similar characteristics can be aggregated.

IFRS 8 sets out the information about each reportable operating segment that should
be disclosed, including total assets, profit or loss, revenue from external customers,
revenue from sales to other segments, interest income and expense, depreciation,
material items of income or expense and tax. The amount reported for each item should
be the same measure that is reported for the segment to the chief operating decision
maker of the entity.

IFRS8 applies to quoted companies only.


14.13 ROWSLEY
Introduction
All four scenarios relate to the rules of IAS 37 Provisions, contingent liabilities and contingent
assets. In each scenario, the key issue is whether or not a provision should be recognised.
Under IAS 37, a provision should only be recognised when three conditions are met:
 there is a present obligation as a result of a past event; and
 it is probable that a transfer of economic benefits will be required to settle the obligation; and
 a reliable estimate can be made of the amount of the obligation.

© Emile Woolf International 237 The Institute of Chartered Accountants of Pakistan


Financial accounting and reporting II

Factory closure
As the factory closure changes the way in which the business is conducted (it involves the relocation
of business activities from one part of the country to another) it appears to fall within the IAS 37
definition of a restructuring.
The key issue here is whether the group has an obligation at the end of the reporting period to incur
expenditure in connection with the restructuring. There is clearly no legal obligation, but there may
be a constructive obligation. IAS 37 states that a constructive obligation only exists if the group has
created valid expectations in other parties such as employees, customers and suppliers that the
restructuring will actually be carried out. As the group is still in the process of drawing up a formal
plan for the restructuring and no announcements have been made to any of the parties affected,
there cannot be an obligation to restructure. A board decision alone is not sufficient. Therefore no
provision should be made.
If the group starts to implement the restructuring or makes announcements to those affected after
the end of the reporting period but before the accounts are approved by the directors it may be
necessary to disclose the details in the financial statements as a non-adjusting post event after the
reporting period in accordance with IAS 10. This will be the case if the restructuring is of such
importance that non-disclosure would affect the ability of the users of the financial statements to
reach a proper understanding of the group’s financial position.
Operating lease
The lease contract appears to be an ‘onerous contract’ as defined by IAS 37 as the unavoidable
costs of meeting the obligations under it exceed the economic benefits expected to be received from
it.
Because the enterprise has signed the lease contract there is a clear legal obligation and the
enterprise will have to transfer economic benefits (pay the lease rentals) in settlement. Therefore,
the group should recognise a provision for the net present value of the remaining lease payments.
In principle, a corresponding asset may be recognised in relation to the future rentals expected to be
received, if these receipts are virtually certain. The current arrangement with the charity generates
only nominal rental income and so the asset is unlikely to be material enough to warrant recognition.
The chances of renting the premises at a commercial rent are less than 50% and so no further
potential rent receivable may be taken into account as the outcome is not virtually certain and so
recognition would not be prudent.
The financial statements should disclose the carrying amount of the onerous lease provision at the
end of the reporting period, a description of the nature of the obligation and the expected timing of
the lease payments. Disclosure should also be made of the contingent assets where the amount of
any expected rentals receivable from sub-letting are material and the likelihood is believed probable.
Legal proceedings
It is unlikely that the group has a present obligation to compensate the customer; therefore no
provision should be recognised. However, there is a contingent liability. Unless the possibility of a
transfer of economic benefits is remote, the financial statements should disclose a brief description
of the nature of the contingent liability, an estimate of its financial effect and an indication of the
uncertainties relating to the amount or timing of any outflow.
Environmental damage
It is clear that there is no legal obligation to rectify the damage. However, through its published
policies, the group has created expectations on the part of those affected that it will take action to do
so. There is, therefore, a constructive obligation to rectify the damage and a transfer of economic
benefits is probable.
The group must recognise a provision for the best estimate of the cost. As the most likely outcome is
that more than one attempt at re-planting will be needed, the full amount of Rs. 30 million should be
provided. The expenditure will take place sometime in the future, and so the provision should be
discounted at a pre-tax rate that reflects current market assessments of the time value of money and
the risks specific to the liability.
The financial statements should disclose the carrying amount at the end of the reporting period, a
description of the nature of the obligation and the expected timing of the expenditure. The financial
statements should also give an indication of the uncertainties about the amount and timing of the
expenditure.

© Emile Woolf International 238 The Institute of Chartered Accountants of Pakistan


Answers

14.14 MULTAN PETROCHEM LTD


(a) Provisions and contingencies

Environmental Legal Onerous Total


damage claims lease
At 1 Jan 2016 1,300,000 – 80,000 1,380,000
Unwinding of the
discount (8%) 104,000 6,400 110,400
Utilised in the year – – (15,000) (15,000)
Charge/(credit) to
statement of profit or
loss – 1,200,000 (6,000) 1,194,000
At 31 Dec 2016 (W) 1,404,000 1,200,000 65,400 2,669,400

Environmental damage
The provision in respect of the environmental damage relates to restoration of land following
the initial ground work undertaken to set up a new oil refinery. The company has an
advertised policy that it will restore all environmental damage caused by its business
operations. The provision is based on the estimated cost of reinstating the environmental
damage caused and is not likely to be paid until 2040.
Legal claims
During the year an explosion at one of the company’s oil extraction plants caused a number
of employees to suffer injury. This provision is to cover personal injury claims made by the
individuals concerned. The provision is based on lawyers’ best estimate of the likely amount
at which the claims can reasonably be settled. It is hoped that the claims will be settled in the
next financial year. It is expected that the full amount of these claims will be reimbursed by an
insurance company following their payment.
Onerous lease
The company has an ongoing lease obligation in respect of office space that is not being
utilised by the company. The outstanding lease liability at the year-end was Rs. 65,000 and
the lease has another four years to run. MPP has found a tenant for the office space on a six-
month short lease and this will reduce the outstanding obligation by Rs. 6,000 in 2017.
Contingent liability
Following the explosion at the oil extraction plant a number of employees have made claims
against the company for undue stress. Based on lawyers’ advice the company do not believe
that it is probable that a court case against the company will be brought. If such a case was to
be heard the estimated payout in total is Rs. 20,000.
Workings
Personal injury claims: 8 × 150,000 = 1,200,000
Onerous lease: (80,000 – 15,000) – 6,000 = 59,000
(b) Summary of amounts included in income statement for year ended 31 December 2016
Operating costs: Rs.
Movement in provision 394,000
Consultancy fees 12,000
Depreciation on oil refinery environmental damage (1,300,000 ÷ 52,000
25yrs)
Borrowing costs
Unwinding of the discount 110,400
Other operating income:
Insurance reimbursement 400,000

© Emile Woolf International 239 The Institute of Chartered Accountants of Pakistan


Financial accounting and reporting II

14.15 VIOLET POWER LIMITED


Rs. in
million
Assets carrying value as at June 30, 2016 (Asset)
Cost (Given) 6,570
20
Decommissioning liability on July 1, 2015 (780 / (1+0.08) ) 167
Depreciation for the year (321) Working 1
Adjustment for revision in provision for decommissioning cost 157 Working 2
6,573

Decommissioning liability on June 30, 2016 (1,021 / (1+0.06)19) 337

Working 1: Depreciation for the year (P&L)


Cost 6,570
Decommissioning liability on July 1, 2015 167
Residual value (320)
6,417
Depreciation (6,417 / 20) 321

Working 2: Increase in decommissioning liability during the year


ended June 30, 2016
Decommissioning liability on June 30, 2016 337
Less: Decommissioning liability on July 1, 2015 (167)
Less: Unwinding of interest for the year (167 x 8%) (13)
157

14.16 GOHAR LIMITED


(a) Determination of reportable segments

Chemicals Soda Ash Polyester Paint Pharma Total


-------------------------- Rs. in million --------------------------
Sales 1,790 216 227 247 252 2,732
Less: Inter-segment (38) - - - - (38)
sales
Sales to external 1,752 216 227 247 252 2,694
customers
Gross profit 1,101 117 48 26 31 1,323
Operating expenses (63) (57) (23) (16) (12) (171)
Profit before tax 1,038 60 25 10 19 1,152
Assets 637 444 115 127 132 1,455

© Emile Woolf International 240 The Institute of Chartered Accountants of Pakistan


Answers

Criteria for reporting segment Reporting segment of External sales


indentification indentified segment indentifying

1. 10% of sales i.e. Rs. 273,2 million Chemicals 65.30%

2. 10% of PBT i.e. 115.2 million - -

Further segment needs to be indetified as reportable segment’s external sale is less

3. Highest in term of sales and % of assets Pharm 9.22%


amount remaining segments

82.27%

(b) Disclosure in the financial statements of Gohar Limited 34


- OPERATING SEGMENT RESULTS

Chemicals Soda Ash Pharma Total

------------------------ Rs. in million ---------------------


Revenue from external customers 1,752 216 252 474 2,694
Inter segment revenue 38 - - - 38

Revenue from reportable segment 1,790 216 252 2,258

Other material information


Operating expenses 63 57 12 39 171
Segment profit before tax 1,038 60 19 35 1,152
Segment assets 637 444 132 242 1,455
Segment liabilities 442 355 98 202 1,097

34.1 - Reconciliation of reportable segment revenues, profit or loss, assets and liabilities

Reportable Other than Elimination Other Gohar


segment total reportable of inter- adjustments Limited's total
segment total segment
transactions

------------------------------ Rs. in million ------------------------------

Revenues 2,258 474 (38) - 2,694

Operating expenses 132 39 - 75 246

Segment profit before tax 1,117 35 (11) (75) 1,066

Segment assets 1,213 242 - 150 1,605

Segment liabilities 895 202 - 27 1,124

The reconciling items represents amounts related to corporate headquarter which are not
included in segment information

© Emile Woolf International 241 The Institute of Chartered Accountants of Pakistan


Financial accounting and reporting II

14.17 JAY LIMITED


(a) An operating segment is a component of an entity:

 That engages in business activities from which it may earn revenues and incur
expenses (including revenues and expenses relating to transactions with other
components of the same entity);

 Whose operating results are regularly reviewed by the entity’s chief operating decision
maker to make decisions about resources to be allocated to the segment and assess
the performance; and

 For which discrete financial information is available.


A business activity which has yet to earn revenues, such as a start up, is an operating
segment if it is separately reported on to the chief operating decision maker.

(b) As Jay Limited has both profit and loss making segments, the result of those in profit and
those in loss must be totaled to see which is the greater:

Profits (194+81+10) 285

Losses (22+63) (85)

200

So the 10% of profit or loss test must be applied by reference to Rs. 285 million.

Reportable
Segment Explanation
(Yes / No)

A Yes Because it generates more than 10% of revenue.

B No Because it fails to meet any of the criteria specified in IFRS-8

C Yes Because it generates more than 10% of revenue.

D Yes Because it has more than 10% of assets.

E Yes Because its losses are more than 10% of absolute profit.

Check the 75% test is satisfied: (705+300+90)/1,177 = 93%

14.18 GLADIATOR LIMITED


Impact on
Net Total Total
profit assets liabilities
----------- Rs. in million -----------

As per question 125.00 1,420.00 925.00


Revaluation of plant (W-1) 8.35
Increase in provision (W-1) 7.94
Revised Amounts 125.00 1,428.35 932.94

© Emile Woolf International 242 The Institute of Chartered Accountants of Pakistan


Answers

W-1: Revaluation of plant Rs. in million


Net Revalued amount 112.00
PV of revised dismantling cost (40 × 0.7938) 31.75
Gross revalued amount 143.75
Carrying amount as on 30 June 2017 (135.40)
Increase in value of plant 8.35
Increase in dismantling cost 10÷(1.08) 3 (7.94)
Revaluation surplus balance 3.15
(4.79)
Revaluation surplus 3.56

14.19 BRAVO LIMITED


Bravo Limited
Amounts as they would appear in the statements of financial position and comprehensive
income
For the year ended 30 September 2013
-------------- Rs. in million --------------
Decrease in decommissioning Finance/ P. P. & Decommission
liability: Depreciation expenses E - ing liability
Carrying value as at 30.09.2012
(130+19)÷20×(20–4.5) 115.48
[19×(1.1)4.5] 29.18
Deprecation: Oct. 2012-Mar. 2013
[(130+19)÷20×0.5] 3.73 (3.73)
Finance cost: Oct. 2012-Mar. 2013
[19×(1.1)5]- [19×(1.1)4.5] 1.42 1.42
Decrease due to revision in liability
(30.6–25) (5.60) (5.60)
Revised balance as at 1-4-2013 106.15 25.00
Deprecation: Apr-Sept. 2013
(106.15÷15×0.5) 3.54 (3.54)
Finance cost: Apr-Sept. 2013
{[25×(1.1)0.5]-25} 1.22 1.22
9.91 102.61 26.22

14.20 WASTE MANAGEMENT LIMITED


Journal entry

Dr. Cr.
Date Particulars Ref.
Rs. in million

31-03-11 PL Account (Depreciation exp) 70,000/8 8.750

Accumulated depreciation 8.750

PL Account (Unwinding of discount) 1 0.681

Site restoration liability (Unwinding of discount) 1 0.681

Revaluation surplus (Incremental depreciation) 1 0.461


Retained earnings (Incremental depreciation) 1 0.461

© Emile Woolf International 243 The Institute of Chartered Accountants of Pakistan


Financial accounting and reporting II

Date Particulars Ref. Dr. Cr.

PL account (Excess of increase in site restoration cost over


revaluation balance) 2.542-1.843 0.699

Revaluation surplus (Increase in site restoration cost) 2 1.843

Site restoration liability (Increase in site restoration cost) 2 2.542

12.434 12.434

Site
Revaluation
WORKING Ref. restoration
surplus
liability
01-04-05 PV of site restoration cost of Rs. 10 million at
10% discount rate 10/(1.1)10 3.855
31-03-06 Unwinding at 10% 0.386
31-03-07 Unwinding at 10% 0.424
31-03-07 Carrying value of the plant (80+3.855)*8/10 67.084
31-03-07 Revalued amount of the plant 70.000 2.916
Unwinding at 10% / Incremental dep.
31-03-08 (2.916/8) 0.467 (0.365)
31-03-09 Unwinding at 10% / Incremental dep. 0.513 (0.365)
5.645 2.186
31-03-09 Increase / (decrease) in liability / revaluation 5.066-
surplus on revision of discount rate to 12% 5.645 (0.579) 0.579
31-03-09 PV of site restoration cost of Rs. 10 million at
12% discount rate 10/(1.12)6 5.066# 2.765
Unwinding at 12% / Incremental dep.
31-03-10 (2.765/6) 0.608 (0.461)
31-03-11 Unwinding at 12% / Incremental dep. 1 0.681 (0.461)
6.355 1.843
31-03-11 Increase / (decrease) in liability relating to 8.897-
site restoration costs 6.355 2 2.542 (1.843)
31-03-11 PV of site restoration cost of Rs. 14 million at
-
12% discount rate 14/(1.12)4 8.897

© Emile Woolf International 244 The Institute of Chartered Accountants of Pakistan


Answers

CHAPTER 15 – ETHICAL ISSUES IN FINANCIAL REPORTING


15.1 ETHICAL ISSUES
The range of comments made by Arif raises questions over his ethical behaviour and professional
standards.
A chartered accountant should be unbiased when involved in preparing and reviewing financial
information. A chartered accountant should prepare financial statements fairly, honestly, and in
accordance with relevant professional standards and must not be influenced by considerations of
the impact of reported results.
Arif’s failings
Arif appears to be influenced by the need to achieve a specified level of profit. This is not
appropriate and calls his integrity into question.
In addition Arif’s professional competence seems to be suspect. His comment on not being up to
date on all of the little technicalities in IFRS s suggests that he has not maintained a level of
professional competence appropriate to his professional role.
ICAP members have a responsibility to engage in continuing professional development in order to
ensure that their technical knowledge and professional skills are kept up to date. Arif should seek
continuing professional development activities and improve his knowledge on ethical standards.
Furthermore, it might be expected that as Waheed’s superior he should set an example to Waheed
and guide him in his responsibilities. Clearly this is not happening.
As a member of ICAP Arif should be aware of the ICAP code of ethics. Arif should know of the
danger of self-interest threats and intimidation threats to himself and to others. His attempt to
influence the outcome of a fellow professional by applying such a threat to that individual is very
unprofessional.
Waheed’s ethical issues
Waheed faces a self-interest threat, in that there is the possibility of a bonus provided the earnings
per share figure remains the same as last year. Arif has also suggested that she can influence the
Board’s decision over employing him as a replacement finance director – another self-interest threat
to Waheed. Both of these threats must be ignored.
Arif’s comments imply that his application of professional responsibility is lacking. This may extend
into the way in which the current financial statements have been prepared. Waheed must be very
careful (as always) to carry out the review with all due care.
Waheed should first discuss his recommendations with Arif and remind his of his professional
responsibilities to ensure that the accounting standards are correctly followed. If the financial
statements are found to contain errors or incorrect accounting treatment then they must be
amended. If Arif refuses to amend the draft financial statements if necessary Waheed should
discuss the matter with other board members (including non- executives and the audit committee, if
possible). Further action might include consulting with ICAP.
15.2 SINDH INDUSTRIES LTD
(a) Financial reporting issues
Revenue
IFRS 15 Revenue from contract with customer sets out the rules to be followed in recognising
revenue.
The fact that the customer cannot cancel the contract is not relevant to the recognition of
revenue. Revenue from providing a service is recognised according to the stage of completion
subject to satisfying criteria set out in IFRS 15. In the absence of other information the
revenue in this contract should be recognised over the life of the contract as time progresses.
As the contract was only signed just before the year end, none of the revenue can be
recognised in 2015.
The credit for the amount received should be recognised as a liability. This represents the
obligation that the company has to provide the service over the next two years.

© Emile Woolf International 245 The Institute of Chartered Accountants of Pakistan


Financial accounting and reporting II

The fact that the customer cannot cancel the contract is not relevant to the recognition of
revenue. If Sindh Industries failed to provide the service they would be sued for restitution.
Therefore the revenue can only be recognised as the service is provided.
New factory
Borrowing costs directly attributable to construction of an asset which necessarily takes a
substantial period to get ready for its intended use should be capitalised as part of the cost of
that asset under IAS 23 Borrowing Costs. IAS 23 states that the capitalisation of borrowing
costs should commence when three conditions are all met for the first time: borrowing costs
are being incurred, expenditure is being incurred and activities to prepare the asset are being
undertaken. Although borrowing costs were incurred throughout the year and expenditure was
incurred from 1 February 2015 (the date the land was purchased), construction only started
on 1 June 2015. Therefore this is the date on which capitalisation commences.
Capitalisation ceases when substantially all of the activities required to make the asset ready
for use/sale have been completed, that is on 30 September 2015. (The actual date on which
the factory was brought into use is irrelevant.) Therefore the period of capitalisation should be
four months.
Where construction is financed from general borrowings, the calculation of the amount to be
capitalised should be based on the weighted average cost of borrowings. This is:
(Rs.1,000,000 × 9.75%) + (Rs.1,750,000 × 10%) + (Rs.2,500,000 × 8%)/ (Rs.1,000,000 +
Rs.1,750,000 + Rs.2,500,000) = 9%
Therefore the amount capitalised should be 9% × Rs.4.5 million (land Rs.1.8 million plus
construction costs Rs.2.7 million) × 4/12 = Rs.135,000. The total cost of the factory should be
measured at Rs.4,635,000 (Rs.1.8 million plus Rs.2.7 million, plus Rs.135,000). The amount
that has been recognised in the statement of financial position should be reduced by
Rs.315,000 (Rs.450,000 – Rs.135,000). Finance costs recognised in profit or loss should be
increased by Rs.315,000.
Land should not be depreciated because it has an indefinite life. Under IAS 16 Property, Plant
and Equipment depreciation charges should start when the asset becomes available for use,
from 1 October 2015 in this case.
Depreciation of Rs.35,000 ((Rs.2.7 million, plus (Rs.135,000 × 2.7/4.5) ÷ 20) × 3/12) should
be recognised in profit or loss for the year ended 31 December 2015 and the carrying amount
of the asset reduced by the same amount to Rs.4.6 million.
Useful life of the blast furnace
Depreciation of the blast furnace has been based on an estimated useful life of 20 years. This
is at variance with a report by a qualified expert. The asset valuation specialist treats the
furnace as being made up of two components, the main structure and the lining, which must
be replaced at regular five yearly intervals over the life of the asset. This is the approach
required by IAS 16. The uncertainties inherent in business mean that many items in financial
statements cannot be measured with certainty, but estimates should always be made using
the most up to date and reliable information. Where estimates have been prepared by
professionals with relevant qualifications, then it is nearly always most appropriate to use
those estimates. Therefore in accordance with the valuer’s report the main structure of the
furnace should be depreciated over 15 years from 1 January 2015 and the lining should be
depreciated over five years from that date.
The reassessment of the estimated lives of assets is a change in accounting estimate, rather
than a change in accounting policy (IAS 8 Accounting Policies, Changes in Accounting
Estimates and Errors). Changes in accounting estimate should be dealt with on a prospective
basis. This is achieved by including the effect of the change in profit or loss in current and
future periods. The additional depreciation should be calculated as:

© Emile Woolf International 246 The Institute of Chartered Accountants of Pakistan


Answers

Rs.000
Revised depreciation: main structure 140
((Rs.3.5m – Rs.1.4m)/15 years)
lining (Rs.1.4m/5 years) 280
420
Current depreciation (Rs.3.5m/20 years) (175)
Additional depreciation 245

IAS 8 requires the disclosure of the nature and amount of the effect of the change in the
estimate of useful lives on the profit for the year.
(b) Revised financial statements
Statement of profit or loss extract for the year ended 31 December 2015

Borrowing Blast
Draft Revenue costs furnace Revised
Rs.000 Rs.000 Rs.000 Rs.000 Rs.000
Profit before tax 2,500 (1,000) (315)+ (35) (245) 905

Statement of financial position at 31 December 2015

Borrowing Blast
Draft Revenue costs furnace Revised
Rs.000 Rs.000 Rs.000 Rs.000 Rs.000
Non-current assets
Property, plant and equipment 12,000 (315) + (35) (245) 11,405
Current assets 3,500 3,500
Total assets 15,500 14,905

Share capital 2,000 2,000


Retained earnings 6,000 (1,000) (315) + (35) (245) 4,405
Equity 8,000 6,405
Non-current liabilities 5,000 500 5,500
Current liabilities 2,500 500 3,000
Total equity and liabilities 15,500 15,905

(c) Ethical issues


It is noticeable that all the adjustments required reduce profit. This and the background to the
previous finance director’s resignation suggest serious problems.
It is not clear who actually prepared the draft financial statements. If they were prepared by
more junior staff in the absence of a finance director, some of the adjustments (for example,
the calculation of borrowing costs to be capitalised) could be the result of genuine errors or
lack of accounting knowledge. However, it seems reasonably clear that the managing director
has attempted to influence the treatment of the revenue and the estimated useful life of at
least one significant non-current asset. (Note: the directors have reviewed the useful lives of
several items of plant and machinery and it is possible that other assets besides the furnace
are being depreciated over unrealistically long periods.)

© Emile Woolf International 247 The Institute of Chartered Accountants of Pakistan


Financial accounting and reporting II

It seems almost certain that the previous finance director resigned as a result of pressure from
the managing director (and possibly from other members of the Board) to present the financial
statements in a favourable light. The directors intend to seek a stock market listing in the near
future. Therefore they have clear motives for manipulating the profit figure and also (perhaps)
for making controversial decisions before the financial statements come under much greater
scrutiny as a result of the listing. The job title of financial controller is also significant. It
suggests that the role has been downgraded and that the person holding it has less authority
than the rest of the Board.
Possible courses of action:
 Discuss with the managing director the financial reporting standards that apply to the
transactions and explain the implications of non-compliance. If the managing director is
himself a member of a professional body then it might be worth pointing out to him that
he himself is bound by an ethical code.
 Advise him that as a Chartered Accountant you are bound by the ICAP code of ethics,
and that you would not be prepared to compromise your views of the figures he has
prepared for career advancement.
 Consider speaking to the other directors (or audit committee if there is one) and
seeking their support.
 If all of these actions produce a negative response then it would be appropriate to
consult the ICAP ethical handbook and/or the Institute.
 If all else fails then consider seeking alternative employment.
15.3 CHARMING LIMITED
In the given situation, CFO may be in breach of :
i. Principle of professional behavior:
This principle imposes an obligation on all chartered accountants to avoid any action that the
chartered accountant knows or should know may discredit the profession. CFO should have
avoided discussing his personal interest in official meeting.
ii. Principle of objectivity:
Chartered Accountant should not compromise their professional or business judgment because
of bias, conflict of interest or the undue influence of others. In this circumstance, he has
compromised his professional and business judgment due to his personal interest as he
requested the EVP to consider application of his son who has applied for house financing in
USB.
iii. Principle of integrity:
Chartered Accountant should be straight forward and honest in all professional and business
relationship. It seems that CFO may be inclined to accept higher mark-up rate as compared to
existing rate being paid by CL, resulting breach of integrity.
Intimidation threat faced by Mr. Umer
Umer may face intimidation threat from his superior if he would raise his objection on acceptance
of higher mark-up rate offered by the Bank specially where his superior i.e. Abid is a relative of
principal shareholder too.
Available safeguards
If this threat is significant Umer should consult with superiors within the organization in order to
eliminate or reduce it to an acceptable level.
Where it is not possible to reduce the threats to an acceptable level, Umer:
i. should refuse to associate with this financing arrangement.
ii. should consider informing appropriate authorities like Audit Committee / CEO.
iii. seek legal advice or may resign.

© Emile Woolf International 248 The Institute of Chartered Accountants of Pakistan


Answers

15.4 MEHRAN LIMITED


Chartered Accountants should be straight forward and honest in all professional and business
relationships. Since the CEO advised Usman to process the payment about which Usman believes
that the said payment is unreasonable and would be made to obtain a sales contract, therefore he is
in breach of principle of integrity and professional behavior.
In the given circumstances, the decision of CEO may also induce lack of objectivity due to the
expected bonuses to the management.

Self interest threat faced by Usman


Usman might get influenced by the CEO due to the expected bonus therefore he might process the
payment in his own self interest.

Intimidation threat faced by Usman


Usman may have to leave this job if the disagreement continues.

Available safeguards
Where it is not possible to reduce the threats to an acceptable level, Usman:
i. should refuse to sign the cheque / refuse to associate with the transaction.
ii. should consider informing appropriate authorities like Audit Committee.
iii. seek legal advice or may resign.

15.5 MNZ LIMITED


In given situation, CFO is in breach of :
i. Principle of integrity:
Chartered Accountant should be straight forward and honest in all professional and business
relationship. Since he asked Accounts manager to identify the areas where through adjustments,
profit may be reported on higher side, he has breached the principle of integrity.
ii. Principle of professional behaviour:
This principle imposes an obligation on all chartered accountants to avoid any action that the
chartered accountant knows or should know may discredit the profession. Since CFO asked
Accounts Manager for booking the adjustments to increase the current year profit, which have a
negative effect on the reputation of the profession.
iii. Principle of objectivity:
Chartered Accountant should not compromise their professional or business judgment because
of bias, conflict of interest or the undue influence of others. In this circumstance, he has
compromised his professional and business judgment due to biasness.
Self interest threat faced by Mr. Atif
Self interest threat occurs as a result of financial or other interest of members or their immediate
family member. In this case, he has been told by the CFO that he would be given an additional
bonus this year so he faces self interest threat.
Available safeguards
If this threat is significant Atif should consult with superiors within the organisation in order to
eliminate or reduce it to an acceptable level.
Where it is not possible to reduce the threats to an acceptable level, Atif:
i. should refuse to remain associated with information which is or may be misleading
ii. should consider informing appropriate authorities keeping in view the confidentiality and the
legal requirements, if issuance of misleading information is either significant or persistent.
iii. seek legal advice or may resign.

© Emile Woolf International 249 The Institute of Chartered Accountants of Pakistan


Financial accounting and reporting II

15.6 UNIQUE ENGINEERING LIMITED


Mr. Zia breached the following fundamental principles of ICAP code of ethics:
i. Confidentiality
Under the Code of Ethics, member must respect the confidentiality of information acquired as a
result of professional and business relationship. Confidential information acquired should not be
used for the personal advantage by a member.
In the above scenario, Mr Zia has breached the principle of confidentiality by using the
confidential information for the personal advantage since the information was not publicly
available.
ii. Professional behaviour
Under the Code of Ethics, member must comply with relevant laws and regulations and should
avoid any action which discredits the profession.
Since it can be a non compliance of laws and regulation, he may be in breach of the principle of
professional behaviour.
Potential threats involved in the circumstances:
Self interest threat
Since Mr. Zia is part of a team which is negotiating the price of the shares and he has
purchased shares in the name of his wife and son, it creates self interest threat and he would be
reluctant to take any decision that would be against his own interest.
15.7 ALI AND BASHIR
The existence of threats to fundamental principles will depend on following factors:
 Whether financing from other banks is available at lower mark up;
 Whether it is feasible to borrow @15% for the expansion.
If financing from other banks is available or it may not be feasible to finance the project at the rate of
15%, and still MD is pressurizing the CFO to obtain financing at higher rate of markup the MD may
be in breach of :

i. Principle of objectivity
It can be a bias decision on part of MD, as he may be favoring his friend who is the president of
the bank or may have any other interest in taking loan from that particular bank.

ii. Principle of integrity


MD may be in breach of principle of integrity because he is asking CFO to manipulate the
financial information.

Potential threat to CFO along with safeguards:


Preparation of financial information as per the instructions of MD, will result in intimidation threat
to integrity and objectivity.
Identified threat is significant as the CFO is being instructed from the highest level of
management. In order to reduce the threat to an acceptable level, the following safeguards
should be applied.
 Consult with superiors such as audit committee or those charged with governance or with a
relevant professional body.
 Where it is not possible to reduce the threat to an acceptable level, CFO shall refuse to be
remain associated with the financial information.
 CFO may consider to obtain legal advice or may consider resigning from the post of CFO.

© Emile Woolf International 250 The Institute of Chartered Accountants of Pakistan


Answers

15.8 FARAZ
In the given situation, Faraz may face following threats:

i. Self-interest threat
Self-interest threat occurs as Faraz has been told by the CEO that he would be promoted to
CFO.

ii. Intimidation threat


Faraz may quit this job if he would not confirm the draft financial statement as per CEO’s
instructions.

Available safeguards:
Where it is not possible to reduce the threats to an acceptable level, Faraz:
(i) should refuse to remain associated with information which is or may be misleading
(ii) should consider to consult with superiors such as audit committee or those charged with
governance or with a relevant professional body.
(iii) seek legal advice or may resign.

© Emile Woolf International 251 The Institute of Chartered Accountants of Pakistan


Financial accounting and reporting II

© Emile Woolf International 252 The Institute of Chartered Accountants of Pakistan

You might also like